Exit Exam 2023 qs :)

Pataasin ang iyong marka sa homework at exams ngayon gamit ang Quizwiz!

*********The nurse assessing a client who reports falling 2 days ago and has a history of gouty arthritis that is controlled with allopurinol. The client states the left knee is swollen and extremely pain to touch. Which instruction should the nurse include in the discharge teaching? A. Decrease consumption of red meat and most seafood B. Substitute natural fruit juices for carbonated drinks C. Limit use of mobility equipment to avoid muscle atrophy D. Use electric heating pad when pain is at its worse

A. Decrease consumption of red meat and most seafood

An increased number of elderly persons are electing to undergo a new surgical procedure which cures glaucoma. Which effect is the nurse likely to note as a result of this increase in glaucoma surgeries? A. Decreased prevalence of glaucoma in the population B. Increased incidence of glaucoma in the population C. Decreased morbidity in the elderly population D. Increased mortality in the elderly population

A. Decreased prevalence of glaucoma in the population

****Prolonged exposure to high concentrations of supplemental oxygen over several days can cause which pathophysiological effect? A. Disrupted surfactant production B. Metabolic acidosis C. Aphasia and memory loss D. Deep sleep or coma

A. Disrupted surfactant production

********An older client's daughter calls the home health nurse and reports that her mother has become forgetful and is very confused at night. The daughter stated that her mother's behavior changed suddenly a few days ago and is now getting worse. Which actions should the nurse take? (SATA) A. Encourage increased intake of high protein foods B. Instruct the daughter to check her mother's temperature C. Review the client's current food and medication allergies D. Ask if the mother is experiencing any pain with urination E. Determine if the mother has recently experienced a fall.

A. Encourage increased intake of high protein foods B. Instruct the daughter to check her mother's temperature D. Ask if the mother is experiencing any pain with urination

*******A middle-aged client, admitted to a critical care unit several weeks ago because of serious injuries sustained in a motor vehicle accident, is currently in stable condition. Based on the client's age and recent life-threatening crisis, which intervention should the nurse implement? A. Encourage the client to reflect on personal goals and priorities B. Allow long periods of uninterrupted rest in order to reduce fatigue C. Discuss the cause of the accident with the client and his family D. Provide a routine schedule of activities to facilitate trust

A. Encourage the client to reflect on personal goals and priorities

********A male client with cirrhosis has jaundice and pruritus. He tells the nurse that he has been soaking in hot baths at night with no relief of his discomfort. Which action should the nurse take? A. Encourage the client to use cooler water and apply calamine lotion after soaking B. Obtain a PRN prescription for an analgesic that the client can use for symptom relief C. Suggest that the client take brief showers and apply oil-based lotion after showering D. Explain that the symptoms are caused by liver damage and cannot be relieved

A. Encourage the client to use cooler water and apply calamine lotion after soaking

**********An adult female client tells the nurse that though she is afraid her abusive boyfriend might one day kill her, she keeps hoping that he will change. Which action should the nurse take first? A. Explore client's readiness to discuss the situation B. Determine the frequency and type of client's abuse C. Report the finding to the police department D. Discuss treatment options for abusive partners

A. Explore client's readiness to discuss the situation

*****The nurse is taking the blood pressure measurement of a client with Parkinson's disease. Which information in the client's admission assessment is relevant to the nurse's plan for taking the blood pressure reading? (Select all the apply) A. Frequent syncope B. Occasional nocturia C. Flat affect D. Blurred vision E. Frequent drooling

A. Frequent syncope C. Flat affect D. Blurred vision

*****The nurse assess a client being treated for Herpes zoster (shingles). Which assessments should the nurse include when evaluating the effectiveness of the the treatment? (Select all that apply) A. Functional ability B. Skin integrity C. Pain scale D. Bowel sounds E. heart sounds

A. Functional ability B. Skin integrity c.pain scale

************A female client presents in the emergency department and tells the nurse that she was raped last night. Which question is most important for the nurse to ask? A. Has she taken a bath since the rape occurred? B. Is the place where she lives a safe place? C. Does she know the person who raped her? D. Did she report the rape to the police department?

A. Has she taken a bath since the rape occurred?

*****When taking a health history, which information collected by the nurse correlates most directly to a diagnosis of chronic peripheral arterial insufficiency? A. History of intermittent claudication B. A positive Brodie-Trendelenburg test C. Ankle ulceration and edema D. A serum cholesterol level of 250mg/dl (6.47mmol/L)

A. History of intermittent claudication

********During an admission assessment, a client reports currently using heroin. Which information is most important for the nurse to consider in the plan of care? A. History of suicide attempts B. Feelings of disorientation C. Undiagnosed social anxiety symptoms (SAD) D. Family history of schizophrenia

A. History of suicide attempts

********The nurse is caring for a client with the sexually transmitted infection (STI) chlamydia. The client reports having sex with someone who had many partners. Which response should the nurse provide? A. Inform that follow-up may end after the treatment is finished B. Reassure that complications will not occur if the infection is treated C. Notify that persons with STIs are reported to local health departments D. Explain how the infection is transmitted and the health risks involved

A. Inform that follow-up may end after the treatment is finished.

************A child newly diagnosed with sickle cell anemia (SCA) is being discharged from the hospital. Which information is most important for the nurse to provide the parents prior to discharge? A. Instructions about how much fluid the child should drink daily. B. Signs of addiction to opioid pain medications C. Information about non-pharmaceutical pain relief measures D. Referral for social services for the child and family

A. Instructions about how much fluid the child should drink daily

************The nurse is caring for a seated client who is experiencing a tonic-clonic seizure. Which actions should the nurse implement? (Select all that apply) A. Loosen restrictive clothing B. Insert a bite block C. Ease the client to the floor D. Note the duration of the seizure E. Restrain the client

A. Loosen restrictive clothing C. Ease the client to the floor D. Note the duration of the seizure

*******A client with Type 1 diabetes mellitus and a large draining ulcer of the right foot is admitted with a suspected Staphylococcus aureus infection. Which interventions should the nurse implement? (Select all that apply) A. Monitor the client's white blood cell count B. Explain the purpose of a low bacteria diet C. Send wound drainage for culture and sensitivity D. Institute contact precautions for staff and visitors E. Use standard precautions and wear a mask

A. Monitor the client's white blood cell count C. Send wound drainage for culture and sensitivity D. Institute contact precautions for staff and visitors

******Following a cardiac catheterization and placement of a stent in the right coronary artery, the nurse administers prasugrel, a platelet inhibitor, to the client. To monitor for adverse effects from the medication, which assessment is most important for the nurse to include in this client's plan of care? A. observe color of urine B. Measure body temperature C. Assess skin turgor D. Check for pedal edema

A. Observe color of urine

******A client taking clopidogrel reports the onset of diarrhea. Which nursing action should the nurse implement first? A. Observe the appearance of the stool B. Assess the elasticity of the client's skin C. Review the client's laboratory values D. Auscultate the client's bowel sounds

A. Observe the appearance of the stool

*****The nurse is managing the care of a client with Cushing's syndrome. Which interventions should the nurse delegate to the unlicensed assistive personnel (UAP)? (Select all that apply) A. Report any client complaint of pain or discomfort B. Evaluate the client for sleep disturbances C. Assess the client for weakness and fatigue D. Weigh the client and report any weight gain E. Note and report the client's food and liquid intake during meals and snacks

A. Report any client complaint of pain or discomfort D. Weigh the client and report any weight gain E. Note and report the client's food and liquid intake during meals and snacks

********Two days prior to discharge from the rehabilitation facility, the nurse is teaching a client who is recovering from Guillain-Barre syndrome about home care. Which actions should the nurse include when providing discharge teaching to the client and spouse? (Select all that apply) A. Review safe transfer strategies B. Develop a nutritional plan C. Help identify community support D. Initiate a rigorous exercise routine E. Provide cooking instructions

A. Review safe transfer strategies B. Develop a nutritional plan C. Help identify community support

******A toddler presenting with a history of intermittent skin rashes, hives, abdominal pain, and vomiting that occurs after ingesting of milk products arrives to the clinic accompanied by the parents. Which type of testing should the nurse provide education to the toddler's family about? A. Serum immunoglobulin E (IgE) B. Intradermal test C. Atopy patch test D. Placebo-controlled food challenge

A. Serum immunoglobulin E (IgE)

*********The nurse is caring for client with flail chest secondary to 3 right rib fractures after sustaining a fall from a ladder. The client is anxious, but stable with an oxygen saturation of (SpO2) 93%. Which action should the nurse take? A. Splint affected side B. Insert nasal airway C. Coach through taking deep breaths D. Apply a non-rebreather mask

A. Splint affected side

******A nurse receives report on a client who is four hours post-total abdominal hysterectomy. The previous nurse reports that it was necessary to change the client's perineal pad hourly and that it is again saturated. The previous nurse also reports that the client's urinary output has decreased. Which action should the nurse implement first? A. Evaluate the skin turgor B. Assess for weakness or dizziness C. Change the perineal pad D. Measure the urinary output

B. Assess for weakness or dizziness

*******A client with chronic kidney disease has an arteriovenous fistula in the left forearm. Which observation by the nurse indicates that the fistula is patent? A. Assessment of a bruit on the left forearm B. Auscultation of a thrill on the left forearm C. The left radial pulse is 2+ bounding.

B. Auscultation of a thrill on the left forearm

*******The nurse is evaluating the diet teaching of a client with hypertension. What dinner selection indicates that the client understands the dietary recommendations for hypertension? A. Grilled steak, baked potato with sour cream, green beans, coffee, and raisin cream pie B. Baked pork chops, applesauce, corn on the cob, 1% milk, and key-lime pie C. Tomato soup, grilled cheese sandwich, pickles, skim milk, and lemon meringue pie D. Beef stir fry, fried rice, egg drop soup, diet soda, and pumpkin pie

B. Baked pork chops, applesauce, corn on the cob, 1% milk, and key-lime pie

******Which self care measure is most important for the nurse to include in the plan of care of a client recently diagnosed with type 2 diabetes mellitus? A. Self-injection techniques B. Blood glucose monitoring C. Diabetic diet meal planning D. A realistic exercise plan

B. Blood glucose monitoring

******The nurse is providing discharge teaching to the parents of a 13 month old child who underwent repair for an atrial septal defect. The healthcare provider prescribes aspirin and an antibiotic for the first 6 months postoperatively to prevent infective endocarditis (IE). What information is most important for the nurse discuss with the parents about the child's recovery and prevention of IE? A. Refer the mother to the healthcare provider to discuss infective endocarditis B. Brush the child's teeth every day and ensure the child receives regular dental followup C. Give the child acetaminophen for pain or fever and visit the surgeon for follow-up D. Monitor the child for regular bowel movements and urine output that exceeds intake

B. Brush the child's teeth every day and ensure the child receives regular dental followup

*******The nurse is caring for a client with heart failure. Which method is used in computing the cardiac index to measure how the client's heart is functioning? A. Mean arterial pressure minus right atrial pressure B. Cardiac output divided by body surface area C. Stroke volume divided by end diastolic volume D. Stroke volume multiplied by heart rate

B. Cardiac output divided by body surface area

*******The nurse assesses a client one hour after starting a transfusion of packed red blood cells and determines that there are no indications of a transfusion reaction. What instruction should the nurse provide the unlicensed assistive personnel (UAP) who is working with the nurse? A. Notify the nurse when the transfusion has finished, so further client assessment can be done B. Continue to measure the client's vital signs every thirty minutes until the transfusion is complete C. Monitor the client carefully for the next three hours and report the onset of a reaction immediately D. Since a reaction did not occur, the priority is to maintain client comfort during the transfusion

B. Continue to measure the client's vital signs every thirty minutes until the transfusion is complete

The nurse is evaluating the chest drainage system of a client with a chest tube inserted to treat a left hemothorax. Which finding requires intervention by the nurse? A. Rise and fall of water level with respiration B. Continuous bubbling in the water-seal chamber C. Total fluid level in water-seal chamber unchanged D. An average collection of 50 mL/hr drainage

B. Continuous bubbling in the water-seal chamber

*********While caring for a client's postoperative dressing, the nurse observes purulent drainage at the wound. Before reporting this finding to the healthcare provider, the nurse should review which of the client's laboratory values? A. Serum albumin B. Culture for sensitive organisms C. Serum blood glucose level D. Creatinine level

B. Culture for sensitive organisms

*****An S3 heart sound is auscultated in a client in her third trimester of pregnancy. What intervention should the nurse take? A. Prepare the client for an echocardiogram B. Document in the client's record C. Notify the healthcare provider D. Limit the client's fluids

B. Document in the client's record

********The nurse is teaching the client about home care after surgery for an ileal conduit placement. When reviewing the information, which statement should the nurse recognize as needing additional education? A. report presence of mucus in the urine B. Empty pouch when it is half full C. Look at the stoma when replacing appliance D. Anticipate shrinking of the stoma

B. Empty pouch when it is half full

***********An adult client is admitted to the emergency department after falling from the ladder. While waiting to have a computed tomography (CT) scan, the client requests something for a severe headache. When the nurse offers a prescribed dose of acetaminophen, the client asks for something stronger. Which intervention should the nurse implement? A. Review client's history for use of illicit drugs B. Explain the reason for using only non-narcotics C. Assess client's pupils for their reaction to light D. Request that the CT scan be done immediately

B. Explain the reason for using only non-narcotics

*******A client with type 2 diabetes mellitus arrives to the clinic reporting episodes of weakness and palpitations. Which finding should the nurse identify may indicate an emerging situation? A. Potassium 3.5 mEq/L B. Fingertips feel numb C. Sodium 135 mEq/L D. Cervical spine stiffness

B. Fingertips feel numb

********When conducting diet teaching for a client who was diagnosed with a myocardial infarction, which snack foods should the nurse encourage the client to eat? (Select all that apply) A. Fresh vegetables with mayonnaise dip B. Fresh turkey slices and berries C. Chicken bouillon soup and toast D. Soda crackers and peanut butter E. Raw unsalted almonds and apples

B. Fresh turkey slices and berries E. Raw unsalted almonds and apples

*******A client asks the nurse for information about how to reduce risk factors for benign prostatic hyperplasia (BPH). Which information should the nurse provide? A. Consume a high protein diet B. Increase physical activity C. Take vitamin supplements D. Obtain a prostate-specific antigen blood level test

B. Increase physical activity

****An unlicensed assistive personnel (UAP) is assigned to ambulate a client with influenza who has droplet precautions implemented. The UAP requests a change in assignment, stating the reason of having not been fitted yet for a N95 respirator mask. Which action should the nurse take? A. send the UAP to be fitted for a particulate filter mask immediately so she can provide care to this client. B. Instruct the UAP that a standard face mask is sufficient for the provision of care for the assigned client C. Before changing assignments, determine which staff members have fitted particulate filter masks D. Advise the UAP to wear a standard face mask to take vital signs, and then get fitted for a filter mask before providing personal care

B. Instruct the UAP that a standard face mask is sufficient for the provision of care for the assigned client

******Which intervention should the nurse include in the plan of care for a child with tetanus? A. Encourage coughing and deep breathing B. Minimize the amount of stimuli in the room C. Reposition from side to side every hour D. Open window shades to provide natural light

B. Minimize the amount of stimuli in the room

*****The nurse is reviewing the diagnostic tests prescribed for a client with a positive skin test. Which subjective findings reported by the client supports the diagnosis of tuberculosis? A. Barking cough and vomiting B. Mucopurulent cough and night sweats C. Dry cough and chest tightness D. Chronic cough and fatty stools

B. Mucopurulent cough and night sweats

**********A client with bacterial meningitis is receiving phenytoin. Which assessment finding indicates to the nurse that the client is experiencing a therapeutic response?

B. Normal electroencephalogram after drug administration

********In evaluating the effectiveness of a postoperative client's intermittent pneumatic compression devices, which assessment is most important for the nurse to complete? A. Monitor the amount of drainage from the client's incision B. Observe both lower extremities for redness and swelling C. Evaluate the client's ability to use an incentive spirometer D. Palpate all peripheral pulse points for volume and strength

B. Observe both lower extremities for redness and swelling

*******The nurse is demonstrating correct transfer procedures to the unlicensed assistive personnel (UAP) working on a rehabilitation unit. The UAP asks the nurse how to safely move a physically disabled client from the wheelchair to a bed. Which action should the nurse recommend? A. apply a gait belt around the client's waist once a standing position has been assumed B. Place the client's locked wheelchair on the client's strong side next to the bed C. Pull the client into position by reaching from the opposite side of the bed D. Hold the client at arm's length while transferring to better distribute the body weight

B. Place the client's locked wheelchair on the client's strong side next to the bed

*****The nurse is providing education to a client who experiences recurrent levels of moderate anxiety to situations and perceived stress. In addition to information about prescribed medication and administration, which instruction should the nurse include in the teaching? A. Find outlets for more social interaction B. Practice using muscle relaxation techniques C. Center attention on positive upbeat music D. Think about reasons the episodes occur

B. Practice using muscle relaxation techniques

*******An older client with a long history of coronary artery disease (CAD), hypertension (HTN), and heart failure (HF) arrives in the Emergency Department (ED) in respiratory distress. The healthcare provider prescribes furosemide IV. Which therapeutic response to furosemide should the nurse expected in the client with acute HF? A. Increased cardiac contractility B. Reduced preload C. Relaxed vascular tone D. Decreased afterload

B. Reduced preload

******When providing client care the nurse identifies a problem and develops a related clinical question. Next, the nurse intends to gather evidence so that the decision-making process in response to the problem and clinical question is evidence-based. When gathering evidence, which consideration is most important? A. Past experience with similar problems B. Relevance to the situation C. Related personal values D. Frequency that the problem occurs

B. Relevance to the situation

*********Three days after initiating parenteral fluids for a newborn with a ventricular septal defect (VSD), the nurse assesses an increase in heart rate and blood pressure. Which intervention is most important for the nurse to implement? A. View the graph of daily weights B. Restrict intake of oral fluids C. Assess bilateral lung sounds D. Decrease IV flow rate

B. Restrict intake of oral fluids

*******A client with type 2 diabetes mellitus is admitted for frequent hyperglycemic episodes and a glycosylated hemoglobin (A1C) of 10%. Insulin glargine 10 units subcutaneously once a day at bedtime and a sliding scale of insulin aspart every 6h are prescribed. What actions should the nurse include in this client's plan of care? (Select all that apply) A. Do not contaminate the insulin aspart so that it is available for IV use B. Review with the client proper foot care and prevention of injury C. Teach subcutaneous injection technique, site rotation, and insulin management D. Coordinate carbohydrate controlled meals at consistent times and intervals. E. Mix bedtime dose of insulin glargine with insulin aspart sliding scale dose F. Fingerstick glucose assessments every 6h with meals

B. Review with client proper foot care and prevention of injury C. Teach subcutaneous injection technique, site rotation, and insulin management D. Coordinate carbohydrate controlled meals at consistent times and intervals F. Fingerstick glucose assessments every 6h with meals

*********The nurse is completing the admission assessment of a 3-year old who is admitted with bacterial meningitis and hydrocephalus. Which assessment finding is evidence that the child is experiencing increased intracranial pressure (ICP)? A. Tachycardia and tachypnea B. Sluggish and unequal pupillary responses C. Increased head circumference and bulging fontanels D. Blood pressure fluctuations and syncope

B. Sluggish and unequal pupillary responses

********A client is recovering in the critical care unit following a cardiac catheterization. IV nitroglycerin and heparin are infusing. The client is sedated but responds to verbal instructions. After changing positions, the client complains of pain at the right groin insertion site. What action should the nurse implement? A. Check femoral site for hematoma formation B. Stimulate the client to take deep breaths C. Evaluate the integrity of the IV insertion site D. Assess distal lower extremity capillary refill

B. Stimulate the client to take deep breaths

*****In assessing a client with type 1 diabetes mellitus, the nurse notes that the client's respirations have changed from 16 breaths/min with a normal depth to 32 breaths/min and deep, and the client become lethargic. Which assessment data should the nurse obtain next? A. Temperature B. Breath sounds C. Blood glucose D. White blood cell count

C. Blood glucose

********A nurse who is working in the emergency department triage area is presented with four clients at the same time. The client presenting with which symptoms requires the most immediate intervention by the nurse? A. One inch bleeding laceration on the chin of crying 5 year old B. Low grade fever, headache and malaise for the past 72 hours C. Chest discomfort one hour after consuming a large, spicy meal D. Unable to bear weight on the left food, with swelling and bruising

C. Chest discomfort one hour after consuming a large, spicy meal

*****The nurse implements a tertiary prevention program for type 2 diabetes in a rural health clinic. Which outcome indicates that the program was effective? A. Only 30% of clients did not attend self-management education sessions. B. More than 50% of at-risk clients were diagnosed early in their disease process C. Clients who developed disease complications promptly received rehabilitation D. Average client scores improved on specific risk factor knowledge tests

C. Clients who developed disease complications promptly received rehabilitation

*****The nurse is providing care for a client with schizophrenia who receives haloperidol decanoate 75mg IM every 4 weeks. The client begins developing a puckering and smacking of the lips and facial grimacing. Which intervention should the nurse implement? A. Monitor lying, sitting, and standing blood pressures B. Provide coaching in relaxation techniques C. Complete abnormal involuntary movement scale (AIMS) D. Discontinue all medications immediately

C. Complete abnormal involuntary movement scale (AIMS)

*****While the nurse is assessing an older client's fall risk, the client reports living at home alone and never falling. Which action should the nurse take? A. Inform the client that falls occur more often in the hospital than at home B. Record a minimal risk for falls, documenting the client's statement C. Continue to obtain client data needed to complete the fall risk survey D. Place the client on a high fall risk protocol because of advanced age

C. Continue to obtain client data needed to complete the fall risk survey

*****Prior to surgery, written consent must be obtained. Which is the nurse's legal responsibility with regard to obtaining written consent? A. Explain the surgical procedure to the client and ask the client to sign the consent form B. Ask the client or a family member to sign the surgical consent form C. Determine that the surgical consent form has been signed and is included in the client's record. D. Validate the client's understanding of the surgical procedure to be conducted

C. Determine that the surgical consent form has been signed and is included in the client's record

*******A client in the emergency center demonstrates rapid speech, flight of ideas, and reports sleeping only three hours during the past 48 hours. Based on these findings, it is most important for the nurse to review the laboratory value for which medication? A. Lorazepam B. Fluoxetine C. Divalproex D. Olanzapine

C. Divalproex

***********A male client tells the nurse that he is concerned that he may have a stomach ulcer, because he is experiencing heartburn and a dull gnawing pain that is relieved when he eats. Which is the best response by the nurse? A. Instruct the client that these mild symptoms can generally be controlled with changes in his diet B. Advise the client that he needs to seek immediate medical evaluation and treatment of these symptoms C. Encourage the client to obtain a complete physical exam, since these symptoms are consistent with an ulcer D. Assure the client that his symptoms may only reflect reflux, since ulcer pain is not relieved with food

C. Encourage the client to obtain a complete physical exam, since these symptoms are consistent with an ulcer

*****A client in the third trimester of pregnancy reports that she fells some "lumpy places" in her breasts and that her nipples sometimes leak a yellowish fluid. She has an appointment with her healthcare provider in two weeks. What action should the nurse take? A. Tell the client to begin nipple stimulation to prepare for breast feeding. B. Reschedule the client's prenatal appointment for the following day C. Explain that this normal secretion can be assessed at the next visit D. Recommend that the client start wearing a supportive brassiere

C. Explain that this normal secretion can be assessed at the next visit

**********The nurse is auscultating a client's lung sounds. Which description should the nurse use to document this sound? A. Stridor B. Low pitched or coarse crackles C. High pitched or fine crackles D. High pitched wheeze

C. High pitched or fine crackles

********A young woman with multiple sclerosis just received several immunizations in preparation for moving into a college dormitory. Two days later, she reports to the nurse that she is experiencing increasing fatigue and visual problems. What teaching should the nurse provide? A. Plans to move into the dormitory need to be postponed for at least a semester B. These are common side effects of the vaccines and will resolve in a few days C. Immunizations can trigger a relapse of the disease, so get plenty of extra rest D. these early signs of an infection may require medical treatment with antibiotics

C. Immunizations can trigger a relapse of the disease, so get plenty of extra rest

*******An older client is brought to the ED with a sudden onset of confusion that occurred after experiencing a fall at home. The client's daughter, who has power of attorney, has brought the client's prescriptions. Which information should the nurse provide first when reporting to the healthcare provider using SBAR communication? A. currently prescribed medications B. Client's healthcare power of attorney C. Increasing confusion of the client D. Fall at home as reason for admission

C. Increasing confusion of the client

********The nurse on a pediatric unit observes a distraught mother in the hallway scolding her 3 year old son for wetting his pants. What initial action should the nurse take? A. Provide disposable training pants while calming the mother B. Refer the mother to a community parent education program C. Inform the mother that toilet training is slower for boys D. Suggest that the mother consult a pediatric nephrologist

C. Inform the mother that toilet training is slower for boys

*******When admitting a client with a diagnosis of transient ischemic attack (TIA), which intervention is most important for the nurse to include in this client's plan of care? A. Assess bilateral breath sounds B. Review client's daily medications C. Initiate neurological monitoring every 2 hours D. Palpate suprapubic region for urinary retention

C. Initiate neurological monitoring every 2 hours

*****A client who is scheduled for a bronchoscopy in the morning is anxious and asking the nurse numerous questions about the procedure. In preparing the client for the procedure, which intervention has the highest priority? A. Allow client to gargle with warm salt water B. Administer a sedative to alleviate anxiety C. Instruct client to write down the questions D. Deny client's request for a midnight snack

C. Instruct client to write down the questions

*******The nurse is providing care for a client with severe peripheral arterial disease (PAD). The client reports a history of rest ischemia, with leg pain that occurs during the night. Which action should the nurse take in response to this finding? A. Elevate the legs to assess for color changes B. Provide a heating pad for PRN use C. Offer cold packs when the pain occurs D. Suggest dangling the legs when pain occurs

C. Offer cold packs when the pain occurs

*******The nurse has received funding to design a health promotion project for African-American women who are at risk for developing breast cancer. Which resource is most important in designing this program? A. A listing of African-American women who live in the community B. Morbidity data for breast cancer in women of all races C. Participation of community leaders in planning the program D. Technical assistance to produce a video on breast self-examination

C. Participation of community leaders in planning the program

****A client is receiving enoxaparin 30mg subcutaneously twice a day. In assessing for adverse effects of the medication, which serum laboratory value is most important for the nurse to monitor? A. Glucose B. Calcium C. Platelet count D. White blood cell count

C. Platelet count

******The nurse is managing 4 clients in the intensive care unit who are mechanically ventilated. After performing a quick visual assessment, the nurse should prioritize care for the client who is exhibiting which finding? A. An audible voice when client is trying to communicate B. High pressure alarm sounds when client is coughing C. Restrained and restless with a low volume alarm sounding D. Diminished breath sounds in the right posterior base

C. Restrained and restless with a low volume alarm sounding

********A client is diagnosed with Meniere's disease. Which problem should the nurse identify as most important in the plan of care? A. Risk for ineffective self-health management related to deficient knowledge B. Ineffective coping related to personal vulnerability C. Risk for injury related to vertigo D. Anxiety related to disruption of lifestyle

C. Risk for injury related to vertigo.

********Which laboratory finding for an adult client is most critical for the nurse to report to the healthcare provider? A. Serum sodium 142mEq/L B. Serum potassium 3.9mEq/L C. Serum glucose 62 mg/dL D. Blood urea nitrogen 18 mg/dL

C. Serum glucose 62 mg/dL

***********The nurse leading a care team on a medical surgical unit is assigning client care to a practical nurse (PN) and an unlicensed assistive personnel (UAP). Which task should the nurse delegate to the UAP? A. Evaluate a client's mobility progress toward the plan of care B. Assess for side effects of administered pain medications C. Turn and reposition a client with a total hip replacement D. Monitor an intravenous infusion rate on an established schedule

C. Turn and reposition a client with a total hip replacement

******On admission to the Emergency Department, a female client who was diagnosed with bipolar disorder 3 years ago reports that this morning she took a handful of medications and left a suicide note for her family. Which information is most important for the nurse to obtain? A. Which family member has the client's suicide note B. What drugs the client used for the suicide attempt C. When the client last took drugs for bipolar disorder D. Whether the client over attempted suicide in the past

C. When the client last took drugs for bipolar disorder

********A young adult male who is being seen at the employee health care clinic for an annual assessment tells the nurse that his mother was diagnosed with schizophrenia when she was his age and that life with a schizophrenic mother was difficult indeed. Which response is best for the nurse to provide? A. Encourage the client to seek genetic counseling to determine his risk for mental illness B. Inform the client that his mother's schizophrenia has affected his psychological development C. Tell the client that mental illness has a familial predisposition so he should see a psychiatrist D. Ask the client if he is worried about becoming schizophrenic at the age his mother was diagnosed

D. Ask the client if he is worried about becoming schizophrenic at the age his mother was diagnosed

********When the nurse enters the room of a male client who was admitted for a fractured femur, his cardiac monitor displays a normal sinus rhythm, but he has no spontaneous respirations and his carotid pulse is not palpable. Which intervention should the nurse implement? A. Analyze the cardiac rhythm in another lead B. Obtain a 12-lead electrocardiogram C. Observe for swelling at the fracture site D. Begin chest compressions at 100/minute

D. Begin chest compressions at 100/minute

*****A client presents to the labor and delivery unit with a report of leaking fluid that is greenish-brown vaginal discharge. Which action should the nurse take first? A. Start an intravenous infusion B. Administer oxygen via facemask C. Perform a vaginal exam D. Begin continuous fetal monitoring

D. Begin continuous fetal monitoring

*****Then nurse identifies several nursing problems for client who is immobile and who has been experiencing fecal incontinence and diarrhea for several days. The client's spouse is the primary caregiver. In planning care, which problem has the highest priority? A. Impaired bed mobility B. Caregiver role strain C. Fluid volume deficit D. Bowel incontinence

D. Bowel incontinence

****A 3-year-old boy was successfully toilet trained prior to his admission to the hospital for injuries sustained from a fall. His parents are very concerned that the child has regressed in his toileting behaviors. Which information should the nurse provide to the parents? A. A retraining program will need to be initiated when the child returns home. B. Diapering will be provided since hospitalization is stressful to preschoolers C. A potty chair should be brought from home so he can maintain his toileting skills D. Children usually resume their toileting behaviors when they leave the hospital

D. Children usually resume their toileting behaviors when they leave the hospital

********The nurse assesses a client who has bilateral total knee replacements (TKR) four hours ago. The nurse notes that the dressing on the client's right knee is saturated with serosanguineous drainage. What action should the nurse implement? A. Monitor the client's current WBC B. Withhold next scheduled dose of low molecular weight heparin C. Confirm that the continuous passive motion device is intact D. Determine if the wound drainage device is functioning correctly

D. Determine if the wound drainage device is functioning correctly

*******The healthcare provider prescribes a sepsis protocol for a client with multi-organ failure caused by a ruptured appendix. Which intervention is most important for the nurse to include in the plan of care? A. Assess warmth of extremities B. Keep head of bed raised 45 degrees C. Monitor blood glucose level D. Maintain strict intake and output

D. Maintain strict intake and output

******The nurse instructs a client in use of a incentive spirometer. The client performs a return demonstration as seen in the video. Which action should the nurse take in response to the return demonstration? A. Auscultate the client's lungs for adventitious sounds B. Encourage the client to practice until successful C. Emphasize the need to inhale slowly into the spirometer D. Remind the client to cough after using the spirometer

D. Remind the client to cough after using the spirometer

*******A client with urge incontinence was treated with onabotuilinumtoxin A injections and is now experiencing urinary retention. Which action should the nurse include in the client's plan of care? A. Provide a bedside commode for immediate use in the client's room B. Teach the client techniques for performing intermittent catheterization C. Explain the need to limit intake of oral fluids to reduce client discomfort D. Remind the client to practice pelvic floor (Kegel) exercises regularly

D. Remind the client to practice pelvic floor (Kegel) exercises regularly

*******A teenager presents to the emergency department with palpitations after vaping at a party. The client is anxious, fearful, and hyperventilating. The nurse anticipates the client developing which acid base imbalance? A. Respiratory acidosis B. Metabolic alkalosis C. Metabolic acidosis D. Respiratory alkalosis

D. Respiratory alkalosis

********A 7-year old is admitted to the hospital with persistent vomiting, and a nasogastric tube attached to low intermittent suction is applied. Which finding is most important for the nurse to report to the healthcare provider? A. Shift intake of 640mL IV fluids plus 30mL PO ice chips B. Serum pH of 7.45 C. Gastric output of 100 mL in the last 8 hours D. Serum potassium of 3.0 mg/dL

D. Serum potassium of 3.0 mg/dL

****When assessing a recently delivered, multigravida client, the nurse finds that her vaginal bleeding is more than expected. Which factor in this client's history is related to this finding? A. The second stage of labor lasted 10 minutes B. She received butorphanol 2mg IVP during labor C. She is over 35 years of age D. She is a gravida 6, para 5

D. She is a gravida 6, para 5

****A client who recently received a prescription for ramelteon to treat sleep deprivation reports experiencing several side effects since taking the drug. Which side effect should the nurse report to the healthcare provider? A. A change in the sleep-wake cycle B. Mild sedation C. Dizziness reported after initial dose D. Somnambulism

D. Somnambulism

******A client is scheduled for a spiral computed tomography (CT) scan with contrast to evaluate for pulmonary embolism. Which information in the client's history requires follow-up by the nurse? A. CT scan that was performed 6 months earlier B. Metal hip prosthesis was placed 20 years ago C. Report of client's sobriety for the last 5 years D. Takes metformin for type 2 diabetes mellitus

D. Takes metformin for type 2 diabetes mellitus

********A client fell in the bathroom when left unattended by the unlicensed assistive personnel (UAP). Which information should the nurse include in the client's health record? A. The UAP left the client to assist another client B. The last time client was assisted to the bathroom C. The unit was understaffed when the client fell D. The client fell sustaining a fracture to the left hip

D. The client fell sustaining a fracture to the left hip

******After receiving report on an inpatient acute care unit, which client should the nurse assess first? A. The client with an obstruction of the large intestine who is experiencing abdominal distention B. The client who had surgery yesterday and is experiencing a paralytic ileus with absent bowel sounds C. The client with a small bowel obstruction who has a nasogastric tube that is draining greenish fluid D. The client with a bowel obstruction due to a volvulus who is experiencing abdominal rigidity

D. The client with a bowel obstruction due to a volvulus who is experiencing abdominal rigidity

********A client with multiple burn injuries is being treated in the burn trauma unit just hours after the injuries occurred. The healthcare provider instructs the nurse to avoid auto contamination when performing dressing changes. Which intervention is most important for the nurse to implement? A. Dress each wound separately B. Assign equipment to this one client C. Utilize reverse isolation protocol D. Use gown, mask, and gloves with dressing changes

D. Use gown, mask, and gloves with dressing changes

*******A client with dyspnea is being admitted to the medical unit. To best prepare for the client's arrival, the nurse should ensure that the client's bed is in which position? (picture) A. Supine B. supine; feet elevated higher than head C. supine; head elevated higher than feet D. Fowlers

Fowlers

********To auscultate for a carotid bruit, the nurse places the stethoscope at what location. (Select the location on the image with a red dot).

I placed the red dot on the base of the neck on the right side

**************Which instruction should the nurse provide a pregnant client who is reporting heartburn? A. Eat small meals throughout the day to avoid a full stomach. B. Take an antacid at bedtime and whenever symptoms worsen. C. Maintain a sitting position for two hours after eating. D. Limit fluids between meals to avoid overdistension of the stomach.

a

*************A nurse determines that more than 25% of the students at a middle school are overweight. The nurse presents the information at the parent-teacher meeting. What action is most important for the nurse to include in the meeting? a. Provide information on ways to increase activity for the family. b. Have several teachers talk about health risks associated with obesity. c. Distribute a shopping list of suggested healthy snack items. d. Determine the parents' degree of concern about their children's weight.

a

*************An unlicensed assistive personnel (UAP) is assigned to provide personal care for a client whose prescribed activity is bedrest with bedside commode use. The UAP reports to the nurse that the client is so obese that the UAP feels unable to safely assist the client in transferring from the bed to the bedside commode. How should the nurse respond? a. Determine the client's level of mobility and need for assistance. b. Instruct the UAP that all clients deserve equal care. c. Advice the client to maintain bedrest so that safety can be ensured. d. Assign another UAP to care for the client.

a

*************The nurse assumes care of a postoperative adult client with type 2 diabetes mellitus and learns that the client has a current blood glucose level of 720 mg/dL. When assessing the client, what is the priority? A. Assess for signs of fluid volume deficit B. Observe wound drainage characteristics C. Measure the level of acute pain D. Determine when the client last ate

a

************A client with leukemia who is receiving a myleosuppressive chemotherapy has a platelet count of 25,000/mm3. Which intervention is most important for the nurse to include in this client's plan of care? A. Assess urine and stool for occult blood B. Monitor for signs of activity intolerance C. Require visitors to wear respiratory masks D. Obtain client's temperature q4 hours

a

************An older woman who has difficulty hearing is being discharged from day surgery following a cataract extraction and lens implantation. Which intervention is most important for the nurse to implement to help ensure the client's compliance with self care? A. Have the client vocalize the instructions provided B. Ensure that someone will stay with the client for 24 hours C. Speak clearly and face the client for lip reading D. Provide written instructions for eye drop administration

a

************The nurse includes assessment for fat embolism syndrome (FES) in the plan of care for a client with a fractured femur. Which findings should the nurse include that are often the earliest indication of a FES? A. Confusion, restlessness B. Petechial rash C. Tachycardia, fever D. Pulmonary crackles

a

************The nurse prepares an intravenous solution and tubing for a client with a saline lock, as seen in the video. What action should the nurse take next? A. Open the roller clamp on the tubing B. Label the bag of IV solution C. Attach the tubing to the saline lock D. Flush the saline lock with saline

a

***********After an inservice about electronic health record (EHR) security and safeguarding client information, the nurse observes a colleague going home with printed copies of client information in a uniform pocket. Which action should the nurse take? a. File a detailed incident report with the specific hiring facility. b. Warn the colleague that their actions are unprofessional. c. Comment anonymously about the action of a staff discussion board. d. Communicate the colleague's actions to the unit charge nurse

a

***********An older woman with history of atrial fibrillation fell at home and fractured her left hip. She is currently taking warfarin 5 mg daily and has an international normalized ratio (INR) value of 5.0. Upon admission, which prescription should the nurse expect to implement? A. Administer Vitamin K injection B. Start continuous heparin infusion C. Continue warfarin at same dose D. Transfuse unit of packed red blood cells

a

***********The charge nurse in an extended care facility is organizing unit activities for the day. Which action may be safely delegated to the practical nurse (PN)? a. Measure the client's body weight each morning. b. Establish blood pressure parameters for client monitoring c. Evaluate a staff member providing wound care. d. Evaluate client teaching through return demonstration.

a

***********The nurse is assessing a client who returns to the unit after a thoracentesis in the procedure room. Which finding should the nurse report to the healthcare provider immediately? A. Diminished breath sounds over the trocar insertion site B. Equal bilateral chest expansion C. Scattered crackles unchanged from baseline D. Respiratory rate of 22 breaths/minute

a

**********The nurse is assessing a 4-year-old child with eczema. The child's skin is dry and scaly, and the mother reports that the child frequently scratches the lesions on the skin to the point of causing bleeding. Which guideline is indicated for care of this child? a. Keep the nails trimmed short. b. Apply baby lotion to the skin twice daily. c. Bathe the child daily with bath oil. d. Allow the child to wear only 100% cotton clothing

a

**********Which information is most important for the nurse to obtain when determining a client's risk for obstructive sleep apnea syndrome (OSAS)? A. Body mass index B. Breath sounds C. Self-description of pain D. Level of consciousness

a

**********an adult male who fell 20 feet from the roof of this home has multiple injuries, including a right pneumothorax. Chest tubes were inserted in the emergency department prior to his transfer to the intensive care unit (ICU). the nurse notes that the suction control chamber is bubbling at the - 10 cm H2O mark, with fluctuation in the water seal, and over the past hour 75 ml of bright red blood is measured in the collection chamber. Which intervention should the nurse implement? a. Add sterile water to the suction control chamber. b. Give blood from the collection chamber as autotransfusion c. Manipulate blood in tubing to drain into chamber. d. Increase wall suction to eliminate fluctuation in water seal.

a

*********A female child is brought to the emergency department after awakening with a bark-like cough and stridor. upon arrival to the hospital, her respirations are labored, and she is drooling. What action should the nurse implement? A. Prepare for emergency tracheotomy B. Assess the child for dehydration C. Examine oropharyngeal area for foreign body D. Collect midstream urine specimen

a

*********The laboratory findings for a client with chronic kidney disease (CKD) include elevated blood urea nitrogen (BUN) and serum creatinine levels. The client reports feeling fatigued and is unable to concentrate during the morning assessments. Based on these findings, which action should the nurse implement? A. Provide high protein snacks B. Administer PRN oxygen C. Schedule frequent rest periods D. Monitor glucose levels q4 hours.

a

********The nurse is caring for a client admitted for evaluation of a descending aortic aneurysm. While outside the room documenting, the nurse hears the client screaming. The client tells the nurse that the pain is "sharp, like something inside is ripping and tearing." The client also reports dizziness. Which of the following is the likely cause? A. Impending rupture of the aneurysm B. The client is having a panic attack C. Clotting of the aneurysm D. The client is hallucinating from the opioids

a

********The nurse is caring for a client who is having a sickle cell crisis. What intervention should the nurse include in this client's plan of care? a. Ensure adequate IV and oral fluid intake. b. Provide ice packs to major joint areas. c. Space analgesics to prevent addiction to narcotics. d. Re-enforce the importance of nutritional balance.

a

*******Which snack selection indicates to the nurse that a school-age boy with gastroesophageal reflux understands his dietary restrictions? A. Sugar cookies B. Pizza C. Chocolate milkshake D. Tacos

a

******What is the priority nursing action when initiating morphine therapy via an intravenous patient-controlled analgesia (PCA) pump? A. Initiate the dosage lockout mechanism on the PCA pump B. Assess the client's ability to use a numeric pain scale C. Assess the abdomen for bowel sounds D. Instruct the client to use the medication before the pain becomes severe

a

A 16 years old male client who has been treated in the past for a seizure disorder is admitted to the hospital immediately after admission he begins to have a grand mal seizure. Which action should the nurse implement? A) Observe the client carefully. B) Place a padded tongue blade between client's teeth. C) Obtain assistance in holding him to prevent injury. D) Call the rapid respond team.

a

A 4 year-old has been hospitalized for 24 hours with skeletal traction for treatment of a fracture of the right femur. The nurse finds that the child is now crying and the right foot is pale with the absence of a pulse. What should the nurse do first? A) Notify the health care provider B) Readjust the traction C) Administer the ordered PRN medication D) Reassess the foot in fifteen minutes

a

A Nurse is assessing a client who has hyponatremia and is receiving IV fluid therapy. Which of the following findings indicate the client is developing a complication of therapy? a. Peripheral edema b. Increased thirst. c. Flattened neck veins. d. Hypotension

a

A charge nurse is teaching a newly licensed nurse about the administration of total parenteral nutrition. Which of the following should the charge nurse include? A. "You will need to monitor the client's electrolytes daily" B. "You will need to change the IV dressing site once per week" C. "You will need to warm the solution in the microwave before administration" D. "You need to weigh the client twice per week"

a

A client admitted with a liver abscess is scheduled for surgical evacuation and drainage of the abscess tomorrow morning. Nursing assess .... Client's abdominal pain has increased from 4 to 8 on a 10-point scale in the last four hours. What is priority nursing action? a. Notify the surgeon of increasing abdominal pain. b. Administer the nest scheduled dose of antibiotic. c. Encourage the client to cough and deep breath. d. Assess for a change in the client's bowel sounds.

a

A client diagnosed with anorexia nervosa states after lunch, "I shouldn't have eaten all of that sandwich, I don't know why I ate it, I wasn't hungry." The client's comments indicate that the client is likely experiencing A) Guilt B) Bloating C) Anxiety D) Fea

a

A client has viral pneumonia affecting 2/3 of the right lung. What would be the best position to teach the client to lie in every other hour during first 12 hours after admission? A) Side-lying on the left with the head elevated 10 degrees B) Side-lying on the left with the head elevated 35 degrees C) Side-lying on the right with the head elevated 10 degrees D) Side-lying on the right with the head elevated 35 degrees

a

A client is admitted reporting an acute onset of right flank pain and urinary urgency. Which assessment is most important for the nurse to obtain? A) Numerical rated pain intensity. B) Amount of daily caffeine intake. C) Current body temperature. D) Fluid intake for the past 24 hours.

a

A client is admitted to the mental health unit with relationship distress with spouse and depressed mood. Finding of which diagnostic tests provide the most information for developing this client's plan care? A) Urine drug screen. B) Complete blood court. C) Basic metabolic panel. D) Electrocardiogram.

a

A client is scheduled to receive an oral solution of radioactive iodine (131I). In order to reduce hazards, the priority information for the nurse to include during the instructions to the client is which of these statements? A) In the initial 48 hours avoid contact with children and pregnant women, and after urination or defecation flush the commode twice. B) Use disposable utensils for 2 days and if vomiting occurs within 10 hours of the dose, do so in the toilet and flush it twice. C) Your family can use the same bathroom that you use without any special precautions. D) Drink plenty of water and empty your bladder often during the initial 3 days of therapy.

a

A client peptic ulcer disease receives a prescription for intermittent suction via a SalemSump nasogastric tube (NGT). After inserting the NGT and obtaining coffee- ground gastric contents, the nurse clamps the NGT because the client must leave the unit for diagnostic studies. Upon return to the unit, the client complains of nausea. What action should the nurse implement first? A) Administering a prescribed antiemetic agent. B) Provide oral suction using a Yankauer tip. C) Connect the NGT to low intermittent suction. D) Irrigate the NGT with sterile normal saline.

a

A client who had a vasectomy is in the post recovery unit at an outpatient clinic. Which of these points is most important to be reinforced by the nurse? A) Until the health care provider has determined that your ejaculate doesn't contain sperm, continue to use another form of contraception. B) This procedure doesn't impede the production of male hormones or the production of sperm in the testicles. The sperm can no longer enter your semen and no sperm are in your ejaculate. C) After your vasectomy, strenuous activity needs to be avoided for at least 48 hours. If your work doesn't involve hard physical labor, you can return to your job as soon as you feel up to it. The stitches generally dissolve in seven to ten days. D) The health care provider at this clinic recommends rest, ice, an athletic supporter or over-the-counter pain medication to relieve any discomfort.

a

A client who has a belief based in Hinduism is nearing death. The nurse should plan for which action? A) After death a Hindu priest will pour water into the mouth of the client and tie a thread around the client's wrist B) The elders may be with the client during the process of the client dying and no last rites are given C) The family must be with the client during the process of dying and be the only ones to wash the body after death D) The body is ritually cleansed and burial is to be as soon as possible after the death occurs

a

A client who is hypotensive is receiving dopamine, and adrenergic agonist, IV at the rate of 8 mcg/kg/min. Which intervention should the nurse implement while administering this medication? A) Measure urinary output every hour. B) Monitor serum potassium frequently. C) Initiate seizure precautions. D) Assess pupillary response to light hourly.

a

A client who underwent an uncomplicated gastric bypass surgery is having difficult with diet management. Which dietary instruction is most important for the nurse to explain to the client? A) Plan volume-controlled evenly-space meal thorough the day. B) Sip fluid slowly with each meal and between meals. C) Eliminate or reduce intake fatty and gas forming food. D) Chew food slowly and thoroughly before attempting to swallow.

a

A client who was splashed with a chemical has both eyes covered with bandages. When assisting the client with eating, which intervention should the nurse instruct the unlicensed assistive personnel (UAP) to implement? A) Orient the client to the location of the food on the plate. B) Ask to visit during meal time to assist with feeding. C) Provide with only finger foods D) Feed the client the entire meal.

a

A client with a history of chronic obstructive pulmonary disease (COPD) is admitted with pneumonia. Vital signs include heart rate of 122 beats/minute, respiratory rate 28 breaths/minute, and blood pressure 170/90 mmHg. Which assessment finding warrants the most immediate intervention by the nurse? a. Bilateral diffuse wheezing b. Temperature of 100.5 c. Yellow expectorated sputum d. Shortness of breath on exertion

a

A client with chronic alcoholism is admitted with a decreased serum magnesium level. Which snack option should the nurse recommend to this client? A) Dry roasted almonds. B) Cheddar cheese and crackers. C) Carrot and celery sticks. D) Beef bologna sausage slices.

a

A client with chronic kidney disease is admitted in heart failure and is complaining of shortness of breath and a headache. Assessment findings include blood pressure 180/90 mmHg, heart rate 130 beats/minute, oxygen saturation 89%, and a temperature of 100 degrees Fahrenheit. A temporary dialysis catheter is inserted for immediate hemodialysis and the client is scheduled for replacement of an arterial venous fistula in the left arm. Which action should the nurse implement? a. Avoid using the left arm for IV access b. Initiate oxygen at 110% per face mask c. Give the PRN dose of enalaprild . Administer PRN antipyretic prescription

a

A client with heart failure has a prescription for digoxin. The nurse is aware that sufficient potassium should be included in the diet because hypokalemia in combination with this medication A) Can predispose to dysrhythmias B) May lead to oliguria C) May cause irritability and anxiety D) Sometimes alters consciousness

a

A client with osteoporosis related to long-term corticosteroid therapy receives a prescription for calcium carbonate. Which client's serum laboratory values requires intervention by the nurse? A) Creatinine 4 mg/dl (354 micromol/L SI) B) Total calcium 9 mg/dl (2.25 mmol/L SI) C) Phosphate 4 mg/dl (1.293 mmol/L SI) D) Fasting glucose 95 mg/dl (5.3 mmol/L SI)

a

A client with renal disease seems anxious and presents with the onset of shortness of breath, lethargy, edema, and weight gain. Which action should the nurse implement first? A) Determine serum potassium level. B) Calculate the client's daily fluid intake. C) Assess client for signs of vertigo. D) Review the client's pulse oximetry reading.

a

A community health nurse receives a referral for a family home visit. Which of the following tasks should the nurse perform first? a. Clarify the source of the referral. b. Implement the nursing process c. Schedule a time for the home visit d. Contact the family by phone

a

A family member reports that the client who is bedridden has not been turned or repositioned all night and is sleeping on a special air mattress with no sheets. What information should the nurse provide to the family member? A) Clarify that an aerated support surface does not use sheets that often cause skin breakdown. B) Described the night staff's plan of care to ensure the client's sleep is not disturbed. C) Explained that turning is only necessary to reposition the client during waking hours. D) Suggest that a family member turn the client during the night when someone is there

a

A female client with chronic kidney disease and renal failure has an indwelling peritoneal catheter in ..... used for peritoneal dialysis. While bathing, the her abdominal dressing becomes wet. What action should the nurse take? a. Change the dressing. b. Reinforce the dressing. c. Flush the peritoneal dialysis catheter. d. Scrub the catheter with povidone-iodine.

a

A hospice nurse is visiting with the son of a client who has terminal cancer. The son reports sleeping very little during the past week due to caring for his mother. Which of the following responses should the nurse make? a. "I can give you information about respite care if you are interested." b. "You should consider taking a sleeping pill before bed each night" c. "It must be difficult taking care of someone who is terminally ill" d, "You are doing a great job taking care of your mother"

a

A male client reports to the on-call clinic nurse that he took tadalif 10 mg PO two hours age and his skin now feels flushed. He reports a history of stable angina, but denies experiencing any current or recent chest pain. What action should the nurse take? A) Tell the client to have someone bring him to an emergency department immediately. B) Advise the client to place one nitroglycerin tablet under his tongue as a precaution. C) Reassure the client that skin flushing is a common side effect of the medication. D) Instruct the client to increase his intake of oral until the skin flushing is relieved.

a

A male client with an antisocial personality disorder is admitted to an in patient mental health unit for multiple substance dependency. When providing a history, the client justifies to the nurse his use of illicit drugs. Based on this pattern of behavior, this client's history is most likely to include which finding? A) Multiple convictions for misdemeanors and Class B felonies B) Delusions of grandiosity and persecution. C) Suicidal ideations and multiple attempts. D) Photos and panic attacks when confronted by authority figures.

a

A middle aged female client tells the clinic nurse that she has lost an inch of height in the last year. What is the priority nursing intervention? A) Assist the client to schedule a bone density exam. B) Observe for the presence of a dowager's C) Advice the client to begin stretching exercises. D) Encourage the client to eat calcium rich foods.

a

A nurse has agreed to serve as an interpreter for an older adult client who is assigned to another nurse. Which of the following statements by the nurse indicates an understanding of this role? a. "I will let the client know that I am available as the interpreter." b. "I will receive a small fee for interpreting for this client." c. "I am glad I'm available today, but when I'm not, you can use a family member." d. "I will let the client know that an interpreter is unavailable during the night shift."

a

A nurse in a pediatric clinic is teaching a newly hired nurse about the varicella rooster. Which of the following information should the nurse include? a. Children who have varicella are contagious until vesicles are crusted. b. Children who have varicella should receive the herpes zoster vaccination. c. Children who have varicella should be placed in droplet precaution. d. Children who have varicella are contagious 4 days before the first vesicle eruption.

a

A nurse in a pediatric unit is preparing to insert an IV catheter for 7-year- old. Which of the following actions should the nurse take? a. Tell the child they will feel discomfort during the catheter insertion. b. Use a mummy restraint to hold the child during the catheter insertion. c. Require the parents to leave the room during the procedure.

a

A nurse in an emergency department is performing triage for multiple clients following a disaster in the community. To which of the following types of injuries should the nurse assign the highest priority? a. Below-the knee amputation b. Fractured tibia c. 95% full-thickness body burn d. 10cm (4in) laceration to the forearm

a

A nurse is administering an analgesic to a client who has a chest tube. The provider is preparing to discontinue the chest tube before the medication has taken affect. Which of the following actions should the nurse prepare to take first? a. Inform the provider of the time of the last dose of pain medication. b. Document the sequence of events as they occur. c. Provide non-pharmacological pain management interventions. d. Instruct the client about the steps of the procedure.

a

A nurse is admitting a client who has acute heart failure. Which of the following prescriptions from the provider should the nurse anticipate? a. Administer enalapril 2.5 mg PO twice daily b. Ambulate the client every 4 hr while awake c. Provide the client with 4 g sodium diet d. Infuse 0.9% sodium chloride 500 mL IV bolus over 1 hr

a

A nurse is admitting medications to a group of clients. Which of the following occurrences requires the completion of an incident report? a. A client receives his antibiotics 2hr late b. A client vomits within 20min of taking his morning medications c. A client requests his statin to be administered at 2100 d. A client asks for pain medication 1hr early

a

A nurse is assessing a child who is being treated for bacterial pneumonia. The nurse notes an increase in the child's glucose. The nurse should identify this finding as an adverse effect of which of the following medications a. Methylprednisolone. b. Ondansetron. c. Guaifenesin. d. Amoxicillin

a

A nurse is assessing a client who has acute kidney injury and a respiratory rate of 34/min. The client's ABG results are ph. 7.28 HCO3 18 mEq/L. PaO2 90 mm Hg. Which of the following conditions should the nurse expect? a. Metabolic acidosis. b. Metabolic alkalosis. c. Respiratory acidosis. d. Respiratory alkalosis.

a

A nurse is assessing a client who is 30 min postoperative following an arterial thrombectomy. Which of the following findings should the nurse to report? a. Chest pain b. Muscle spasms. c. Cool, moist skin. d. Incisional pain.

a

A nurse is assessing a client's pulmonary artery wedge pressure (PAWP). The nurse should recognize that an elevated PAWP indicates which of the following complications? a. Left ventricular failure b. Cardiogenic shock c. Hypovolemia d. Hypotension

a

A nurse is assessing several clients in a long term health care facility. Which client is at highest risk for development of decubitus ulcers? A) A 79 year-old malnourished client on bed rest B) An obese client who uses a wheelchair C) A client who had 3 incontinent diarrhea stools D) An 80 year-old ambulatory diabetic client

a

A nurse is assisting with the development of an informed document for participation in a research study. Which of the following information should the nurse include? a. A statement that participants can leave the study at will. b. An assignment of the participant to either the experimental or control group. c. A list of the clients participating in the study. d. A description of the framework the researchers will use to evaluate the data.

a

A nurse is caring for a 5-month-old infant who has manifestations of severe dehydration and a prescription for paternal fluid therapy. The guardian asks. "What are the indications that my baby needs an IV?" Which of the following responses should the nurse make? a. "Your baby needs an IV because she is not producing any tears" b. "Your baby needs an IV because her fontanels are budging" c. "Your baby needs an IV because she is breathing slower than normal" d. "Your baby needs an IV because her heart rate is decreasing"

a

A nurse is caring for a client who has diabetes mellitus and is receiving long-acting insulin for blood glucose management. The nurse should anticipate administering which of the following types of insulin? a. Glargine insulin. b. Regular insulin. c. NPH insulin.

a

A nurse is caring for a client who has fibromyalgia and requests pain medication. Which of the following medications should the nurse administer? a. Pregabalin b. Lorazepam c. Colchicin d. Codeine.

a

A nurse is caring for a client who has generalized petechiae and ecchymoses. The nurse should expect a prescription for which of the following laboratory tests? a. platelet count b. potassium level c. creatine clearance d. prealbumin

a

A nurse is caring for a client who has returned to the medical-surgical unit following a transurethral resection of the prostate. Which of the following should the nurse identify as priority nursing assessment after reviewing the clients information? Exhibit. a. loss of consciousness b. Skin turgor c. deep tendon reflexes d. Bowel sounds

a

A nurse is caring for a client who is 1 hr postoperative following rhinoplasty. Which of the following manifestations requires immediate action by the nurse? a. Increase in frequency of swallowing b. Moderate sanguineous drainage on the drip pad c. Bruising to the face d. Absent gag reflex

a

A nurse is caring for a client who is 24 hr. postpartum and is breast feeding her newborns. The client asks the nurse to warm up seaweed soup that the client's partner brought for her. Which of the following responses should the nurse make? a. "Does the doctor know you are eating that?" b. "Why are you eating seaweed soup?" c. "Of course I will heat that up for you" d. "The hospital good is more nutritious"

a

A nurse is caring for a client who is at 14 weeks gestation and reports feelings of ambivalence about being pregnant. Which of the following responses should the nurse make? a. "Describe your feelings to me about being pregnant" b. "You should discuss your feelings about being pregnant with your provider" c. "Have you discussed these feelings with your partner?" d. "When did you start having these feelings?"

a

A nurse is caring for a client who is at 38 weeks of gestation and has a history of hepatitis C. The client asks the nurse if she will be able to breastfeed. Which of the following responses by the nurse is appropriate? a. You may breastfeed unless your nipples are cracked or bleeding b. You must use a breast pump to provide breast milk. c. You must use nipple shield when breastfeeding. d. You may breastfeed after your baby develops his antibiotics.

a

A nurse is caring for a client who is comatose and has advance directives that indicate the client does not want life-sustaining measures. The client's family want the client to have life-sustaining measures. Which of the following action should the nurse take? A . Arrange for an ethics committee meeting to address the family's concerns. B . Support the family's decision and initiate life-sustaining measures. C. Complete an incident report. D. Encourage the family to contact an attorney.

a

A nurse is caring for a client who is experiencing mild anxiety. Which of the following findings should the nurse expect? a. Heightened perceptual field b. Rapid speech c. Feelings of dread d. Purposeless activity

a

A nurse is caring for a client who is in a seclusion room following violent behavior. The client continues to display aggressive behavior. Which of the following actions should the nurse take? a. Confront the client about this behavior. b. Express sympathy for the client's situation. c. Speak assertively to the client. d. Stand within 30 cm (1 ft) of the client when speaking with them.

a

A nurse is caring for a client who reports diarrhea for 3 days. The nurse should monitor the client for which of the following manifestations? a. Orthostatic Hypertension b. Dependent Edema c. Decreased Hematocrit d. Neck Vein Distension

a

A nurse is caring for a client who wears glasses. Which of the following actions should the nurse take? A. Store the glasses in a labeled case. B. Clean the glasses with hot water. C. Clean the glasses with a paper towel. D. Store the glasses on the bedside table.

a

A nurse is caring for a client who will undergo a procedure. The client states he does not want the provider to discuss the results with his partner. Which of the following is an appropriate response for the nurse to make? a. You have the right to decide who receives information b. Your partner can be a great source of support for you at this time c. Is there a reason you don't want your partner to know about your procedure? d. The provider will be tactful when talking to your partner

a

A nurse is caring for an infant who has coaction of the aorta. Which of the following should the nurse identify as an expected finding? a. Weak femoral pulses b. Frequent nosebleeds c. Upper extremity hypotension d. Increased intracranial pressure

a

A nurse is caring who is to receive a transfusion of packed RBCs. Which of the following actions should the nurse take? a. Prime IV tubing with 0.9% sodium chloride. b. use a 24- gauge IV catheter c. obtain filter less IV tubing d. place blood in the warmer for 1 hr

a

A nurse is developing a plan of care for a client who has schizophrenia and is experiencing auditory hallucinations. Which of the following actions should the nurse include in the plan? a. Ask the client directly what he is hearing b. Encourage the client to lie down in a quiet room c. Avoid eye contact with the client d. Refer to the hallucinations as if they are real

a

A nurse is discussing a weight loss with a client who is concerned about losing 6.8 kg (15lb) from an original weight of 9o.7 (200 lb). The nurse should identify the weight of the following total percentage? a. 7.5% b. 15% c. 8.1% d. 13.3%

a

A nurse is preparing education material for a client. Which of the following techniques should the nurse use in creating material? A. Emphasize important information using bold lettering. B. Use 7th grade reading level. C. Avoid using cartoons in the teaching material. D. Use words with three or four syllables

a

A nurse is preparing to measure a temperature of an infant. Which of the following action should the nurse take? a. Place the tip of the thermometer under the center of the infant's axilla. b. Pull the pinna of the infant's ear forward before inserting the probe. c. Insert the probe 3.8 cm (1.5in) into the infant's rectum. d. Insert the thermometer in front of the infant's tongue.

a

A nurse is preparing to mix NPH and regular insulin in the same syringe. Which of the following a. Inject air into the NPH insulin vial. b. Withdraw the prescribed dose of regular insulin c. Withdraw the prescribed dose of NPH insulin

a

A nurse is preparing to perform a sterile dressing change. Which of the following actions should the nurse take when setting up the sterile field? a. Place the cap from the solution sterile side up on clean surface b. Open the outermost flap of the sterile kit toward the body c. Place the sterile dressing within 1.25 cm (0.5in) of the edge of the sterile field d. Set up the sterile field 5 cm (2 in) below waist level

a

A nurse is providing care to a primigravida whose membranes spontaneously ruptured (ROM) 4 hours ago. Labor is to be induced. At the time of the ROM the vital signs were T-99.8 degrees F, P-84, R-20, BP-130/78, and fetal heart tones (FHT) 148 beats/min. Which assessment findings taken now may be an early indication that the client is developing a complication of labor? A) FHT 168 beats/min B) Temperature 100 degrees Fahrenheit. C) Cervical dilation of 4 D) BP 138/88

a

A nurse is providing dietary teaching to a client who has a new diagnosis of irritable bowel syndrome. Which of the following recommendations should the nurse include? a. Consume food high in bran fiber b. Increase intake of milk products c. Sweeten foods with fructose corn syrup d. Increase foods high in gluten

a

A nurse is providing dietary teaching to a guardian of a preschooler who has a new diagnosis of celiac disease. Which of the following statements by the guardian indicates an understanding of the teaching? A. "I will put my child on a gluten-free diet". B. "I will administer digestive enzymes with meals and snacks". C. "Provide my child with some high fiber foods.". D. "I will give my child whole wheat toast and milk for breakfast"

a

A nurse is providing discharge teaching to a client who has cancer and a prescription for a fentanyl 25 mcg /hr transdermal patch. Which of the following instructions should the nurse include in the teaching? a. Avoid hot tub while wearing the patch b. Apply patch to your forearm c. Avoid high-fiber foods while taking this medication d. Remove the patch for 8 hours every day to reduce the risk for tolerance.

a

A nurse is providing teaching about the gastrostomy tube feedings to the parents of a school age child. Which of the following instructions should the nurse take? a. Administer the feeding over 30 min. b. Place the child in as supine position after the feeding. c. Charge the feeding bag and tubing every 3 days. d. Warm the formula in the microwave prior to administration.

a

A nurse is providing teaching to a client about the adverse effects of sertraline. Which of the following adverse effects should the nurse include? a. Excessive sweating b. Increased urinary frequency c. Dry cough d. Metallic taste in mouth

a

A nurse is providing teaching to a client who is at 14 weeks of gestation about findings to report to the provider. Which of the following findings should the nurse include in the teaching? a. Swelling of the face b. Urinary frequency c. Faintness upon rising d. Bleeding gums

a

A nurse is providing teaching to a client who is to undergo a cardiac catheterization. Which of the following findings is expected during the procedure? a. Sensation of skin warmth b. Headache c. Increased salivation d. Numbness and tingling of the extremities

a

A nurse is providing teaching to a client who will undergo a magnetic resonance imaging (MRI) scan. Which of the following statements is appropriate to include in the teaching? a. "You should not have this procedure if you are allergic to iodine." b. "You should not have this procedure if you have a tattoo." c. "The nurse will ask you to wear protective eyewear during this procedure." d. "The nurse will ask you to remove any transdermal patches prior to the procedure."

a

A nurse is providing teaching to an older adult client about methods to promote nighttime sleep. Which of the following instructions should the nurse include? a. Eat a light snack before bedtime c. Stay in bed at least 1 hr if unable to fall asleep d. Take a 1 hr nap during the day e. Perform exercises prior to bedtime

a

A nurse is providing teaching to family members of a client who has dementia. Which of the following instructions should the nurse include in the teaching? a. Establish a toileting schedule for the client b. Use clothing with buttons and sippers c. Discourage physical activity during the day d. Engage the client in activities that increase sensory stimulation

a

A nurse is reviewing laboratory data for a client who has chronic kidney disease. Which of the following findings should the nurse expect? a. Increased creatine. b. Increased hemoglobin. c. Increased bicarbonate. d. Increased calcium.

a

A nurse is reviewing the medical history of a client who has angina. Which of the following findings in the client's medical history should identify as a risk factor for angina? a. Hyperlipidemia. b. COPD c. Seizure disorder d. Hyponatremia.

a

A nurse is teaching about preventative measures to a female client who has chronic urinary tract infections. Which of the following interventions should the nurse include in the teaching? a. "Drink 2 liters of warm water per day". b. "Empty your bladder every 6 weeks.". c. "Soak in a warm bath everyday". d. "Take an oral estrogen tablet".

a

A nurse is teaching an in-service about nursing leadership. Which of the following information should the nurse include about an effective leader? a. Acts as an advocate for the nursing unit. c. Priorities staff request over client needs. d. Provides routine client care and documentation.

a

A nurse is teaching at a community health fair about electrical fire prevention. Which of the following information should the nurse include in the teaching? A. Use three pronged grounded plugs. B. Cover extension cords with a rug. C. Check the tingling sensations around the cord to ensure the electricity is working. D. Remove the plug from the socket by pulling the cord

a

A nurse is transcribing new medication prescriptions for a group of clients. For which of the following prescriptions should the nurse contact the provider for clarification? a. Lorazepam .5 mg PO one tablet daily b. Hydrochlorothiazide 12.5 mg PO BID c. Triamcinolone acetonide 100 mcg/inhalation two puffs TID d. Zolpidem 10 mg PO one tablet at bedtime

a

A nurse on a telemetry unit is assessing a client who is receiving continuous cardiac monitoring. The client's heart rate is 69/min and the PR interval is 0.24 seconds. The nurse should interpret this finding as which of the following cardiac rhythms? a. First degree AV block b. Premature ventricular contraction. c. Sinus bradycardia. d. Atrial fibrillation.

a

A nurse realizes that the wrong medication has been administered to a client. Which of the following actions should the nurse take first? a. Notify the provider. b. Report the incident to the nurse manager. c. Monitor vital signs. d. Fill out an incident report.

a

A nurse stops at the site of a motorcycle accident and finds a young adult male lying face down in the road in a puddle of water. It is raining, no one is available to send for help, and the cell phone is in the car about 50 feet away. What action should the nurse take first? A) Stabilize the victim's neck and roll over to evaluate his status. B) Return to the car to call emergency response 911 for help. C) Open the airway and initiate resuscitative measures. D) Examine the victim's body surfaces for arterial bleeding.

a

A nurse who is reassigned to the emergency department needs to understand that gastric lavage is a priority in which situation? A) An infant who has been identified to have botulism B) A toddler who ate a number of ibuprofen tablets C) A preschooler who swallowed powdered plant food D) A school aged child who took a handful of vitamins

a

A nurse who usually works on a step-down unit is moved to work a 12-hour shift in the critical care unit. Which client is best for the charge nurse to assign to this nurse? A) A ventilator dependent client with chronic obstructive pulmonary disease COPD. B) A client who has a new onset diabetic ketoacidosis (DKA) and is on an insulin drip. C) A client admitted for a narcotic overdose who is ventilated with respiratory alkalosis. D) A ventilated client admitted today with respiratory failure and respiratory acidosis.

a

A nurse working on a surgical unit is developing a care plan for a client who has paraplegia. The client has an area of non-blanchable erythema over his ischium. Which of the following interventions should the nurse include in the care plan? a. Teach the client to shift his weight every 15 minutes while sitting b. Place the client upright on a donut-shaped cushion c. Assess pressure points every 24 hr. d. Turn and reposition the client every 3 hrs. while in bed.

a

A school nurse is teaching a parent about absence seizures. Which of the following information should the nurse include? a. "This type of seizure can be mistaken for daydreaming" b. "The child usually has an aura prior to onset" c. "This type of seizure last 30-60 sec" d. "This type of seizure has a gradual onset"

a

A seriously ill male client is transferred to the health care facility in a different state. Included in his records are advanced directive and a physician orders for life sustaining treatment. However, the state to which he is transferred does not endorse POLST. The client lapses into a coma shortly after admission to the new facility. What action should the nurse take? a. Request that the new health care provider cosine the POLST document b. Implement the clients wishes as described in his advanced directive c. Ask the clients family to make life sustaining treatment decisions d. Attached an advance directive copy to a medical record prescription page

a

A young adult female college student visits the health clinic in early winter to obtain birth control pills. The clinic nurse asks if the student has received an influenza vaccination. The student stated she did not receive vaccination because she has asthma. How should the nurse respond? A) Offer to provide the influenza vaccination to the student while she is at the clinic. B) Encourage the student to obtain a vaccination prior to the next influenza season. C) Confirm that a history of asthma can increase risks associated with the vaccine. D) Advise the student that the nasal spray vaccine reduces side effects for people with asthma.

a

After a client has an enteral feeding tube inserted, the most accurate method for verification of placement is A) Abdominal x-ray B) Auscultation C) Flushing tube with saline D) Aspiration for gastric contents

a

After receiving IV fluids in the emergency department, an elderly client is admitted to the acute care unit with a medical diagnosis of dehydration. The client is receiving 0.9% normal saline at 125 ml/hour via saline lock and has a bounding pulse, tachycardia, and pedal edema. When contacting the healthcare provider, the nurse anticipates a prescription what intervention? A) Remove the saline lock from the client's arm. B) Increase the rate of the normal saline infusion. C) Decrease the rate of the normal saline infusion. D) Change the IV solution to 0.45% saline solution.

a

After years of struggling with weight management, a middle-aged man is evaluated for gastroplasty. He has experienced difficulty with managing his diabetes mellitus and hypertension, but he is approved for surgery. Which intervention is most important for the nurse to include in this client's plan of care? a. Apply sequential compression stockings b. Monitor for urinary incontinence c. Observe for signs of depression d. Provide a wide variety of meal choices

a

An adult client experiences a gasoline tank fire when riding a motorcycle and is admitted to the emergency department (ED) with full thickness burns to all surfaces of both lower extremities. What percentage of body surface area should the nurse document in the electronic medical record (EMR)? A) 36% B) 9% C) 45% D) 15%

a

An adult female tells the nurse that her grandmother was diagnosed with colorectal cancer at age 75 and the client is implementing measures to reduce her own risk. Which of the client's plans indicates the need for additional information? A) Annual sigmoidoscopy screening. B) Increased intake of fresh fruits, vegetables, and whole grains. C) Reduced dietary intake of animal fat and protein. D) Yearly fecal occult blood testing.

a

An antibiotic IM injection for a 2 year-old child is ordered. The total volume of the injection equals 2.0 ml The correct action is to A) administer the medication in 2 separate injections B) give the medication in the dorsal gluteal site C) call to get a smaller volume ordered D) check with pharmacy for a liquid form of the medication skip

a

An elderly client admitted after a fall begins to seize and loses consciousness. What action by the nurse is appropriate to do next? A) Stay with client and observe for airway obstruction B) Collect pillows and pad the side rails of the bed C) Place an oral airway in the mouth and suction D) Announce a cardiac arrest, and assist with intubation

a

An unconscious client is admitted to the intensive care unit and is placed on a ventilator. The ventilator alarms continuously and the client's oxygen saturation level is 62%. What action should the nurse take first? A) Begin manual ventilation immediately. B) Silence the alarm and call the technician. C) Monitor manual ventilation immediately. D) Call respiratory therapy

a

As adult client's apical pulse is 110 beats per minute. What intervention should the nurse implement first? A) Assess the client's radial pulse and apical pulse at the same time. B) Assess the client to determine the reason why the pulse is elevated. C) Notify the charge nurse that the client's pulse is elevated. D) Attempt to calm the client and take the pulse again in one hour.

a

During a postpartum assessment of a client who is 5 hours post vaginal delivery, the nurse determines the fundus is 3 finger breadths above the umbilicus and positioned to the client's side. Which action should the nurse implement first? A) Encourage the client to void. B) Massage the fundus until firm. C) Catheterize for residual urinary volume. D) Provide additional oral replacement fluids.

a

Following the evacuation of a subdural hematoma, an older adult develops an infection. The client is transferred to the near intensive care unit with a temperature of 102.8 F (39.3 C) axillary, pulse of 180 beats/minute, and a blood pressure of 90/60. What its the priority intervention to include in the client's plan care? A) Maintain intravenous access. B) Keep the suture line clean and dry. C) Measure hourly urine output. D) Check near vital signs q4 hours

a

Four hours after the nurse administers interferon alpha subcutaneously into a client, the client develops a headache, muscle aches and a fever of 101.8 degrees Fahrenheit. What action should the nurse implement? a. Administer prescribed PRN dose of acetaminophen for these side effects b. Explain that an antihistamine may be needed in response to this allergic reaction c. Document these findings as an idiosyncratic response to this medication d. Observed the site where the medication was injected for signs of local reaction

a

In early septic shock states, what is the primary cause of hypotension? A) Peripheral vasodilation. B) Cardiac failure. C) A vagal response. D) Peripheral vasoconstriction.

a

The health care provider order reads "aspirate nasogastric feeding (NG) tuber every 4 hours and check pH of aspirate." The pH of the aspirate is 10. Which action should the nurse take? A) Hold the tube feeding and notify the provider B) Administer the tube feeding as scheduled C) Irrigate the tube with diet cola soda D) Apply intermittent suction to the feeding tube

a

The healthcare provider prescribes a sedative for a client with severe hypothyroidism. The nurse plans to contact the provider to review the safety of the prescription for the client and consultS first with the charge nurse. The charge nurse notes that the prescription is written legally and completely. How should the charge nurse respond? a. Affirm the nurses plan to review the prescription with the provider b. Advise the nurse to administer the medication as prescribed c. Assume responsibility for discussing the concern with the provider d. Offer to administer the prescription since the nurse has concerns

a

The nurse assesses a client with new onset diarrhea. It is most important for the nurse to question the client about recent use of which type of medication? A) Antibiotics. B) Anticoagulants. C) Antihypertensive. D) Anticholinergics.

a

The nurse completed a dressing change for a client with partial thickness burns to both legs. After completing the dressing change, What intervention should the nurse implement? A) Administer a PRN dose of pain medication. B) Raise this head of bed to a 90 angle. C) Perform passive range of motion. D) Position ankles in a dorsiflexed position.

a

The nurse finds a client at 33 weeks gestation in cardiac arrest. What adaptation to cardiopulmonary resuscitation (CPR) should the nurse implement? A) Position a firm wedge to support pelvis and thorax at 30 degree tilt. B) Apply oxygen by mask after opening the airway. C) Apply less compression force to reduce aspiration. D) Give continuous compression with a ventilation ration at 20:3.

a

The nurse identifies an electrolyte imbalance, an elevated pulse rate, and a weight gain of 4.4 lbs (2 kg) in 24 hours for a client with chronic kidney disease. What intervention should the nurse include in the plan of care? a. Monitor serum electrolytes daily. b. Provide only distilled water. c. Document abdominal girth. d. Perform range of motion exercises.

a

The nurse is assessing a client with a closed head injury sustained in a motor vehicle collision. Which finding indicates the lowest neurologic functioning? A) Decerebrate posturing during position changes. B) Withdrawal from painful stimuli. C) Decorticate posturing during tracheal suctioning. D) Localization of a tactile stimulus.

a

The nurse is assigned to provide care for a client who is scheduled for a laparoscopic cholecystectomy in two hours, at 0900, what nursing action is most important? a. Confirm that the client has been NPO since midnight. b. Review postoperative instructions with the client. c. Offer to assist the client to the restroom to void. d. Determine when the client last had pain medication.

a

The nurse is caring for a 69 year-old client with a diagnosis of hyperglycemia. Which tasks could the nurse delegate to the unlicensed assistive personnel (UAP)? A) Test blood sugar every 2 hours by accu check B) Review with family and client signs of hyperglycemia C) Monitor for mental status changes D) Check skin condition of lower extremities

a

The nurse is caring for a client receiving a blood transfusion who develops urticaria one-half hour after the transfusion has begun. What is the first action the nurse should take? A) Stop the infusion B) Slow the rate of infusion C) Take vital signs and observe for further deterioration D) Administer Benadryl and continue the infusion

a

The nurse is caring for a client with clinical depression who is receiving a MAO inhibitor. When providing instructions about precautions with this medication, which action should the nurse stress to the client as important? A) Avoid chocolate and cheese B) Take frequent naps C) Take the medication with milk D) Avoid walking without assistance

a

The nurse is developing a plan of care for a client who reports tingling of the feet and who is newly diagnosed with peripheral vascular disease. Which outcome should the nurse include in the plan of care for this client? A. The client's skin on the lower legs will be intact at the next clinic visit B. The client will express acceptance of their newly diagnosed health status C. The client's blood pressure readings will be less than 160/90 mmHg D. The nurse will encourage the client to walk thirty minutes every day

a

The nurse is instructing a 65 year-old female client diagnosed with osteoporosis. The most important instruction regarding exercise would be to A) Exercise doing weight bearing activities B) Exercise to reduce weight C) Avoid exercise activities that increase the risk of fracture D) Exercise to strengthen muscles and thereby protect bone

a

The nurse is performing an admission assessment for a newborn who has asymmetrical buttocks. Which assessment test results should the nurse report to the health care provider? a. Ortolani maneuver causing a click at the hip joint b. Plumb line test indicates fetal position curvature c. Babinski tests that reveals fanning out of toes d. Moro test precipitating a startle response

a

The nurse is planning care for a client who admits having suicidal thoughts. Which client behavior indicates the highest risk for the client acting on these suicidal thoughts? A) Begin to show signs of improvement in affect. B) Lacks interest in the activity of the family and friends. C) Express feelings of sadness and loneliness. D) Neglects personal hygiene and has no appetite.

a

The nurse is preparing to administer a tube feeding to a post-operative client. To accurately assess for a gastrostomy tube placement, the priority is to A) Auscultate the abdomen while instilling 10 cc of air into the tube B) Place the end of the tube in water to check for air bubbles C) Retract the tube several inches to check for resistance D) Measure the length of tubing from nose to epigastrium

a

The nurse is teaching a group of clients with rheumatoid arthritis about the need to modify daily activities. Which goal should the nurse emphasize? A) Protect joint function. B) Improve circulation. C) Control tremors. D) Increase weight bearing.

a

The nurse notes that a postoperative adult client's respiratory rate is 10 breaths/minute. Which factor is the most likely explanation for this finding? a. The client's PCA pump with morphine sulfate was discontinued 15 minutes ago b. The client's hemoglobin is 10.1 gm/dl and hematocrit is 30.4% c. The client has a 20-pack year history of smoking cigarettes d. The client has a history of allergic bronchitis with recurrent bacterial pneumoni

a

The nurse working in a critical care unit is assigned the care of two clients, one with pneumonia who is being mechanically ventilated and the other who had a thoracotomy yesterday and is complaining of incisional pain. What should the nurse do first? a. Change the surgical dressing to observe the appearance of the incision b. Assess the level of consciousness and vital signs for both clients c. Review the plan of care and the medications that are due for both clients d. Complete a head-to-toe assessment of the client with pneumonia

a

To prevent unnecessary hypoxia during suctioning of a tracheostomy, the nurse must A) Apply suction for no more than 10 seconds B) Maintain sterile technique C) Lubricate 3 to 4 inches of the catheter tip D) Withdraw catheter in a circular motion

a

When assessing a client, it is important for the nurse to be informed about cultural issues related to the client's background because A) Normal patterns of behavior may be labeled as deviant, immoral, or insane B) The meaning of the client's behavior can be derived from conventional wisdom C) Personal values will guide the interaction between persons from 2 cultures D) The nurse should rely on her knowledge of different developmental mental stages

a

When the nurse attempts to teach self-administration of insulin injections to a client who is newly diagnosed with type one diabetes mellitus, the client tells the nurse in a loud voice to leave the room. What action should the nurse take? a. Leave the clients room and return later in the day b. Explain that insulin is a life saving drug for the client c. Encourage client to implement relaxation techniques d. Refer the client to the social worker for support therap

a

Which of these statements by the nurse reflects the best use of therapeutic interaction techniques? A) "You look upset. Would you like to talk about it?" B) "I'd like to know more about your family. Tell me about them." C) "I understand that you lost your partner. I don't think I could go on if that happened to me." D) "You look very sad. How long have you been this way?"

a

Which statement by the nurse is appropriate when asking an unlicensed assistive personnel (UAP) to assist a 69 year-old surgical client to ambulate for the first time? A) "Have the client sit on the side of the bed for at least 2 minutes before helping him stand." B) "If the client is dizzy on standing, ask him to take some deep breaths." C) "Assist the client to the bathroom at least twice on this shift." D) "After you assist him to the chair, let me know how he feels."

a

Why is it important for the nurse to monitor blood pressure in clients receiving antipsychotic drugs? A) Orthostatic hypotension is a common side effect B) Most antipsychotic drugs cause elevated blood pressure C) This provides information on the amount of sodium allowed in the diet D) It will indicate the need to institute anti parkinsonian drugs

a

a staff nurse is observing a newly licensed nurse suction a client's tracheostomy. Which of the following requires intervention by the staff nurse? A. Waits 2 minutes between suctions. B. Encourages the client to cough during suctioning. C. Apply suctioning for 15 seconds. D. Inserts the catheter without applying suction

a

he nurse is performing an assessment on a client in congestive heart failure. Auscultation of the heart is most likely to reveal A) S3 ventricular gallop B) Apical click C) Systolic murmur D) Split S2

a

************When conducting diet teaching for a client who was diagnosed with hypertension, which food should the nurse encourage the client to eat? (select all that apply.) a.Fruits without sauce b. Canned soup. c. Fresh or frozen vegetables without sauce. d. Cottage cheese. e. Pickled olives.

a c d

To reduce the risk of symptoms exacerbation for a client with multiple sclerosis (MS), which instructions should the nurse include in the client's discharge plan? (Select all that apply). A) Practice relaxation exercises B) Limit fluids to avoid bladder distention C) Space activities to allow for rest periods D) Avoid persons with infections E) Take warm baths before starting exercise

a c d

A nurse is caring for a client who has vision loss. Which of the following actions should the nurse take? SATA a. keep objects in the clients room in the same place. b. ensure there is high-wattage lighting in the clients room. c. approach the client from the side d. allow extra time for the client to perform tasks e. touch the client gently to announce presence

a, b, d

A nurse is caring for a client who has type 1 diabetes mellitus. The client reports that she is not feeling well. Which of the following findings should indicate to the nurse that the client is hypoglycemic? (Select all that apply.) a. Tremors b. Polydipsia c. Acetone Breath odor d. Diaphoresis

a, d

61-An older client is admitted for repair of a broken hip. To reduce the risk for infection in the postoperative period, which nursing care interventions should the nurse include in the client's plan of care? (Select all that apply) A) Teach client to use incentive spirometer q2 hours while awake B) Remove urinary catheter as soon as possible and encourage voiding C) Maintain sequential compression devices while in bed D) Administer low molecular weight heparin as prescribed E) Assess pain level and medicate PRN as prescribed

a,b

A client is receiving ophthalmic drops preoperatively for a cataract extraction and asks the nurse why the healthcare provider has prescribed all these medications. Which information should the nurse included when responding to this client? (Select all that apply.) A) One of the medications is used to anesthetize the corneal surface B) Pupillary dilation is necessary to access the eye chamber for lens removal C) The iris must be paralyzed during surgery to prevent it from reacting to light D) A medication is used to induce sleep during the procedure E) These medications assist in obstructing client´s vision during the surgery

a,b,c

An older woman who lives alone in a two-story home is admitted after falling while shopping. X-rays reveal a fractured left hip. With no immediate family in the area, the client is concerned about her pets. Which interventions should the nurse implement? (Select all that apply) A) Palpate and mark pedal pulses. B) Alert social work of client's concerns C) Assess ability to bear weight when standing. D) Evaluate pain using standard pain scale. E) Support left leg with two pillows.

a,b,d

While assessing a client's chest tube (CT), the nurse discovers bubbling in the water seal chamber of the chest tube collection device. The client's vital signs are: blood pressure of 80/40 mmHg, heart rate 120 beats/minutes, respiratory rate 32 breaths/minutes, oxygen saturation 88%. Which interventions should the nurse implement? (Select all that apply). A) Provide supplemental oxygen B) Auscultate bilateral lung fields C) Administer a nebulizer treatment D) Reinforce occlusive CT dressing E) Give PRN dose of pain medication

a,b,d

A client newly diagnosed with diabetes mellitus suddenly becomes confused and weak.Which interventions should the nurse implement? (Select all that apply) A) Give the client 4 ounces of orange juice B) Obtain blood pressure and pulse rate C) Provide the client with ½ cup diet carbonated soda D) Administer a PRN dose of regular insulin E) Check the client's current finger stick blood glucose

a,b,e

A client who is hospitalized and recently diagnosed with Addison's disease is now confused and lethargic. Which actions should the nurse implement? (Select all that apply) A) Measure capillary glucose level B) Monitor cardiac telemetry pattern C) Reduce rate of intravenous fluid infusion D) Withhold next dose of corticosteroid E) Initiate fall risk precautions

a,b,e

A client with Addison's crisis is admitted for treatment with adrenal cortical supplementation. Based on the client's admitting diagnosis, which findings require immediate action by the nurse? (Select all that apply) a. Headache and tremors b. Irregular heart rate c. Skin hyperpigmentation d. Postural hypotension e. Pallor and diaphoresis

a,b,e

********The nurse is teaching a primigravida about preeclampsia. Which finding are indicators of preeclampsia and should be reported to the healthcare provider? (select all that apply.) a. Blurred vision b. Headache. c. Lack of appetite. d. Urinary frequency. e. Chills and fever. f. Swollen hands.

a,b,f

**************A client with cirrhosis of the liver is admitted with complications related to end stage liver disease. Which interventions should the nurse implement? (sata) A. Report serum albumin and globulin levels B. Provide diet low in phosphorus C. Note signs of swelling and edema D. Monitor abdominal girth E. Increase oral fluid intake to 1,500 mL daily

a,c,d

A client with deep vein thrombosis (DVT) is receiving a continuous intravenous heparin infusion. The client now has tarry, black diarrhea and reports abdominal pain. Which action should the nurse implement? (Select all that apply.) A) Monitor stools for presence of blood. B) Auscultate bowel sounds in all quadrants. C) Assess characteristics of pain. D) Review last partial thromboplastin time results. E) Prepare to administer warfarin.

a,c,e

A male client is admitted for the removal of an internal fixation that was inserted for the fracture ankle. During the admission history, he tells the nurse he recently received vancomycin (vancomycin) for a methicillin-resistant Staphylococcus aureus (MRSA) wound infection. Which action should the nurse take? (Select all that apply.) A) Collect multiple site screening culture for MRSA. B) Call healthcare provider for a prescription for linezolid (Zyrovix). C) Place the client on contact transmission precautions. D) Obtain sputum specimen for culture and sensitivity. E) Continue to monitor for client sign of infection.

a,c,e

An older adult resident of a long-term care facility has a 5-year history of hypertension. The client has a headache and rate the pain 5 on a pain scale 0 to 10. The client's blood pressure is currently 142/89. Which interventions should the nurse implement? (Select all that apply) A) Administer a daily dose of lisinopril as scheduled. B) Assess the client for postural hypotension. C) Notify the healthcare provider immediately D) Provide a PRN dose of acetaminophen for headache E) Withhold the next scheduled daily dose of warfarin

a,d

An older adult client is admitted to the stroke unit after recovery from the acute phrase of an ischemic cerebral vascular accident (CAV). Which interventions should the nurse include in the plan of care during convalescence and rehabilitation? (Select all that apply.) A) Measure neurological vital signs every 4 hours B) Place a bed side commode next to bed C) Suction oral cavity every 4 hours. D) Encourage family participate in the client's care E) Play classical music in room while client is awake

a,d,e

A client with eczema is experiencing severe pruritus. Which PRN prescriptions should the nurse administer? (Select all that apply) A) Topical corticosteroid. B) Topical scabicide. C) Topical alcohol rub. D) Transdermal analgesic. E) Oral antihistamine.

a,e

*************A client arrives for an annual physical exam and complains of having calf pain. The client's health history reveals peripheral atrial disease. Which question should the nurse ask the client about expected finding related to chronic arterial symptoms? a. Were your legs ever suddenly swollen, red, warm, and painful? b. Does the calf pain occur when walking short distances? c. Did you receive treatment for weeping ulcers on lower legs? d. Have you experienced ankle edema and varicose veins?

b

*************A client with Addison's disease becomes weak, confused, and dehydrated following the onset of an acute viral infection. The client's laboratory values include sodium 129 mEq/l (129mmol/l SI), glucose 54 mg/dl (2.97mmol/l SI) and potassium 5.3 mmol/l SI). When reporting the findings to the HCP, the nurse anticipates a prescription for which intravenous medications? A) Regular insulin. B) Hydrocortisone. C) Broad spectrum antibiotic. D) Potassium chloride.

b

*************A client with metabolic syndrome plans to begin an exercise program. Which instruction is most important for the nurse to provide this client? A. Wear long sleeves and a hat when exercising outdoors in direct sunlight B. Monitor blood pressure and heart rate as exercise activity is increased C. Weight bearing exercises are most effective in improving bone strength D. Use hand-held weights to strengthen muscles and build muscle mass

b

*************The nurse caring for a child with mononucleosis can expect the child to exhibit which symptoms? a. Positive Epstein-Barr, and malaise. b. Ear pain and fever. c. Elevated WBC and sedimentation rate. d. Increased BUN and serum creatinine.

b

************A client is receiving IV heparin and oral warfarin after a pulmonary embolism (PE). The nurse determines the client's activated partial prothromboplastin time (aPTT) value is two times the control value; the prothrombin time (PT) level is the same as teh control, and the international normalized ratio (INR) is 1. Which protocol prescription should the nurse implement? A. Withhold the heparin and continue the same dose of warfarin B. Increase the warfarin dose C. Decrease the heparin dose D. Increase the heparin dose and decrease the warfarin dose

b

************An adult male was diagnosed with stage IV lung cancer three weeks ago. His wife approaches the nurse and asks how she will know that her husband's death is imminent because their two adult children want to be there when he dies. Which is the best response by the nurse? A. Gather information regarding how long it will take for the children to arrive B. Explain that the client will start to lose consciousness and the body systems will slow down C. Reassure the spouse that the healthcare provider will notify when to call the children D. Offer to discuss the client's health status with each of the adult children

b

************An older adult male who is in his early 70's is admitted to the emergency department because of a COPD exacerbation. This client is struggling to breathe and the healthcare team is preparing for endotracheal intubation. The spouse's wife, who is 30 years younger than the client, asks the nurse to stop the procedure and provide the nurse a copy of the client's living will. Which action should the nurse take? a. Facilitate a family meeting with the palliative care team. b. Notify the healthcare provider of the client's wishes. c. Place a certified copy of the living will in the client's record. d. Alert the nursing staff of the client's don't resuscitate status.

b

************An older client is admitted to the hospital because of recurring transient ischemic attacks. Neurological serial assessments for the past 24 hours were within normal limits. One day after admission, the client suddenly becomes confused and combative indicating impaired mental status (IMS). What intervention should the nurse implement first? a. Document neurologic changes. b. Reduce environmental stimuli. c. Administer prescribed neuroleptic. d. Review medications for interactions.

b

************The public health nurse receives funding to initiate a primary prevention program in the community. Which program best fits the nurse's proposal? A. Regional relocation center for earthquake victims B. Vitamin supplements for high-risk pregnant women C. Lead screening for children in low-income housing D. Case management and screening for clients with HIV

b

************When assessing a 6-month-old infant, the nurse determines that the anterior fontanel is bulging. In which situation would this findings be most significant? A. Crying B. Sitting upright C. Vomiting D. Straining on stool

b

************Which instruction should the nurse provide to a client who is preparing to have a cystoscopy? A. Report any allergies to shellfish or iodine B. Report any painful urination, blood in urine, or fever C. Lay prone for 24 hours after the procedure D. Avoid strenuous activity and sports for at least 2 weeks

b

************Which laboratory results should the nurse closely monitor in a client who has end-stage renal disease (ESRD)? A. Leukocytes, neutrophils, and thyroxine B. Serum potassium, calcium, and phosphorus C. Blood pressure, heart rate, and temperatue D. Erythrocytes, hemoglobin, and hematocrit

b

***********A 12-year-old client who had an appendectomy two days ago is receiving 0.9% normal saline at 50mL/hr. The client's urine specific gravity is 1.035. Which action should the nurse implement? A. Assess bowel sounds in all quadrants B. Encourage popsicles and fluids of choice C. Evaluate postural blood pressure measurements D. Obtain a specimen for urinalysis

b

***********During a clinic visit, a client with a kidney transplant asks, "What will happen if chronic rejection develops?" Which response is best for the nurse to provide? A. A different combination of immunosuppressant medications will be implemented B. Dialysis would need to be resumed if chronic rejection becomes a reality C. Dialysis may be necessary until the chronic rejection can be reversed D. The immunosuppressant medication will be increased until the rejection subsides

b

***********When developing a teaching plan for a client with newly diagnosed type 1 diabetes, the nurse should explain that an increased thirst is an early sign of diabetic ketoacidosis (DKA). Which action should the nurse instruct the client to implement if this sign of DKA occurs? A. Resume normal physical activity B. Give a dose of regular insulin as prescribed C. Measure urine output over the next 24 hours D. Drink electrolyte fluid replacements

b

**********A client who is admitted to the care unit with syndrome of inappropriate antidiuretic hormone (SIADH) has developed osmotic demyelination. Which intervention should the nurse implement first? A) Patch one eye. B) Evaluate swallow. C) Reorient often. D) Range of motion.

b

**********A client with a history of using illicit drugs intravenously is admitted with Kaposi's sarcoma. Which intervention should the nurse include in this client's admission plan of care? A. Assess for symptoms of AIDS dementia B. Monitor for secondary infections C. Identify local HIV support groups D. Observe for adverse drug reactions

b

**********The nurse is planning to teach infant care and preventive measures for sudden infant death syndrome (SIDS) to a group of new parents. Which information is most important for the nurse to include? A. Swaddle the infant in a blanket for sleeping B. Ensure that the infant's crib mattress is firm C. Place the infant in a prone position whenever possible D. Prop the infant with a pillow when in a side-lying position

b

**********The nurse is teaching a client newly diagnosed with systemic lupus erythematosus (SLE). Which information is accurate for the nurse to provide? A. The client can expect to progressively lose function in a fairly predictable sequence B. The disease is characterized by alternating periods of flare-ups and remissions C. Once an acute attack subsides, the client can expect to feel fine again D. Systemic lupus erythematosus (SLE) is a chronic, incurable, terminal illness

b

*********A client with rheumatoid arthritis (RA) starts a new prescription for etanercept subcutaneously once weekly. The nurse should emphasize the importance of reporting which problem to the healthcare provider? A. Joint stiffness B. Persistent fever C. Headache D. Increased hunger and thirst

b

*********An older client is admitted to the psychiatric unit for assessment of a recent onset of dementia. The nurse notes that in the evening this client often becomes restless, confused, and agitated. Which intervention is most important for the nurse to implement? a. Ask family members to remain with the client in the evening from 1700 to 2100 p.m. b. Ensure that the client is assigned to a room close to the nurses' station. c. Postpone administration of nighttime medications until after 2300 p.m. d. Administer a prescribed PRN benzodiazepine at the onset of a confused state

b

*********In caring for a client with Cushing syndrome, which serum laboratory value is most important for the nurse to monitor? A) Lactate. B) Glucose. C) Hemoglobin. D) Creatinine.

b

*********While caring for a toddler receiving oxygen via face mask, the nurse observes that the child's lips and nares are dry and cracked. Which intervention should the nurse implement? A. Use a topical lidocaine analgesic for cracked lips B. Use a water soluble lubricant on affected oral and nasal mucosa C. Ask the mother what she usually uses on the child's lips and nose D. Apply a petroleum jelly to the child's lips and nose

b

********A male client with right-sided weakness calls for assistance with ambulating to the bathroom. What action should the nurse implement? a. Bring a bedside commode to the client. b. Stand on the client's right side as he walks. c. Walk directly behind the client to prevent a fall. d. Give the client a cane to hold in his right hand.

b

********The nurse is providing care for a child who is brought to the emergency department a few days after a laceration to the leg from a barbed wire fence. The child has not received any tetanus immunizations and is manifesting early signs of muscular rigidity with spasms and jaw clenching or trismus. Which intervention should be the nurse's highest priority for this child? A. Suction oropharyngeal secretions B. Prepare for intubation with mechanical ventilation C. Minimize stimulation from sound, light, and touch D. Monitor IV infusion

b

*******A client with atrial fibrillation receives a new prescription for dabigatran etexilate. Which instruction is important for the nurse to emphasize when teaching the client about this medication? A. Monitor your blood pressure regularly B. Report unusual bruising or bleeding C. Elevate your feet if swelling occurs D. Check your pulse rate every day

b

*******A client with postpartum depression, who is admitted to the behavioral health unit, refuses to leave her room or eat meals. In addition to maintaining physical safety, which short-term goal should the nurse include in the plan of care? A. Sleeps at least 6 hours per night B. Consumes 3 meals and 1500 mL of fluid per day C. Engages in one client to client interaction daily D. Attends one group activity per day

b

*****a multiparous women who delivered her first infant three hours ago asks the nurse __? because it helped reduce perineal pain after her first delivery. What action should the nurse take? a. apply icepack to the perineum for the first 24 hours b. review the use of sitz bath equpiment with the client c. teach the client how to practice kegel excercises d. use an analgesic spray to perineal area

b

A 17-year-old adolescent is brought to the Emergency Department by both parents because the adolescent has been coughing and running a fever with flu-like symptoms for the past 24 hours. Which intervention should the nurse implement first? A) Assess the client's temperature. B) Place a mask on the client's face. C) Determine the client's blood pressure. D) Obtain a chest x-ray per protoco

b

A Nurse is working with a client who has an anxiety disorder and is in the orientation phase of the therapeutic relationship. Which of the following statements should the nurse make during this phase? a. "Let's talk about how you can change your response to stress." b. "We should establish our roles in the initial session." c. . "Let me show you simple relaxation exercises to manage stress." d. "We should discuss resources to implement in your daily life."

b

A client at 28 weeks' gestation is admitted to the obstetrical unit following her involvement in a motor vehicle collision. After stabilizing the client, the nurse obtains a fetal monitor reading. What action should the nurse take if fetal tachycardia is assessed on the monitor? a. Recount the heart rate manually to confirm a monitor malfunction b. Contact the health care provider after initiating oxygen per face mask c. Explain that there is no indication the fetal heart rate is due to trauma d. Evaluate the presence of preterm labor by performing a vaginal examination

b

A client becomes increasingly lethargic and has a respiratory rate of 8 breaths per minute with 30-second periods of apnea, the healthcare provider is notified, and STAT arterial blood gases are drawn. What ABG results should the nurse anticipate? a. Compensated respiratory acidosis b. Uncompensated respiratory acidosis c. Uncompensated metabolic acidosis d. Compensated metabolic acidosis

b

A client continuously calls out to the nursing staff when anyone passes the client's door and asks them to do something in the room. The best response by the charge nurse would be to A) Keep the client's room door cracked to minimize the distractions B) Assign one of the nursing staff to visit the client regularly C) Reassure the client that one staff person will check frequently if the client needs anything D) Arrange for each staff member to go into the client's room to check on needs every hour on the hour

b

A client has been diagnosed with Zollinger-Ellison syndrome.Which information is most important for the nurse to reinforce with the client? a. It is a condition in which one or more tumors called gastrinomas form in the pancreas or in the upper part of the small intestine (duodenum) b. It is critical to report promptly to your health care provider any findings of peptic ulcers c. Treatment consists of medications to reduce acid and heal any peptic ulcers and, if possible, surgery to remove any tumors d. With the average age at diagnosis at 50 years the peptic ulcers may occur at unusual areas of the stomach or intestine

b

A client is admitted for first and second degree burns on the face, neck, anterior chest and hands. The nurse's priority should be A) Cover the areas with dry sterile dressings B) Assess for dyspnea or stridor C) Initiate intravenous therapy D) Administer pain medication

b

A client is admitted with a diagnosis of schizophrenia. The client refuses to take medication and states "I don't think I need those medications. They make me too sleepy and drowsy. I insist that you explain their use and side effects." The nurse should understand that A) A referral is needed to the psychiatrist who is to provide the client with answers B) The client has a right to know about the prescribed medications C) Such education is an independent decision of the individual nurse whether or not to teach clients about their medications D) Clients with schizophrenia are at a higher risk of psychosocial complications when they know about their medication side effects

b

A client is being discharged with a prescription for chlorpromazine (Thorazine). Before leaving for home, which of these findings should the nurse teach the client to report? A) Change in libido, breast enlargement B) Sore throat, fever C) Abdominal pain, nausea, diarrhea D) Dsypnea, nasal congestion

b

A client is diagnosed with a spontaneous pneumothorax necessitating the insertion of a chest tube. What is the best explanation for the nurse to provide this client? A) "The tube will drain fluid from your chest." B) "The tube will remove excess air from your chest." C) "The tube controls the amount of air that enters your chest." D) "The tube will seal the hole in your lung.

b

A client is receiving Total Parenteral Nutrition (TPN) via Hickman catheter. The catheter accidentally becomes dislodged from the site. Which action by the nurse shouldtake priority? A) Check that the catheter tip is intact B) Apply a pressure dressing to the site C) Monitor respiratory status D) Assess for mental status changes

b

A client is receiving external beam radiation to the mediastinum for treatment of bronchial cancer. Which of the following should take priority in planning care? A) Esophagitis B) Leukopenia C) Fatigue D) Skin irritation

b

A client recovering from pneumonia who has a history of severe chronic obstructive pulmonary disease (COPD) and peripheral vascular disease (PVD) is being discharged from a skilled nursing facility. Which action is most important for the nurse to implement? A) Provide typed instructions for healthy diet selections .B) Reinforce need for adequate hydration. C) Explain exercises daily regimen. D) Demonstrate specific strengthening exercises.

b

A client taking isoniazide (INH) for tuberculosis asks the nurse about side effects of the medication. The client should be instructed to immediately report which of these? A) Double vision and visual halos B) Extremity tingling and numbness C) Confusion and lightheadedness D) Sensitivity of sunlight

b

A client tells the nurse, "I have something very important to tell you if you promise not to tell." The best response by the nurse is A) "I must document and report any information." B) "I can't make such a promise." C) "That depends on what you tell me." D) "I must report everything to the treatment team."

b

A client with a new diagnosis of diabetes mellitus is referred for home care. A family member present expresses concern that the client seems depressed. The nurse should initially focus assessment by using which approach? A) The results of a standardized tool that measures depression B) Observation of affect and behavior C) Inquiry about use of alcohol D) Family history of emotional problems or mental illness

b

A client with bleeding esophageal varies receives vasopressin IV. What should the nurse monitor for during the IV infusion of this medication? A) Vasodilation of the extremities. B) Chest pain and dysrhythmia. C) Hypotension and tachycardia. D) Decreasing GI cramping and nausea.

b

A client with diabetic peripheral tells the nurse that her healthy "chubby" baby is irritable and not very active. After obtaining a dietary history, the nurse determines that the infant refuses to eat any infant cereals. Which finding is most important to report to the healthcare provider? A) Breast feeds 10 minutes at night to go to sleep. B) Has porcelain-like skin and tripled birth weight. C) Does not take an infant vitamin supplement. D) Ingests 6 ten-ounce bottles of cow's milk daily.

b

A client with history of adrenal insufficiency is admitted to the intensive care unit with an acute adrenal crisis. The client is complaining of nausea and joint pain.Vital signs are: temperature 102 F (38.9 C) heart rate 138 beats/minute, blood pressure 80/60 mmHg. Which intervention should the nurse implement first? A) Administer PRN oral antipyretic. B) Infuse an intravenous fluid bolus. C) Obtain an analgesic prescription. D) Cover client with cooling blanket.

b

A client with pneumococcal pneumonia had been started on antibiotics 16 hours ago. During the nurse's initial evening rounds the nurse notices a foul smell in the room. The client makes all of these statements during their conversation. Which statement would alert the nurse to a complication? A) "I have a sharp pain in my chest when I take a breath." B) "I have been coughing up foul-tasting, brown, thick sputum." C) "I have been sweating all day." D) "I feel hot off and on.

b

A male client is admitted with a bowel obstruction and intractable vomiting for the last several hours despite the use of antiemetics. Which intervention should the nurse implement first? a. Maintain head of bed at 45 degrees b. Infuse 0.9% sodium chloride 500 ml bolus c. Insert nasogastric tube to intermittent suction d. Document strict and intake and output

b

A male client who is experiencing musculoskeletal pain is discharged with instructions to take ibuprofen, on non-steroidal anti-inflammatory drug by mouth BID. After receiving discharge teaching, the client states he plans to take the medication twice daily, with breakfast and dinner. How should the nurse respond? a. Review the need to limit intake of leafy, green vegetables such as spinach b. Confirm that the client has an effective plan for when to take the medication c. Explain the need to take the medication before meals to increase absorption d. Remind the client to increase fluid intake while taking the medication

b

A male client with heart failure becomes short of breath, anxious, and has audible wheezing with pink frothy sputum. The nurse sits the client and provides oxygen per nasal cannula. The nurse receives a prescription to administer a one-time dose of morphine sulfate intravenously. Which action should the nurse take? a. Withhold the morphine until the client's dyspnea resolves b. Administer the dose of morphine sulfate as prescribed c. Consult with the charge nurse regarding the morphine prescription d. Review the need for the prescription with the health care provide

b

A mother brings her 3-week-old son to the clinic because he is vomiting "all the time." In performing a physical assessment, the nurse notes that the infant has poor skin turgor, has lost 20% of his birth weight, and has a small palpable oval-shaped mass in his abdomen. Which intervention should the nurse implement first? a. Insert a nasogastric tube for feeding b. Initiated a prescribed IV for parenteral fluid c. Give the infant 5% dextrose in water orally d. Feed the infant 3 ounces of isomil

b

A mother brings her 4-month-old son to the clinic with a quarter taped over his umbilicus and tells the nurse the quarter is supposed to fix her child's hernia. Which explanations should the nurse provide? A) An abdominal binder can be worn daily to reduce the protrusion. B) This hernia is a normal variation that resolves without treatment. C) The quarter should be secured with an elastic bandage wrap. D) Restrictive clothing will be adequate to help the hernia go away.

b

A mother runs into the emergency department with a toddler in her arms and tells the nurse that her child got into some cleaning products. The child smells of chemicals on hands, face and on the front of the child's clothes. After ensuring the airway is patent, what action should the nurse implement first? A) Obtain equipment for gastric lavage. B) Determine type of chemical exposure. C) Assess child for altered sensorium. D) Call poison control emergency number

b

A nurse assesses a young adult in the emergency room following a motor vehicle accident. Which of the following neurological signs is of most concern? A) Flaccid paralysis B) Pupils fixed and dilated C) Diminished spinal reflexes D) Reduced sensory responses

b

A nurse from the maternity unit is floated to the critical care unit because of staff shortage on the evening shift. Which client would be appropriate to assign to this nurse? A client with A) A Dopamine drip IV with vital signs monitored every 5 minutes B) A myocardial infarction that is free from pain and dysrhythmias C) A tracheotomy of 24 hours in some respiratory distress D) A pacemaker inserted this morning with intermittent capture

b

A nurse in a PACU is transferring care of a client to a nurse on the medical-surgical unit. Which of the following statements should the nurse include in the hand-off report? a. The client was intubated without complications. b. The estimated blood loss was 250 milliliters. c. There was a total of 10 sponges used during the procedures. d. The client is a member of the board of directors.

b

A nurse in a surgical suite is planning care for a client who requires surgery and has a latex sensitivity. Which of the following is appropriate for this client? a. Disinfect and powder any latex products before use b. Tape stockinet over monitoring device and cords c. Schedule the client as the last surgery of the day d. Remove poop-socks from the IV

b

A nurse in an emergency department is caring for a client who received a dose of penicillin and is now anxious, flushing, tachycardic and has difficulty swallowing. Which of the following actions is the nurse's priority? a. Monitor the client's ECG b. Take the client's vital signs. c. Administer oxygen d. Insert an IV line.

b

A nurse in the emergency department suspects domestic violence as the cause of a client's injuries. What action should the nurse take first? A) Ask client if there are any old injuries also present B) Interview the client without the persons who came with the client C) Gain client's trust by not being hurried during the intake process D) Photograph the specific injuries in question

b

A nurse is administering digoxin 0.125 mg Po to an adult client. For which of the following findings should the nurse report to the provider? A. Potassium level 4.2 mEq/L. b. Apical pulse 58/min. c. Digoxin level 1 ng/ml. d. Constipation for 2 days.

b

A nurse is assessing a client who is 36 weeks of gestation. Which of the following findings should the nurse report to the provider? a. 3+ deep tendon reflexes b. Protruding Hemorrhoids c. Urinary Frequency d. Supine Hypotension

b

A nurse is assessing a client who is in active labor. Which of the following findings should the nurse report to the provider? A. Contractions lasting 80 seconds B. FHR baseline 170/min C. Early decelerations in the FHR D. Temperature 37.4C (99.3

b

A nurse is assessing the remote memory of an older adult client who has mild dementia. Which of the following questions should the nurse ask the client? a. "Can you tell me who visited you today?" b. "What high school did you graduate from c. "Can you list your current medications?" d. "What did you have for breakfast yesterday?"

b

A nurse is caring for a client following application of a cast. Which of the following actions should the nurse take first? a. place an ice pack over the cast b. palpate the pulse distal to the cast c. teach the client to keep the cast clean and dry d. position the casted extremity on a pillow

b

A nurse is caring for a client who has a new temporary synchronous pacemaker. Which of the following should the nurse report to the provider? a. The client's pulse oximetry level is 96%. b. The client develops hiccups. c. The ECG shows pacing spikes after the QRS complex.

b

A nurse is caring for a client who has arteriovenous fistula which of the following findings should the nurse report? a. Thrill upon palpation. b. Absence of a bruit. c. Distended blood vessels d. Swishing sound upon auscultation

b

A nurse is caring for a client who is 12 hr. postpartum and has a third- degree perineal laceration. The client reports not having a bowel movement for 4 days. Which of the following medications should the nurse administer? a. Bisacodyl 10 mg rectal suppository. b. Magnesium hydroxide 30 ml PO. c. Famotidine 20 mg PO. d. Loperamide 4 mg PO.

b

A nurse is caring for a client who is 4 hr postpartum and reports that she cannot urinate. Which of the following interventions should the nurse implement? a. Perform fundal massage b. Pour water from a squeeze bottle over the client's perineal area. d. Insert an indwelling urinary catheter. e. Apply cold therapy to the client's perineal area.

b

A nurse is caring for a client who is 4 hr postpartum and reports that she cannot urinate. Which of the following interventions should the nurse implement? a. Perform fundal massage b. Pour water from a squeeze bottle over the patients perineal area. c. Insert an indwelling urinary catheter. d. Apply cold therapy to the client's perineal area.

b

A nurse is caring for a client who is at 38 weeks gestation, is in active labor, and has ruptured membrane. Which of the following actions should the nurse take? a. Insert an indwelling urinary catheter. b. Apply fetal heart rate monitor. c. Initiate fundal massage. d. Initiate an oxytocin IV infusion.

b

A nurse is caring for a client who is in active labor and requests pain management. Which of the following actions should the nurse take? a. Administer ondansetron. b. Place the client in a warm shower. c. Apply fundal pressure during contractions. d. Assist the client to a supine position.

b

A nurse is caring for a client who is newly diagnosed with pancreatic cancer and has questions about the disease. To research the nurse should identify that which of the following electronic database has the most comprehensive collection nursing articles? a. medline b. Cinahl c. ProQuest d. health source

b

A nurse is caring for a client who is receiving brachytherapy for treatment of prostate cancer. Which of the following actions should the nurse take? a. Cleanse equipment before removal from the client's room. b. Limit the client's visitors to 30 min per day. c. Discard the client's linens in a double bag. d. Discard the radioactive source in a biohazard bag

b

A nurse is caring for a client who is receiving intermittent eternal tube feeding. Which of the following places the client at risk for aspiration? a. A residual of 65mL 1 hr postprandial b. A History of gastroesophageal reflux disease c. Sitting in a high-Fowler's position during the feeding d. Receiving a high osmolarity formula

b

A nurse is caring for a client who is receiving morphine 4 mg via IV bolus every 4 hr. PRN. The nurse should monitor for which of the following adverse effects? A. Productive cough. B. Urinary retention. C. Rhinitis D. Fever.

b

A nurse is caring for a toddler who has acute lymphocytic leukemia. In which of the following should the toddler participate? a. Looking at alphabet flashcards. b. Playing with a large plastic truck. c. Use scissors cut out paper shapes. d. Watching a cartoon in the dayroom.

b

A nurse is caring for four clients. Which of the following tasks can the nurse delegate to an assistive personnel? a. Assess effectiveness of antiemetic medication- b. Perform chest compressions during cardiac resuscitation- c. Perform a dressing change for a new amputee- d. Apply a transdermal nicotine patch-

b

A nurse is collecting a specimen for urinalysis and culture from a client who has an indwelling urinary catheter. Which of the following actions should the nurse take during collection? a. Drain the specimen from the drainage bag b. Clamp the catheter distal to the injection port c. Collect 2 mL of urine for each specimen d. Obtain the urinalysis specimen before the culture specimen

b

A nurse is conducting a health promotion class about the use of oral contraceptives. Which of the following disorders is a contraindication for oral contraceptive use? a. Asthma b. Hypertension c. Fibromyalgia d. Fibrocystic breast condition

b

A nurse is conducting a home visit for a family who has two young children. The nurse notes several welts across the backs of the legs of one of the children. Which of the following actions should the nurse take first? a. Document clinical findings. b. Contact child protective services. c. Refer the parents to a self-help group. d. Instruct the parents about methods of discipline.

b

A nurse is creating for a client who has aids. The client states, "My mouth is sore when I eat." Which of the following instructions should the nurse provide? A. "Add salt to season" B. "Ice chips" C. "Rinse your mouth with an alcohol-based mouthwash" D. "Eat foods served at hot temperatures

b

A nurse is devdeloping an in-service about personality disorders. Which of the following information should the nurse include when discussing borderline personality disorder? a. The client is overly concerned about minor details. b. The client exhibits impulsive behavior. c. The client is exceptionally clingy to others. d. The client may act seductively.

b

A nurse is planning care for a preschool-age child who is in the acute phase Kawasaki disease. Which of the following interventions should the nurse include in the plan of care? a. Give scheduled doses of acetaminophen every 6 hr b. Monitor the child's cardiac status c. Administer antibiotics via intermittent IV bolus for 24 hr d. Provide stimulation with children of the same age in the playroom

b

A nurse is providing care for a client who has esophageal cancer and has received radiation therapy. Which of the following finding should the nurse identify as the priority? a. Excoriation of the skin on the neck and chest b. Dysphagia c. Client reports a pain level of 6 on scale from 0-10 d. Xerostomia

b

A nurse is providing discharge teaching for a client who has an implantable cardioverter defibrillator which of the following statements demonstrates understanding of the teaching? a. "I will soak in the tub rather and showering" b. "I will wear loose clothing around my ICD" c. "I will stop using my microwave oven at home because of my ICD" d. "I can hold my cellphone on the same side of my body as the ICD"

b

A nurse is providing discharge teaching for a group of clients. The nurse should recommend a referral to a dietitian a. A client who has a prescription for warfarin and states "I will need to limit how much spinach I eat". b. A client who has gout and states, "I can continue to eat anchovies on my pizza." c. A client who has a prescription for spironolactone and states "I will reduce my intake of foods that contain potassium". d. A client who has states "I'll plan to take my calcium carbonate with a full glass of water".

b

A nurse is providing discharge teaching to a client who has a new prescription for phenelzine. The nurse should instruct the client that it is safe to eat which of the following foods while taking this medication? a. Avocados b. Whole grain bread c. Pepperoni pizza d. Smoked salmon

b

A nurse is providing teaching to a client who is on glucocorticoid therapy. Which of the following statements by the client indicates an understanding of the teaching? a. "I have my eyes examines annually" b. "I take a calcium vitamin supplement daily" c. "I limit my intake of foods with potassium" d. "I constantly take my medication between 8 and 9 each evening"

b

A nurse is providing teaching to an adolescent who has peptic ulcer disease. Which of the following statements by the client indicates an understanding of the teaching? a. "I will take sucralfate with meals three times per day" b. "I will avoid food and beverages that contain caffeine" c. "I will decrease my daily protein intake to 15 grams per day" d. "I will use ibuprofen as needed to control abdominal pain"

b

A nurse is receiving a telephone prescription from a provider for a client who requires additional medication for pain control. Which of the following entries should the nurse make in the medical record? a. "Morphine 3 mg SQ every 4 hr. PRN for pain." b. "Morphine 3 mg Subcutaneous" c. "Morphine 3.0 mg sub q every 4 hr. PRN for pain." d. "Morphine 3 mg SC q 4 hr. PRN for pain."

b

A nurse is receiving change-of-shift report for a group of clients. Which of the following clients should the nurse plan to assess first? A. A client who has sinus arrhythmia and is receiving monitoring B. A client who has a hip fracture and a new onset of tachypnea A client who has epidural analgesia and weakness in the lower extremities A client who has diabetes and a hemoglobin A1C of 6.8%

b

A nurse is reviewing the facility's safety protocols considering newborn abduction with the parent of a newborn. Which of the following statements indicates an understanding of the teaching? a. "Staff will apply identification band after first bath" b. "I will not publish public announcement about my baby's birth" c. "I can remove my baby's identification band as long as she is in my room" d. "I can leave my baby in my room while I walk in the hallway"

b

A nurse is reviewing the laboratory findings of a client who has diabetes mellitus and reports that she has been following her care. The nurse should identify which of the following findings indicates a need to revise the client's plan of care. a. Serum sodium 144 mEq/ b. Hba1c 10 % c. Random serum glucose 190 mg/dl.

b

A nurse is teaching a group of newly licensed nurses about client advocacy. Which of the following statements by a newly licensed nurse indicates an understanding of the teaching? a. "I feel to be in his best health care decision" b. "I will intervene if there is conflict between a client and his provider" c. "I should not advocate for a client unless he is able to ask me himself" d. "I will inform a client that his family should help make his health care decisions."

b

A nurse is working with one licensed practical nurse (PN), a student nurse and an unlicensed assistive personnel (UAP). Which newly admitted clients would be most appropriate to assign to the UAP? A) A 76-year-old client with severe depression B) A middle-aged client with an obsessive compulsive disorder C) A adolescent with dehydration and anorexia D) A young adult who is a heroin addict in withdrawal with hallucinations

b

A nurse manager is developing a protocol for an urgent care clinic that often cares for clients who do not speak the same language as clinical staff. Which of the following instructions should the nurse include? A. Use the client's children to provide interpretation. B. The nurse was going to do the interpretation C. Offer client's translation services for a nominal fee. D. Evaluate the clients' understanding at regular intervals

b

A nurse manager is planning to teach staff about critical pathways. Which of the following information should the nurse include? a. Critical pathways have unlimited timeframe for completion b. Decrease health care costs. c. Critical pathway if variances D. Used to create the critical pathway.

b

A nurse manager is updating protocols for the use of belt restraints. Which of the following guidelines should the nurse include? a. Remove the client's restraint every 4hr b. Document the client's condition every 15 min c. Attach the restrain to the bed's side rails d. Request a PRN restrain prescription for clients who are aggressive

b

A nurse receives a telephone call from a parent reporting that their school-age child has a nosebleed and that they cannot stop the bleeding. Which of the following instructions should the nurse provide to the provider? a. "Have your child lie down and turn their head to their side for 10 minutes" b. "Use your thumb and forefinger to apply pressure to the your child's nose" c. "Place a warm wet washcloth over your child's forehead and the bridge of their nose" d. "Tell your child to blow their nose gently and then sit down and tilt your head back"

b

A primigravida in the third trimester is hospitalized for preeclampsia. The nurse determines that the client's blood pressure is increasing. Which action should the nurse take first? a. Check the protein level in urine b. Have the client turn to the left side c. Take the temperature d. Monitor the urine output

b

A small round area appears under the client's skin as the administer an intradermal medication. What action should the nurse take? A) Elevate the area and apply light pressure over the site. B) Notify the healthcare provider of the allergic response. C) Document the site where the medication was given. D) Apply a cold pack to the area for twenty minutes

b

A young adult female presents at the emergency center with acute lower abdominal pain. Which assessment finding is most important for the nurse to report to the healthcare provider? A) Reports white, curly vaginal discharge. B) Last menstrual period 7 weeks ago. C) History of irritable bowel syndrome (IBS) D) Pain scale rating of a <9= on a 0-10 scale.

b

An 18 month-old child is on peritoneal dialysis in preparation for a renal transplant in the near future. When the nurse obtains the child's health history, the mother indicates that the child has not had the first measles, mumps, rubella (MMR) immunization. The nurse understands that which of the following is true in regards to giving immunizations to this child? A) Live vaccines are withheld in children with renal chronic illness B) The MMR vaccine should be given now, prior to the transplant C) An inactivated form of the vaccine can be given at any time D) The risk of vaccine side effects precludes giving the vaccine

b

An 86 year-old nursing home resident who has decreased mental status is hospitalized with pneumonic infiltrates in the right lower lobe. When the nurse assists the client with a clear liquid diet, the client begins to cough. What should the nurse do next? A) Add a thickening agent to the fluids B) Check the client's gag reflex C) Feed the client only solid foods D) Increase the rate of intravenous fluids

b

An adult male is brought to the emergency department by ambulance following a motorcycle accident. He was not wearing a helmet and presents with periorbital bruising and bloody drainage from both ears. Which assessment finding warrants immediate intervention by the nurse? A) Rebound abdominal tenderness. B) Nausea and projectile vomit. C) Rib pain with deep inspiration. D) Diminished bilateral breath sounds.

b

An elderly client who lives in a retirement community is admitted with these behaviors as reported by the daughter: absence in the daily senior group activity, missing the weekly card games, a change in calling the daughter from daily to once a week, and the client's tomato garden is overgrown with weeds. The nurse should assign this client to a room with which one of these clients? A) An adolescent who was admitted the day before with acute situational depression B) A middle aged person who has been on the unit for 72 hours with a dysthymia C) An elderly person who was admitted 3 hours ago with cycothymia D) A young adult who was admitted 24 hours ago for detoxification

b

An explosion has occurred at a high school for children with special needs and severe developmental delays. One of the students accompanied with a parent is seen at a community health center a day later. After the initial assessment the nurse concludes that the student appears to be in a crisis state. Which of these interventions based on crisis intervention principles is appropriate to do next? A) Help the student to identify a specific problem B) Ask the parent to identify the major problem C) Ask the student to think of different alternatives D) Examine with the parent a variety of options

b

At 40-week gestation, a laboring client who is lying is a supine position tells the nurse that she has finally found a comfortable position. What action should the nurse take? A) Place a pillow under the client's head and knees. B) Place a wedge under the client's right hip. C) Encourage the client to turn on her left side. D) Explain to the client that her position is not safe.

b

Discharge instructions for a client taking alprazolam (Xanax) should include which of the following? A) Sedative hypnotics are effective analgesics B) Sudden cessation of alprazolam (Xanax) can cause rebound insomnia and nightmares C) Caffeine beverages can increase the effect of sedative hypnotics D) Avoidance of excessive exercise and high temperature is recommended

b

Following morning care, a client with a C-5 spinal cord injury who is sitting in a wheelchair becomes flushed and complains of a headache. Which interventions should the nurse implement first? A) Assess the client's blood pressure every 15 minutes. B) Relieve any kinks or obstruction in the client's Foley tubing. C) Teach the client to response symptoms of dyreflexia. D) Administer a prescribed PRN dose of hydrazine (Apresoline.)

b

Four clients arrive on the labor and delivery unit at the same time. Which client should the nurse assess first? A) A 3-week multigravida with a prescription for serial blood pressures. B) A 39-week primigravida with biophysical profile score of 5 out of 8. C) A 38- week primigravida who reports contractions occurring every 10 minutes. D) A 41-week multigravida who is scheduled induction of labor today.

b

Several clients are admitted to an adult medical unit. The nurse would ensure airborne precautions for a client with which medical condition? A) Autoimmune deficiency syndrome (AIDS) with cytomegalovirus (CMV) B) A positive purified protein derivative with an abnormal chest x-ray C) A tentative diagnosis of viral pneumonia with productive brown sputum D) Advanced carcinoma of the lung with hemoptasis

b

The charge nurse is marking assignments on a psychiatric unit for a practical nurse (PN) and a newly licensed registered nurse (RN). Which client should be assigned to the RN? A) An older male who tells the staff and other clients that he is superman and can fly. B) A young male with schizophrenia who says voices are telling him to kill his psychiatrist C) A middle- aged client who is in the depressive phase of bipolar disease and is receiving lithium .D) An adult client who has been depressed for the past several months and denies social ideation.

b

The client with infective endocarditis must be assessed frequently by the home health nurse. Which finding suggests that antibiotic therapy is not effective, and must be reported by the nurse immediately to the healthcare provider? A) Nausea and vomiting B) Fever of 103 degrees Fahrenheit (39.5 degrees Celsius) C) Diffuse macular rash D) Muscle tenderness

b

The healthcare provider prescribed furosemide for a 4-year old child who has a ventricular septal defect. Which outcome indicates to the nurse that this pharmacological intervention was effective? A) Urine specific gravity change from 1.021 to 1.031 B) Daily weight decrease of 2 pounds (0.9 kg) C) Urinary output decrease of 5 ml/hour. D) Blood urea nitrogen (BUN) increase from 8 to 12 mg/dl (2.9 to 4.3)

b

The most effective nursing intervention to prevent atelectasis from developing in a post operative client is to A) Maintain adequate hydration B) Assist client to turn, deep breathe, and cough C) Ambulate client within 12 hours D) Splint incision

b

The mother of a toddler who is being treated for pesticide poisoning asks: "Why is activated charcoal used? What does it do?" What is the nurse's best response? A) "Activated charcoal decreases the systemic absorption of the poison from the stomach." B) "The charcoal absorbs the poison and forms a compound that doesn't hurt your child." C) "This substance helps to get the poison out of the body by the gastrointestinal system." D) "The action may bind or inactivate the toxins or irritants that are ingested by children or adults."

b

The nurse administers the osmotic diuretic mannitol to a client who has a closed head injury. Which assessment finding indicates immediate response to administration of the mannitol? A) A decrease in skin turgor. B) A decrease in intracranial pressure. C) An increase in serum sodium. D) An increase in serum osmolality values.

b

The nurse admits a 2 year-old child who has had a seizure. Which of the following statement by the child's parent would be important in determining the etiology of the seizure? A) "He has been taking long naps for a week." B) "He has had an ear infection for the past 2 days." C) "He has been eating more red meat lately." D) "He seems to be going to the bathroom more frequently."

b

The nurse assesses a 72 year-old client who was admitted for right sided congestive heart failure. Which of the following would the nurse anticipate finding? A) Decreased urinary output B) Jugular vein distention C) Pleural effusion D) Bibasilar crackles

b

The nurse enters room of a client with Parkinson's disease who is taking carbidopa levodopa. The client is arising slowly from the chair while the unlicensed assistive personnel (UAP) stands next to the chair. What action should the nurse take? A) Demonstrate how to help the client move more efficiently. B) Affirm that the client should arise slowly from the chair. C) Tell the UAP to assist the client in moving more quickly. D) Offer a PRN analgesic to reduce painful movement

b

The nurse enters the room of a client who is awaiting surgery for appendicitis. The unlicensed assistive personnel (UAP) has helped the client to a position of comfort with the right leg flexed and has applied a heating pad to the client's abdomen to relieve the client's pain. Which action should the nurse implement first? A) Determine if the consent form has been signed by the client. B) Remove the heating pad from the client's abdominal area. C) Confirm that the UAP has assisted the client to a position of comfort. D) Evaluate the effectiveness of the heating pad in relieving pain.

b

The nurse is caring for a 7 year-old with acute glomerulonephritis (AGN). Findings include moderate edema and oliguria. Serum blood urea nitrogen and creatinine are elevated. What dietary modifications are most appropriate? A) Decreased carbohydrates and fat B) Decreased sodium and potassium C) Increased potassium and protein D) Increased sodium and fluids

b

The nurse is caring for a child immediately after surgical correction of a ventricular septal defect. Which of the following nursing assessments should be a priority? A) Blanch nail beds for color and refill B) Assess for post operative arrhythmias C) Auscultate for pulmonary congestion D) Monitor equality of peripheral pulses

b

The nurse is caring for a client who requires a mechanical ventilator for breathing. The high pressure alarm goes off on the ventilator. What is the first action the nurse should perform? A) Disconnect the client from the ventilator and use a manual resuscitation bag B) Perform a quick assessment of the client's condition C) Call the respiratory therapist for help D) Press the alarm re-set button on the ventilator

b

The nurse is caring for a client with a suspected diagnosis of osteomyelitis. Which diagnostic test should the nurse prepare the client to expect the health care provider to prescribe? a. Radiographs b. Radionuclide bone scan c. C reactive protein tests d. Erythrocytes sedimentation rate

b

The nurse is measuring the output of an infant admitted for vomiting and diarrhea. During a 12- hour shift, the infant drinks 4 ounces of Pedialyte, vomits 25 ml, and voids twice. The dry diaper weight is 50 grams, and one wet diaper weight 75 grams, and the other weights 105 grams. Which computer documentation should the nurse enter in the infant's record? A) Document in the flow sheet that the infant voided times 2 and vomited 25 ml. B) Calculate differences in wet and dry diapers and document 80 ml urine output. C) Compare the difference between the infant's body weight and admission weight. D) Subtract vomits from 120 ml Pedialyte than document 95 ml oral intake

b

The nurse is planning care for a client who has a fourth-degree midline laceration that occurred during vaginal delivery of an 8 pound 10 ounce infant. Which intervention has the highest priority for this client? a. Administer prescribed PRN sleep medications b. Administer prescribed stool softener c. Encourage use of prescribed analgesic perennial sprays d. Encourage breastfeeding to promote uterine involution

b

The nurse is planning care for a young adult client with acromegaly. It is most important for the nurse to monitor which of the client's serum laboratory test results? A) White blood cell count. B) Glucose. C) Hemoglobin. D) Partial thromboplastin time

b

The nurse is preparing a 50 ml dose of 50% Dextrose IV for a client with insulin shock. How should the nurse administer the medication? A. Dilute the dextrose in one liter of 0.9% Normal Saline solution. B. Push undiluted slowly though the currently infusing IV. C. Mix the dextrose in a 50 ml piggyback for a total volume of 100 ml. D. Ask the pharmacist to add the Dextrose to a TPN solution.

b

The nurse is preparing a hepatitis teaching program. Which individual has the greatest need for teaching about prophylactic hepatitis B immunizations? A. A child daycare worker who has a history of type 2 diabetes mellitus B. An office worker who requires hemodialysis for chronic kidney disease (CKD) C. A restaurant chef who was diagnosed one year ago with hepatitis A D. A sales person who travels internationally and eats food in foreign countries.

b

The nurse is preparing to administer an oral antibiotic to a client with unilateral weakness, ptosis, mouth drooping, and aspiration pneumonia. What is the priority nursing assessment that should be done before administering this medication? A) Ask the client about soft food preferences. B) Determine which side of the body is weak. C) Obtain and record the client's vital signs. D) Auscultate the client's breath sounds.

b

The nurse is reviewing the medical record of a client who is requesting combination oral contraceptives. Which of the following conditions in the client's history is a contradiction to the use of oral contraceptives? a. Hyperthyroidism. b. Thrombophlebitis. c. Diverticulosis. d. Hypocalcemia.

b

The nurse is supervising an unlicensed assistive personnel (UAP) who will be providing personal care for a client with watery diarrhea caused by Clostridium difficile. Which action by the nurse takes priority? A) Remind the UAP to keep the client's water pitcher filled. B) Review use of personal protective equipment with the UAP. C) Provide barrier cream for application to the perineal area. D) Instruct the UAP to record the number of bowel movements.

b

The nurse is teaching about non steroidal anti-inflammatory drugs to a group of arthritic clients. To minimize the side effects, the nurse should emphasize which of the following actions? A) Reporting joint stiffness in the morning B) Taking the medication 1 hour before or 2 hours after meals C) Using alcohol in moderation unless driving D) Continuing to take aspirin for short term relief

b

The nurse is to administer a new medication to a client. Which actions are in the best interest of the client? Verify the order for the medication. Prior to giving the medication the nurse should say A) "Please state your name?" Upon entering the room the nurse should ask: B) "What is your name? What allergies do you have?" then check the client's name band and allergy band As the room is entered say C) "What is your name?" then check the client's name band Verify the client's allergies on the admission sheet and order. D) "Verify the client's name on the name plate outside the room then as the nurse enters the room ask the client "What is your first, middle and last name?"

b

The nurse on a pediatric unit of a healthcare facility observes a colleague leaving and open client electronic health record unattended while taking a lunch break. Which action should the nurse take? a. Close the computer and complete the day's assignments b. Remind the colleague of information security principles c. Comment about the action on a staff discussion board d. Discuss the incident with the facilities risk manager

b

The unlicensed assistive personnel (UAP) reports a sudden increase in temperature to 101 degrees F for a post surgical client. The nurse checks on the client's condition and observes a cup of steaming coffee at the bedside. What instructions are appropriate to give to the UAP? A) Encourage oral fluids for the temperature elevation B) Check temperature 15 minutes after hot liquids are taken C) Ask the client to drink only cold water and juices D) Chart this temperature elevation on the flow sheet

b

What information should the nurse include in the discharge teaching plan of a client with low back pain who is taking cyclobenzaprine to control muscle spasms? A) Avoid using heat or ice to injured muscles while taking this medication. B) Use cold and allergy medications only as directed by a healthcare provider. C) Take the medication on an empty stomach. D) Discontinue all non steroidal anti-intiflammatory medications.

b

What nursing assessment of a paralyzed client would indicate the probable presence of a fecal impaction? A) Presence of blood in stools B) Oozing liquid stool C) Continuous rumbling flatulence D) Absence of bowel movements

b

When administering enteral feeding to a client via a jejunostomy tube, the nurse should administer the formula A) Every four to six hours B) Continuously C) In a bolus D) Every hour

b

When caring for a client who has acute respiratory distress syndrome (ARDS), the nurse elevates the head of the bed 30 degrees. What in the reason for this intervention ? A) To promote retraction of the intercostal accessory muscles. B) To reduce abdominal pressure on the diaphragm. C) To decrease pressure on the medullary center which stimulates breathing. D) To promote bronchodilation and effective airway clearance.

b

When caring for a client with a post right thoracotomy who has undergone an upper lobectomy, the nurse focuses on pain management to promote A) Relaxation and sleep B) Deep breathing and coughing C) Incisional healing D) Range of motion exercises

b

When implementing a disaster intervention plan, which intervention should the nurse implement first? A) Initiate the discharge of stable clients from hospital units. B) Identify a command center where activities are coordinated. C) Assess community safety needs impacted by the disaster. D) Instruct all essential off-duty personnel to report to the facility.

b

When should the nurse conduct an Allen's test? A) When pulmonary artery pressures are obtained. B) Just before arterial blood gasses are drawn peripherally. C) Prior to attempting a cardiac output calculation. D) To assess for presence of a deep vein thrombus in the leg.

b

Which of these clients with associated lab reports is a priority for the nurse to report to the public health department within the next 24 hours? A) An infant with a positive culture of stool for Shigella B) An elderly factory worker with a lab report that is positive for acid-fast bacillus smear C) A young adult commercial pilot with a positive histopathological examination from an induced sputum for Pneumocystis carinii D) A middle-aged nurse with a history of varicella-zoster virus and with crops of vesicles on an erythematous base that appear on the skin

b

Which of these observations made by the nurse during an excretory urogram indicate a complicaton? A) The client complains of a salty taste in the mouth when the dye is injected B) The client's entire body turns a bright red color C) The client states "I have a feeling of getting warm." D) The client gags and complains " I am getting sick."

b

Which statement made by a nurse about the goal of total quality management or continuous quality improvement in a health care setting is correct? A) "It is to observe reactive service and product problem solving." B) Improvement of the processes in a proactive, preventive mode is paramount. C) A chart audits to finds common errors in practice and outcomes associated with goals. D) A flow chart to organize daily tasks is critical to the initial stages

b

Which these findings would the nurse more closely associate with anemia in a 10 month-old infant? A) Hemoglobin level of 12 g/dI B) Pale mucosa of the eyelids and lips C) Hypoactivity D) A heart rate between 140 to 160

b

While assisting a client who recently had a hip replacement onto the bed pan, the nurse notices that there is a small amount of bloody drainage on the on the surgical dressing, the client's skin is warm to touch, and there is a strong odor from the urine. Which action should the nurse take? A) Remove dressing and assess surgical. B) Measure the client's oral temperature. C) Insert an indwelling urinary catheter. D) Obtain a urine sample from the bed pan

b

While caring for a client who was admitted with myocardial infarction (MI) 2 days ago, the nurse notes today's temperature is 101.1 degrees Fahrenheit (38.5 degrees Celsius). The appropriate nursing intervention is to A) Call the health care provider immediately B) Administer acetaminophen as ordered as this is normal at this time C) Send blood, urine and sputum for culture D) Increase the client's fluid intak

b

While receiving a male postoperative client's staples de nurse observe that the client's eyes are closed and his face and hands are clenched. The client states, "I just hate having staples removed". After acknowledgement the client's anxiety, what action should the nurse implement? A) Reassure the client that this is a simple nursing procedure. B) Attempt to distract the client with general conversation. C) Encourage the client to continue verbalize his anxiety. D) Explain the procedure in detail while removing the staples.

b

the nurse has given discharge instructions to parents of a child on phenytoin (Dilantin). Which of the following statements suggests that the teaching was effective? A) "We will call the health care provider if the child develops acne." B) "Our child should brush and floss carefully after every meal." C) "We will skip the next dose if vomiting or fever occur." D) "When our child is seizure-free for 6 months, we can stop the medication."

b

************When caring for a client with full thickness burns to both lower extremities, which assessment findings warrant immediate intervention? Select all that apply A. Sloughing tissue around wound edges B. Complaint of increased pain and pressure C. Change in the quality of the peripheral pulses D. Loss of sensation to the left lower extremity E. Weeping serosanguineous fluid from wounds

b c d

A nurse is caring for a client who has cancer and is being transferred to hospice care. The client's daughter tells the nurse, "I'm not sure what to say to my mom if she asks me about dying." which of the following responses by the nurse is appropriate? (SATA) A. Hospice will take good care of your mom, so I wouldn't worry about that. b. Let's talk about you mom's cancer and how things will progress from here. D. Tell her not to worry. She still has plenty of time left. e. You sound like you have questions about your mom dying. Lets talk about it.

b, e

The nurse observes an unlicensed assistive personal (UAP) begin to provide oral care to an unresponsive client who is at risk for aspiration as seen in the picture. What instruction should the nurse provide the UAP? (Select all that apply). A) Flex the client neck forward. B) Turn the clients head to the side. C) Remove the gloved finger from the mouth. D) Elevate the head of the bed to semi fowlers E) Apply lubricant to the toothed.

b,c,d

A client who is newly diagnosed with type 2 diabetes mellitus (DM) receives a prescription for metformin (Glucophage) 500 mg PO twice daily. What information should the nurse include in this client's teaching plan? (Select all that apply.) A) Take an additional dose for signs of hyperglycemia B) Recognize signs and symptoms of hypoglycemia C) Report persist polyuria to the healthcare provider D) Use sliding scale insulin for finger stick glucose elevation E) Take Glucophage with the morning and evening meal.

b,c,e

A nurse is administering diazepam, a benzodiazepine, 10 mg IV push PRN, as prescribed to a client with alcohol withdrawal symptoms. Which actions should the nurse implement when administering the medication? (Select all that apply) A) Protect medication from exposure to light B) Monitor for changes in level of consciousness C) Observe for onset of generalized bruising or bleeding D) Perform ongoing assessment of respiratory status E) Administer slowly over at least two minutes

b,d,e

The nurse is interacting with a female client who is diagnostic with postpartum depression. Which findings should the nurse document as an objective signs of depression? (Select all that apply) A) Expresses suicidal thoughts B) Avoid eyes contact C) Reports feeling sad D) Has a disheveled appearance E) Interacts with felt effect

b,d,e

*************A college student brings a dorm roommate to the campus clinic because the roommate has been talking to someone who is not present. The client tells the nurse that the voices are saying, "kill, kill." What question should the nurse ask the client next? a. "When did these voices begin?" b. "Have you taken any hallucinogens?" c. "Are you planning to obey the voices?" d. "Do you believe the voices are real?"

c

*************A male client on the psychiatric unit is making sexual advances towards a female nurse. Which action should this nurse implement first? A. Document as specifically as possible the client's behavior in the nurse's notes B. Discuss with the client why he is making sexual advances toward the nurse C. Tell the client in a matter-of-fact manner to stop the sexual advances D. Request an immediate team meeting to discuss the inappropriate behavior

c

*************the nurse is evaluating a tertiary prevention program for clients with cardiovascular disease implemented in a rural health clinic. Which outcome indicate the program is effective? a. At-risk clients received an increased number of routine health screenings. b. Clients reported having new confidence in making healthy food choices. c. Clients who incurred disease complications promptly received rehabilitation. d. Client relapse rate of 30% in a 5-year community-wide anti-smoking campaign.

c

************A client is admitted with acute pancreatitis. The client admits to drinking a pint of bourbon daily. The nurse medicates the client for pain and monitors vital signs every 2 hours. Which finding should the nurse report immediately to the healthcare provider? A. Anorexia and abdominal distention B. Abdominal pain and vomiting C. Confusion and tremors D. Yellowing and itching of skin

c

************A client with bacterial meningitis is receiving phenytoin. Which assessment finding indication to the nurse that the client is experiencing a therapeutic response to the phenytoin? a. Increased time of ambulation between periods of rest. b. Decrease in intracranial pressure and cerebral edema. c. Absence of seizure activity for the duration of treatment. d. Normal electroencephalogram after drug administration.

c

************The nurse is assessing a first day postpartum client. Which finding is most indicative of a postpartum infection? A. Blood pressure of 122/74 mmHg B. White blood count of 19,000mm3 C. Moderate amount of foul-smelling lochia D. Oral temperature of 100.2F

c

***********A 3-year-old boy with a congenital heart defect is brought to the clinic by his mother because he has a fever and an earache. During the assessment, the mother asks the nurse why her child is at the 5th percentile for weight and height for his age. Which response is best for the nurse to provide? A. "Does your child seem mentally slower than his peers also?" B. "Haven't you been feeding him according to recommended daily allowances for children?" C. "His smaller size is probably due to the heart disease" D. "You should not worry about the growth tables. They are only averages for children."

c

***********After several months of chronic fatigue, morning stiffness, and join pain, a young adult is diagnosed with rheumatoid arthritis, and the healthcare provider prescribes prednisone. Which education should the nurse provide the client with regard to taking prednisone? a. Take prednisone doses before meals on an empty stomach. b. Wear sunglasses when exposed to bright sunlight. c. If sequential doses are missed, notify the healthcare provider. d. Schedule a monthly laboratory visit for a complete blood count

c

***********The nurse is feeding an older adult who was admitted with aspiration pneumonia. The client is weak and begins coughing while attempting to drink through a straw. Which intervention should the nurse implement? A. Teach coughing and deep breathing exercises B. Assess the client's oral cavity for ulcerations C. Request thick nectar liquids for the client D. Monitor the client when using a straw for liquids

c

**********A client is admitted with the diagnosis of Wernicke's syndrome. Which assessment finding should the nurse use in planning the client's care? A. Depression B. Peripheral neuropathy C. Confusion D. Right lower abdominal pain

c

**********The nurse is caring for a client who is entering the second stage of labor. Which action should the nurse implement first? A. Prepare the client for spinal anesthesia B. Empty the client's bladder using a straight catheter C. Convey to the client that birth is imminent D. Prepare the coach to accompany the client to delivery

c

**********The nurse observes an unlicensed assistive personnel (UAP) applying an alcohol-based hand rub while leaving a client's room after taking vital signs. What action should the nurse take? A. Instruct the UAP to return to the client's room to perform handwashing B. Supervise the UAP in the next client's room to evaluate hand hygiene C. Remind the UAP to continue rubbing the hands together until they are dry D. Advice the UAP to wear gloves when obtaining vital signs for all clients

c

**********When caring for a client with a traumatic brain injury (TBI) who had a craniotomy for increased intracranial pressure (ICP), the nurse assesses the client using the Glasgow coma scale (GCS) every two hours. For the past 8 hours the client's GCS score has been 14. What goes this GCS finding indicate about this client? A) Rehabilitative prognosis is an expected full recovery. B) Insertion of an ICP monitoring device is necessary. C) Neurologically stable without indications of an increased ICP. D) Risk for irreversible cerebral damage related to increased ICP

c

**********When entering a client's room to administer an 0900 IV antibiotic, the nurse finds that the client is engaged in sexual activity with a visitor. Which action should the nurse implement? A. Tell the client to stop the inappropriate behavior B. Complete an unusual occurrence report C. Leave the room and close the door quietly D. Ignore the behavior and hang the IV antibiotic

c

*********A client who received hemodialysis yesterday is experiencing a blood pressure of 200/100 mmHg, heart rate 110 beats/minute, and respiratory rate 36 breaths/minute. The client is manifesting shortness of breath, bilateral 2+ pedal edema, and an oxygen saturation on room air of 89%. Which action should the nurse take first? a. Elevate the foot of the bed. b. Restrict the client's fluid. c. Begin supplemental oxygen. d. Prepare the client for hemodialysis

c

*********A client with syndrome of inappropriate antidiuretic hormone secretion (SIADH) is admitted with hyponatremia. Which intervention is most important for the nurse to include in the plan of care to protect the client from injury? a. Initiate seizure precautions. b. Assess neurological status every 8 hours. c. Limit oral water intake. d. Administer a hypertonic IV fluids as prescribed.

c

*********A new nurse preparing to irrigate an intravenous catheter is attaching a 24-gauge needle. Which action should the charge nurse implement? A. Suggest the nurse use a 20-gauge needle B. Instruct the nurse to remove the needle C. Direct the nurse to change the IV tubing D. Prompt the nurse to apply povidone to the site

c

*********The nurse observes a practical nurse (PN) pouring warm water over the perineal area of a female client who his frequent urinary incontinence while the client is positioned on a bedpan. What action the nurse take? A) Recommend a complete bath to cleanse the perineal area more fully. B) Instruct the PN that this technique promotes infection in elderly females. C) Evaluate the effectiveness of this measure to stimulate client voiding. D) Suggest contacting the healthcare provider for a prescription for Cather insertion

c

*********While providing a health history, a female client tells the clinic nurse that she frequently thinks about hurting herself. Which question is most important for the nurse to ask? a. "Do you often have feeling of sadness?" b. "Are you having problems concentrating?" c. "Have you though about taking your life?" d. "What problems are you facing right now?"

c

********While changing a client's postoperative dressing, the nurse observes purulent drainage at the site. Before reporting this finding to the healthcare provider, the nurse should note which of the client's laboratory values? A. Platelet count B. Serum sodium level C. Neutrophil count D. Hematocrit

c

*******While caring for a client's postoperative dressing the nurse observes purulent wound drainage. Previously, the wound was inflamed and tender but without daring. Which is the most important action for the nurse to take? A) Determines if the drainage has an unpleasant B) Cleanse the wound with a sterile saline solution. C) Request a culture and sensitivity of the wound. D) Monitor the client's white blood cell count (WBC)

c

24. A charge nurse on a medical-surgical unit is planning assignments for a licensed practical nurse (LPN) who has been sent from the (Unable to read) unit due to a staffing shortage. Which of the following client should the nurse delegate to the LPN? A. A client who has an Hgb of 6.3 g/dl and a prescription for packed RBCs. B. A client who sustained a concussion and has unequal pupils. C. A client who is postoperative following a bowel resection with an NG tube set to continuous suction. D. A client who fractured his femur yesterday and is experiencing shortness of breath.

c

25. A nurse is caring for a client who is dilated to 10 cm and pushing. Which of the following pain-management is a safe option for the client? A. Naloxone hydrochloride. B. Spinal anesthesia. C. Pudendal block. D. Butorphanol tartrate

c

A 20 year-old client has an infected leg wound from a motorcycle accident, and the client has returned home from the hospital. The client is to keep the affected leg elevated and is on contact precautions. The client wants to know if visitors can come. The appropriate response from the home health nurse is that: A) Visitors must wear a mask and a gown B) There are no special requirements for visitors of clients on contact precautions C) Visitors should wash their hands before and after touching the client D) Visitors should wear gloves if they touch the client

c

A 60 year-old male client had a hernia repair in an outpatient surgery clinic. He is awake and alert, but has not been able to void since he returned from surgery 6 hours ago. He received 1000 mL of IV fluid. Which action would be most likely to help him void? A) Have him drink several glasses of water B) Crede' the bladder from the bottom to the top C) Assist him to stand by the side of the bed to void D) Wait 2 hours and have him try to void again

c

A 65-year-old Catholic Hispanic-Latino client with prostate cancer adamantly refuses pain medication because the client believes that suffering is part of life. The client states "everyone's life is in God's hands." The next action for the nurse to take is to A) Report the situation to the health care provider B) Discuss the situation with the client's family C) Ask the client if talking with a priest would be desired D) Document the situation on the notes

c

A certified IV nurse is providing education about peripherally inserted catheters (PICC) to a newly licensed nurse. Which of the following statements by the newly licensed nurse indicated an understanding of the teaching? a. "Use a vein in the middle of the lower arm to insert a PICC." b. "Flush a PICC using a 3-milliliter syringe." c. "Informed consent is required prior to PICC placement." d. "Position the client's arm in adduction for PICC placement."

c

A charge nurse is teaching a group of newly licensed nurses about the correct use of restraints. Which of the following should the nurse include in the teaching? a. Placing a belt restraint on a school-age child who has seizures. b. Securing wrist restraints to the bed rails for an adolescent. c. Applying elbow immobilizers of an infant receiving cleft lip injury d. Keeping the side rails of a toddler's crib elevated.

c

A charge nurse is teaching new staff members about factors that increase a client's risk to become violent. Which of the following risk factors should the nurse include as the best predictor of future violence? a. Experiencing delusions b. Male gender c. Pervious violent behavior d. A history of being in prison

c

A child is admitted to the pediatric unit with a diagnosis of suspected meningococcal meningitis. Which admission orders should the nurse do first? A) Institute seizure precautions B) Monitor neurologic status every hour C) Place in respiratory/secretion precautions D) Cefotaxime IV 50 mg/kg/day divided q6h

c

A client asks the nurse to call the police and states: "I need to report that I am being abused by a nurse." The nurse should first A) Focus on reality orientation to place and person B) Assist with the report of the client's complaint to the police C) Obtain more details of the client's claim of abuse D) Document the statement on the client's chart with a report to the manager

c

A client has a history of chronic obstructive pulmonary disease (COPD). As the nurse enters the client's room, his oxygen is running at 6 liters per minute, his color is flushed and his respirations are 8 per minute. What should the nurse do first? A) Obtain a 12-lead EKG B) Place client in high Fowler's position C) Lower the oxygen rate D) Take baseline vital signs

c

A client has an indwelling catheter with continuous bladder irrigation after undergoing a transurethral resection of the prostate (TURP) 12 hours ago. Which finding at this time should be reported to the health care provider? A) Light, pink urine B) occasional suprapubic cramping C) minimal drainage into the urinary collection bag D) complaints of the feeling of pulling on the urinary catheter

c

A client has returned from a cardiac catheterization. Which one of the following assessments would indicate the client is experiencing a complication from the procedure? A) Increased blood pressure B) Increased heart rate C) Loss of pulse in the extremity D) Decreased urine output

c

A client is admitted to a medical unit with a diagnosis of gastritis and chronic heavy alcohol abuse. What should the nurse administer to prevent the development of Wernicke's syndrome? A) Atenolol. B) Famotidine. C) Thiamine. D) Lorazepam.

c

A client is admitted to the intensive care unit with diabetes insidious due to a pituitary gland tumor. Which potential complication should the nurse monitor closely? A) Ketonuria. B) Peripheral edema. C) Hypokalemia. D) Elevated blood pressure.

c

A client is caring for a client following a paracentesis. Which of the following findings should the nurse identify as an indication of a complication? a. Decreased hematocrit. b. Increased blood pressure. c. Tachycardia. d. Hypothermia.

c

A client is requesting information from a nurse about a nitrazine test. Which of the following statements should the nurse make? a. "Your bladder should be full prior to me performing this test b. "If this test is positive you will be required to have a non-stress test. c. "This test will determine if there is leaking amniotic fluid" d. "I will be taking a blood sample to test for changes in your hormones levels"

c

A client presents to the clinic with concerns regarding her left breast. Which assessment findings most important for the nurse to report to the healthcare provider? A) Multiple firm, round, freely moveable masses. B) A slight asymmetry of the breasts. C) A fixed nodular mass with dimpling of skin. D) Bloody discharge from the nipple.

c

A client who had a percutaneous coronary intervention (PCI) two weeks ago returns to the clinic for a follow up visit. The client has a postoperative ejection fraction of 30%. Today the client has links which are clear, +1 pedal edema, and a 5 pound weight gain. Which intervention should the nurse implement? A) Insert saline lock for IV diuretic therapy. B) Arrange transport for admission to the hospital. C) Assess compliance with routine prescriptions. D) Instruct the client to monitor daily caloric intake

c

A client who is to have antineoplastic chemotherapy tells the nurses of a fear of being sick all the time and wishes to try acupuncture. Which of these beliefs stated by the client would be incorrect about acupuncture? A) Some needles go as deep as 3 inches, depending on where they're placed in the body and what the treatment is for. The needles usually are left in for 15 to 30 minutes. B) In traditional Chinese medicine, imbalances in the basic energetic flow of life — known as qi or chi — are thought to cause illness. C) The flow of life is believed to flow through major pathways or nerve clusters in your body. D) By inserting extremely fine needles into some of the over 400 acupuncture points in various combinations it is believed that energy flow will rebalance to allow the body's natural healing mechanisms to take over.

c

A client with a traumatic brain injury becomes progressively less responsive to stimuli. The client has a <Do Not Resuscitate= prescription, and the nurse observes that the unlicensed assistive personnel (UAP) has stopped turning the client from side to side as previously scheduled. What action should the nurse take? A) Encourage the UAP to provide comfort care measures only. B) Assume total care of the client to monitor neuralgic function. C) Advise the UAP to resume positioning the client on schedule. D) Assign a practical nurse to assist the UAP in turning the client.

c

A client with bladder cancer had surgical placement of a ureteroileostomy (ileal conduit) yesterday. Which postoperative assessment finding should the nurse report to the healthcare provider immediately? A) Stomal output of 40 ml in last hour. B) Liquid brown drainage. C) Red edematous stomal appearance. D) Mucous strings floating in the drainage.

c

A client with heart failure has Lanoxin (digoxin) ordered. What would the nurse expect to find when evaluating for the therapeutic effectiveness of this drug? A) diaphoresis with decreased urinary output B) increased heart rate with increase respirations C) improved respiratory status and increased urinary output D) decreased chest pain and decreased blood pressure

c

A community health nurse is planning primary prevention activities to reduce the occurrence of abuse. Which of the following strategies should the nurse include in the plan? a. Instruct healthcare professionals to identify abusive situations b. Locate financial support to open a shelter for abuse survivors c. Teach parenting skills to families at risk for abuse d. Connect abuse survivors with legal counsel

c

A home health nurse is preparing for an initial visit with an older adult client who lives alone. Which of the following actions should the nurse take first? a. Educate the client about current medical diagnosis b. Refer the client to a meal delivery program c. Identify environmental hazards in the home d. Arrange for client transportation to follow-up appointments

c

A male client in the final stages of terminal cancer tells his nurse that he wishes he could just be allowed to die. The client states that he is tired of fighting his illness and is only continuing treatments because his family wants him to live. Which action should the nurse take? a. Notify the family that treatments have been discontinued. b. Arrange a meeting with the family, physician, and client. c. Ask the chaplain to discuss death issues with the client. d. Request a consultation with the hospital social worker.

c

A male client is returned to the surgical unit following a left nephrectomy and is medicated with morphine sulfate 4 mg IV. His dressing has a small amount of bloody drainage, and a Jackson-Pratt bulb surgical drainage device is in place. Which intervention is most important for the nurse to include in this client's plan of care? A) Assess for back muscle aches. B) Obtain body weight daily. C) Monitor urinary output hourly. D) Record drainage from drain.

c

A middle-aged man in the outpatient clinic receives a prescription for tetracycline due to folliculitis of the scalp. Which instruction should the clinic nurse provide? a. Keep the infected area covered until the infection is resolved b. Use a fine-tooth comb to remove any knits observed on the scalp c. Take your medication with a glass of water two hours after meals d. Wash your bed linens and hot water after starting the medicatio

c

A nurse and an assistive personnel (AP) are caring for a group of clients. Which of the following tasks is appropriate for the nurse to delegate to the AP? a. Documenting the report of pain for a client who is postoperative b. Administering oral fluids to a client who has dysphagia c. Applying a condom catheter for a client who has a spinal cord injury. d. Reviewing active range-of-motion exercise with a client who had a stroke

c

A nurse in a mental health unit is planning room assignments for four clients. Which of the following client should be closest to the nurse's station? a. A client who has an anxiety disorder and is experiencing moderate anxiety. b. A client who has somatic symptom disorder and reports chronic pain. c. A client who has depressive disorder and reports feeling hopeless. d. A client who has bipolar disorder and impaired social interactions.

c

A nurse is assessing a client in the emergency department. Which of the following actions should the nurse take first? Exhibit 1 Laboratory Results Cerebrospinal fluid WBC 2,000/mm3 Neutrophils 88% Protein 320 mg/dl Glucose 35 mg/dl Cloudy in appearance Exhibit 2 History and Physical Reports severe headache and photophobia. Disoriented to person, place, and time. Lethargic. Exhibit 3 Vital Signs BP 166/96 mm Hg Respiratory rate 24/min Pulse rate 112/min Temperature 39.3C (102.8F) Pain of 6 on a scale from 0 to 10 Glasgow score 9 a. Place the client on a cooling blanket. b. Administer an analgesic. c. Obtain arterial blood gas levels. d. Elevate the head of the client's bed 30 degrees.

c

A nurse is assessing a client who is prescribed spironolactone. Which of the following laboratory values should the nurse monitor for this client? a. Total bilirubin b. Urine ketones c. Serum potassium d. Platelet count

c

A nurse is caring for a 1-day-old newborns who has jaundice and is receiving phototherapy. Which of the following actions should the nurse take? a. The infant 30 ml (1 oz) glucose water every 2 hr. b. Keep the infants head covered with a cap. c. Ensure that the newborn wears a diaper. d. Apply lotion to the newborn every 4 hr.

c

A nurse is caring for a client who has chronic pancreatitis. Which of the following dietary recommendations should the nurse make? a. Coffee with creamer. b. Lettuce with sliced avocados. c. Broiled skinless chicken breast with brown rice. d. Warm toast with margarine.

c

A nurse is caring for a client who has major depressive disorder and a new prescription for amitriptyline. The nurse should monitor for which of the following adverse effects? a. Increased salivation b. Weight loss c. Urinary retention d. Hypertension

c

A nurse is caring for a client who is postpartum and request information about contraception. Which of the following instructions should the nurse include? a. "The lactation amenorrhea method is effective for your first year postpartum" b. "You can continue to use the diaphragm used before your pregnancy" c. "Place transdermal birth control patch on your upper arm" d. "I should avoid vaginal spermicides while breast feeding."

c

A nurse is caring for a client who is receiving intermittent feedings via a feeding via a feeding pump and is experiencing dumping syndrome. Which of the following actions should the nurse take? a. Administer a refrigerated feeding. b. Increased the amount of water use to flush the tubing. c. Rate of the client's feedings. d. Instruct the client to move onto their right side.

c

A nurse is caring for a client who is receiving phenytoin for management of grand mal seizures and has a new prescription for isoniazid and rifampin. Which of the following should the nurse conclude if the client develops ataxia and incoordination? a. The client is experiencing an adverse reaction to rifampin b. The client's seizure disorder is no longer under control c. The client is showing evidence of phenytoin toxicity d. The client is having adverse effects due to combination antimicrobial therapy

c

A nurse is caring for a client who reports xerostomia following radiation therapy to the mandible. Which of the following is an appropriate action by the nurse? a. Offer the client saltine crackers between meals b. Suggest rinsing his mouth with an alcohol-based mouthwash c. Provide humidification of the room air d. Instruct the client on the use of esophageal speech

c

A nurse is caring for a client who states he recently purchased lavender oil to use when he gets the flu. The nurse should recognize which of the following findings as a potential contraindication for using lavender? a. The client takes vitamin C daily b. The client has a history of alcohol use disorder c. The client has a history of asthma d. The client takes furosemide twice daily

c

A nurse is performing CPR on an adult who went into cardiopulmonary arrest. Another nurse enters the room in response to the call. After checking the client's pulse and respirations, what should be the function of the second nurse? A) Relieve the nurse performing CPR B) Go get the code cart C) Participate with the compressions or breathing D) Validate the client's advanced directive

c

A nurse is performing physical therapy for a client who has Parkinson's disease. Which of the following statements by the client indicates the need for a referral to physical therapy? a. "I have been experiencing more tremors in my left arm than before" b. "I noticed that I am having a harder time holding on to my toothbrush" c. " Lately, I feel like my feet are freezing up, as they are stuck to the ground" d. "Sometimes, I feel I am making a chewing motion when I'm not eating"

c

A nurse is planning an educational program for high school students about cigarette smoking. Which of the following potential consequences of smoking is most likely to discourage adolescents from using tobacco? a. Use of tobacco might lead to alcohol and drug abuse b. Smoking in adolescence increases the risk of developing lung cancer later in life c. Use of tobacco decreases the level of athletic ability d. Smoking in adolescence increases the risk of lifelong addiction

c

A nurse is planning care for a client who has bipolar disorder and is experiencing mania. Which of the following interventions should the nurse include in the plan? a. Encourage the client to spend time in the day room b. Withdraw the client's TV privileges is the does not attend group therapy c. Encourage the client to take frequent rest periods d. Place the cline in seclusion when he exhibits signs of anxiety

c

A nurse is planning care for a client who has thrombocytopenia. Which of the following actions should the nurse include? a. Encourage the client to floss daily. b. Remove fresh flowers from the client's room. c. Provide the client with a stool softener. d. Avoid serving the client raw vegetable.

c

A nurse is planning care for a client who is receiving hemodialysis. Which of the following actions should the nurse include in the plan of care? A. Withhold all medications until after dialysis. B. Rehydrate with dextrose 5% in water for orthostatic hypotension. C. Check the vascular access site for bleeding after dialysis. D. Give an antibiotic 30 min before dialysis

c

A nurse is planning for a client who practices Orthodox Judaism. The client tells the nurse that ...... Passover holiday. Which of the following action should the nurse include in the plan of care? A. Provide chicken with cream sauce. B. Avoid serving fish with fins and scales. C. Provide unleavened bread. D. Avoid serving foods containing lamb

c

A nurse is preparing an in-service for a group of nurses about malpractice issues in nursing. Which of the following examples should the nurse include in the teaching? a. Leaving a nasogastric tube clamped after administering oral medication b. Documenting communication with a provider in the progress notes of the client's medical records c. Administering potassium via IV bolus d. Placing a yellow bracelet on a client who is at risk for falls

c

A nurse is preparing to assess a 2-week-old newborn. Which of the following actions should the nurse plan to take? A. Obtain the newborn's body temperature using a tympanic thermometer. B. FACES pain scale. C. Auscultate the newborn's apical pulse for 60 seconds. D. Measure the newborn's head circumference over the eyebrows and below the occipital prominence.

c

A nurse is providing care to a 63 year-old client with pneumonia. Which intervention promotes the client's comfort? A) Increase oral fluid intake B) Encourage visits from family and friends C) Keep conversations short D) Monitor vital signs frequently

c

A nurse is providing discharge teaching to a client who has chronic kidney disease and is receiving hemodialysis. Which of the following instructions should the nurse include in the teaching? a. Take magnesium hydroxide for indigestion b. Drink at least 3L of fluid daily c. Eat 1g/kg of protein per day d. Consume foods high in potassium

c

A nurse is reviewing the medical records of four clients. The nurse should identify that which of the following client findings follow up care? a. A client who is taking bumetanide and has potassium level of 3.6 mEq/L b. A client who is scheduled for colonoscopy and taking sodium phosphate c. A client who received a Mantoux test 48 hrs ago and has induration d. A client who is taking warfarin and has INR of 1.8

c

A nurse is teaching a group of newly licensed nurses about measures to take when caring for a client who is on contact precautions. Which of the following should the nurse include in the teaching? a. Remove the protective gown after the client's room. b. Place the client in a room with negative pressure. c. Wear gloves when providing care to the client.

c

A nurse is teaching a newly licensed nurse about ergonomic principles. Which of the following actions by a newly licensed nurse indicates an understanding of the teaching? a. Stands with feet together when lifting a client up in bed. b. Raises the client's head of bed before pulling the cline up. c. Uses a mechanical lift to move client from bed to chair. d. Places a gait belt around the client's upper chest before assisting a client to stand.

c

A nurse is teaching who has chronic pain about avoiding constipation from opioid medications. Which of the following should the nurse include in the teaching? a. Drink 1.5L fluids each day. b. Take mineral oil at bedtime. c. Increase exercise activity d. Decrease insoluble fiber.

c

A nurse working in a rehabilitation facility is developing a discharge plan for a client who has left-sided hemiplegia the following actions is the nurse's priority? a. Consult with a case manager about insurance coverage. b. Counsel caregivers about respite care options. c. Ensure that the client has a referral for physical therapy. d. Refer the client to a local stroke support group.

c

A nurses is assessing a preschooler who has recently experienced an unexpected death in the family. Which of the following should the nurse recognize as an expected finding? a. The child expresses curiosity about the death process. b. The child refuses to talk about death. c. The child believes the person will return. d. The child focuses on his own mortality.

c

A teenage female is admitted with the diagnosis of anorexia nervosa. Upon admission, the nurse finds a bottle of assorted pills in the client's drawer. The client tells the nurse that they are antacids for stomach pains. The best response by the nurse would be A) "These pills aren't antacids since they are all different." B) "Some teenagers use pills to lose weight." C) "Tell me about your week prior to being admitted." D) "Are you taking pills to change your weight?"

c

A young client involved in a motorcycle collision experienced a laceration of the gastrocnemius muscle. Which instruction should the nurse provide to the practical nurse who is caring for this client? a. Avoid washing the limb when assisting with bathing b. Elevate limb above the heart when lying in bed c. Avoid planter flexion of the affected limb d. Perform range of motion on the affected limb

c

After an explosion at a factory one of the workers approaches the nurse and says "I am an unlicensed assistive personnel (UAP) at the local hospital." Which of these tasks should the nurse assign to this worker who wants to help during the care of the wounded workers? A) Get temperatures B) Take blood pressure C) Palpate pulses D) Check alertness

c

After working with a very demanding client, an unlicensed assistive personnel (UAP) tells the nurse, "I have had it with that client. I just can't do anything that pleases him. I'm not going in there again." The nurse should respond by saying A) "He has a lot of problems. You need to have patience with him." B) "I will talk with him and try to figure out what to do." C) "He is scared and taking it out on you. Let's talk to figure out what to do." D) "Ignore him and get the rest of your work done. Someone else can take care of him for the rest of the day."

c

An adult client with a broken femur is transferred to the medical surgical unit to await surgical internal fixation after the application of an external traction device to stabilizer the leg. An hour after an opioid analgesic was administered, the client reports muscle spasm and pain at the fracture site. While waiting for the client to be transported to surgery, which action the nurse implement? A) Reduce the weight on the traction device. B) Administer PRN dose of a muscle relaxant. C) Observe for signs of deep vein thrombosis. D) Check client's most recent electrolyte values

c

During the administration of albuterol per nebulizer, the client complains of shakiness. The client's vital signs are heart rate 120 beats/minutes, respirations 20 breaths/minute, blood pressure 140/88. What action should the nurse take? A) Administer an anxiolytic. B) Obtain 12 lead electrocardiogram. C) Educate client about the side effects of albuterol. D) Stop the albuterol administration and restart in 30 minutes

c

Five days after surgical fixation of a fractured femur, a client suddenly reports chest pain and difficulty in breathing..... had a pulmonary embolus. What action should the nurse take first? a. Bring the emergency crash cart to the bedside. b. Prepare a continuous heparin infusion per protocol. c. Provide supplemental oxygen. d. Notify the healthcare provider

c

Following a cardiac catherization, an adult client is sent to the cardiovascular unit. The nurse instructs the client to keep the affected leg immobile. Which intervention should the nurse plan to include in the client's plan of care? A) Apply a sequential compression device. B) Ambulate once vital signs stable. C) Monitor telemetry of dysrhythmias. D) Maintain NPO until bowel sounds return.

c

Following a gunshot wound, an adult client has a hemoglobin level 4 grams/dl (40 mmil/L SI). The nurse prepares to administer a unit of blood for an emergency transfusion. The client has AB negative blood type and the blood bank sends a unit of Type A Rh negative, reporting that there is no Type AB negative blood currently available. Which intervention should the nurse implement? A) Administer normal saline solution until Type AB negative is available. B) Obtain additional consent for administration of Type A negative blood. C) Transfuse Types A negative blood until Type AB negative is available. D) Recheck the client's hemoglobin, blood type, and Rh factor.

c

For the second time in four months, and overweight client is seen in the clinic because of vulvovaginitis resulting from a candida infection. Which intervention should the nurse implement first? a. Determine the client's typical menstrual cycle b. Obtain the client's blood glucose level c. Ask the client about recent sexual activity d. Review the client results for a complete blood count

c

In caring for a client who is receiving linezolid IV for nosocomial pneumonia, which assessment finding is most important for the nurse to report to the health care provider? a. Nausea and headache b. Yellow tinged sputum c. Watery diarrhea d. Increased fatigue

c

Several clients on a busy antepartum unit are scheduled for procedures that require informed consent. Which situation should the nurse explore further before witnessing the client's signature on the consent form? A) The client is illiterate but verbalizes understanding and consent for the procedure. B) A 15-year-old primigravida who has been self-supporting for the past 6 months. C) The obstetrician explained a procedure that a neurologist will perform. D) The client was mediated for pain with a narcotic IM 6 hours ago.

c

The charge nurse is planning assignments on a medical unit. Which client should be assigned to the PN? A) Test a stool specimen for occult blood B) Assist with the ambulation of a client with a chest tube C) Irrigate and redress a leg wound D) Admit a client from the emergency room

c

The mother of a child with cerebral palsy (CP) asks the nurse if her child's impaired movements will worsen as the child grows. Which response provides the best explanation? A) Continued development of the brain lesion determines the child's outcome. B) Severe motor dysfunction determines the extent of successful habilitation. C) Brain damage with CP is not progressive but does have a variable course. D) CP is one of the most common permanent physical disability in children.

c

The nurse auscultates a client's abdomen and hears a loud bruit near the umbilicus. What is the nurse's best action based on this assessment finding? A) Document the assessment finding in the medical record. B) Palpate the abdomen lightly in all four quadrants. C) Report the finding to the health care provider. D) Place the client in a semi-Fowler's position.

c

The nurse is admitting a client from the postanesthesia unit to the postoperative surgical care unit. Which prescription should the nurse implement first? a. Straight catheterization if unable to avoid b. Advance from clear liquids as tolerated c. Cefazolin 1 gram IVPB q6 hours d. Complete blood count cell count (CBC) in AM

c

The nurse is assessing a client 2 hours postoperatively after a femoral popliteal bypass. The upper leg dressing becomes saturated with blood. The nurse's first action should be to A) Wrap the leg with elastic bandages B) Apply pressure at the bleeding site C) Reinforce the dressing and elevate the leg D) Remove the dressings and re-dress the incision

c

The nurse is assessing a client who recently had an upper respiratory infection and now presents to the emergency department with lower extremity numbness and difficulty swallowing. Based on these finding, this client is at greatest risk for which pathophysiology condition? A) Epstein Bar Virus. B) Cytomegalovirus. C) Guillen Barre syndrome. D) Mycoplasma Pneumonia.

c

The nurse is caring for a 3 years old child who is two hours postoperative from a cardiac catherization via the right femoral artery. Which assessment finding is an indication of arterial obstruction? A. Blood pressure trend is downward, and pulse is rapid and irregular. B. The pressure dressing at right femoral area is moist and oozing blood. C. Right foot is cool to the touch and appears pale and blanched. D. Pulse distal to the femoral artery is weaker on left foot than right foot.

c

The nurse is caring for a client undergoing the placement of a central venous catheter line. Which of the following would require the nurse's immediate attention? A) Pallor B) Increased temperature C) Dyspnea D) Involuntary muscle spasms

c

The nurse is complaining an admission assessment for a male client with paranoid schizophrenia. The client tells the nurse that the staff dislikes him. What action should the nurse take? A) Assess the client's speech pattern for a flight of class. B) Observe the client for obsessive activities such as repeated hand washing. C) Determine if the client has formulated any plans regarding the staff. D) Ask the client if he has a plan to harm himself.

c

The nurse is developing a plan of care for an older male client with type 2 diabetes who reports blurred vision. Which outcome shows a plan of care for this client? a. The client will express acceptance of his changing health status. b. The client's family will state signs and symptoms about the disease. c. The nurse will demonstrate the procedure for accurate eye care. d. The client's daily blood pressure will be less than 140/80 mmHg this month.

c

The nurse is discussing with a group of students the disease Kawasaki. What statement made by a student about Kawasaki disease is incorrect? A) It also called mucocutaneous lymph node syndrome because it affects the mucous membranes (inside the mouth, throat and nose), skin and lymph nodes. B) In the second phase of the disease, findings include peeling of the skin on the hands and feet with joint and abdominal pain C) Kawasaki disease occurs most often in boys, children younger than age 5 and children of Hispanic descent D) Initially findings are a sudden high fever, usually above 104 degrees Fahrenheit, which lasts 1 to2 weeks

c

The nurse is performing a physical assessment on a client who just had an endotracheal tube inserted. Which finding would call for immediate action by the nurse? A) Breath sounds can be heard bilaterally B) Mist is visible in the T-Piece C) Pulse oximetry of 88 D) Client is unable to speak

c

The nurse is planning an educational session for new parents on ways to prevent sudden infant death syndrome. Which information is most important to provide parents of newborns and infants? a. Do not pop bottles for an infant during naps and bedtime b. Remove pillows and soft toys from the crib at bedtime c. Position the intent in a supine position while sleeping d. Keep a bulb syringe accessible for use for an infant

c

The nurse is planning care for a client with a CVA. Which of the following measures planned by the nurse would be most effective in preventing skin breakdown? A) Place client in the wheelchair for four hours each day B) Pad the bony prominence C) Reposition every two hours D) Massage reddened bony prominence

c

The nurse is planning discharge teaching for a client who had an evacuation of gestational trophoblastic disease (GTD) two days age. Which information is most important for the nurse to include in this client's teaching plan? A) Location and times for a local support group. B) Rho(D) immune globulin to prevent isoiminuization. C) Schedule follow up visit with the healthcare provider. D) Oral contraceptive use for at least one year.

c

The nurse is providing teaching about folic acid to a client who is prima gravida. Which of the following information should the nurse include in the teaching? a. "You should take folic acid to decrease the risk of transmitting infections to your baby" b. "You should consume a maximum of 300 micrograms of folic acid every day". c. "You can increase your dietary intake of folic acid by eating cereals and citrus fruits". d. "You can expect your urine to appear red-tingled while taking folic acid supplements".

c

The nurse is teaching a group of clients at a community health fair about genetic disease. Which of the following statements by a client indicates an understanding of the teaching? a. "If there is a genetic risk for future pregnancies, we can get treatment now to prevent the disease" b. "There is no need to have genetic counseling if I know that I have a family history of mental illness." c. "My family has genetic risk for breast cancer, so I am considering a total mastectomy" d. "Even if I have a genetic risk for a disease the chance I will get the disease is probably low due to current medical treatments."

c

The nurse is teaching a group of women about osteoporosis and exercise. The nurse should emphasize the need for which type of regular activity? A. Core strengthening B. Aerobic exercise C. Weight-bearing exercise D. Muscle stretching and toning

c

The nurse is teaching an 87 year-old client methods for maintaining regular bowel movements. The nurse would caution the client to AVOID A) Glycerine suppositories B) Fiber supplements C) Laxatives D) Stool softeners

c

The nurse provides teaching about a scheduled procedure to a male client who was admitted for diagnostic testing to determine the extent of metastasis of his cancer. An hour later the client asked the nurse for information about the scheduled procedure. What action should the nurse implement? a. Reassure the client that whatever the outcome, he will be able to cope with the results b. Encourage the client to take deep breaths in to avoid thinking negative thoughts c. Repeat the client teaching and leave written instructions for the client d. Remind the client of the instructions that were provided an hour ago

c

The nurse receives a report on an older adult client with middle stage dementia. What information suggests the nurse should do immediate follow up rather than delegatecare to the nursing assistant? The client A) Has had a change in respiratory rate by an increase of 2 breaths B) Has had a change in heart rate by an increase of 10 beats C) Was minimally responsive to voice and touch D) Has had a blood pressure change by a drop in 8 mmHg systolic

c

The parents bring their one-year-old child with a ventricular septal defect to the clinic for a well child visit. Which assessment finding should the nurse report to the health care provider immediately? a. Respirations of 26 breaths/minute at rest b. Expected weight and growth care for an infant c. 2+ pitting edema in the extremities d. Heart rate of 105 beats/minute

c

The school nurse is teaching the faculty the most effective methods to prevent the spread of lice in the school. The information that would be most important to include would be which of these statements? A) "The treatment requires reapplication in 8 to 10 days." B) "Bedding and clothing can be boiled or steamed." C) Children are not to share hats, scarves and combs. D) Nit combs are necessary to comb out nits.

c

When administering ceftriaxone sodium intravenously to a client before surgery, which assessment finding requires the most immediate intervention by the nurse. A) Headache. B) Pruritis. C) Stridor. D) Nausea

c

When attempting to establish risk reduction strategies in a community, the nurse note that regional studies indicates a high number of persons with growth stunting and irreversible mental deficiencies caused by hypothyroidism (cretinism). The nurse should seek funding to implement which screening measure? A) TSH levels in women over 45. B) T3 levels in school-aged children. C) T4 levels in newborn. D) Iodine levels in all persons over 60

c

When teaching a client about the side effects of fluoxetine (Prozac), which of the following will be included? A) Tachycardia blurred vision, hypotension, anorexia B) Orthostatic hypotension, vertigo, reactions to tyramine rich foods C) Diarrhea, dry mouth, weight loss, reduced libido D) Photosensitivity, seizures, edema, hyperglycemia

c

Which statement made by a client to the admitting nurse suggests that the client is experiencing a manic episode? A) "I think all children should have their heads shaved." B) "I have been restricted in thought and harmed." C) "I have powers to get you whatever you wish, no matter the cost." D) "I think all of my contacts last week have attempted to poison me."

c

While providing home care to a client with congestive heart failure, the nurse is asked how long diuretics must be taken. What is the nurse's best response? A) "As you urinate more, you will need less medication to control fluid." B) "You will have to take this medication for about a year." C) "The medication must be continued so the fluid problem is controlled." D) "Please talk to your health care provider about medications and treatments."

c

While teaching a young male adult to use an inhaler for his newly diagnosed asthma, the client stares into the distance and appears to be concentrating on something other than the lesson the nurse is presenting. What action should the nurse take? A) Remind the client that a rescue inhaler might save his life. B) Gently touch the client then continue with the teaching. C) Ask the client what he is thinking about at this time. D) Leave the client alone so that he can grieve his illness.

c

the nurse is caring for a client in atrial fibrillation. The atrial heart rate is 250 and the ventricular rate is controlled at 75. Which of the following findings is cause for the most concern? a. Diminished bowel sounds b. Loss of appetite c. A cold, pale lower leg d. Tachypnea

c

********After an unsuccessful resuscitation attempt, the nurse calls the family of the deceased. The family wish to see the body before it is taken to the funeral home. Which interventions should the nurse take to prepare the body before the family enters the room? (Select all that apply) A) Take out dentures and place in a labeled cup B) Apply a body shroud C) Place a small pillow under the head D) Remove resuscitation equipment from the room E) Gently close the eyes

c d e

A charge nurse on a medical-surgical unit is assisting with the emergency response plan following an external disaster in the community. In anticipation of multiple client admissions, which of the following current clients should the nurse recommend for early discharge? (SATA) a. A client who has COPD and a respiratory rate of 44/min b. A client who has cancer with a sealed implant for radiation treatment. c. A client who is 1 day postoperative following a vertebroplasty d. A client who is receiving heparin for deep vein thrombosis.

c, d

A group of nurses implemented a pilot study to evaluate a proposed evidence-based change to providing client care. Evaluation indicates successful outcomes, and the nurses want to integrate the change throughout the facility. Which action should be taken? Select all that apply. a. Arrange in service training through the education department b. Obtain informed consent from clients who will receive care c. Submit a Sentinel event report to the research committee d. Invite data review by the quality improvement department e. Propose clinical practice guidelines to the nursing committee

c,d,e

***************An older male was recently admitted to the rehabilitation unit with unilateral neglect syndrome as the result of a cerebrovascular accident (CVA). Which action should the nurse include in the plan of care? A. Use hand and arm gestures to improve communication and comprehension B. Provide additional light in the room to promote sensory stimulation C. Place a clock and calendar in the room to improve orientation D. Teach the client to turn his head from side to side for visual scanning

d

*************During shift report, the charge nurse receives notice of several problems. Which problem should the nurse address first? a. The census report has not been completed. b. A client's wife has asked to speak with the charge nurse. c. One staff member has not reported to work. d. A bucket of water was spilled in the hallway.

d

*************To prevent medication errors by an older client who is sometimes confused, which intervention by the home health nurse is likely to be most effective? A. Have an alert family member administer medications B. Encourage taking medications at the same times daily C. Instruct the client to wear glasses when reading labels D. Provide education both verbally and in written format

d

************A male client who fell of a roof has right and left femur fractures and crushing injuries to both ankles. he is supine with bilateral skin traction applied to the lower extremities while awaiting surgery within the next 4 hours. When asked to evaluate his pain on a scale of 1 to 10, he screams that it is 20. For the last 4 hours, he has received morphine 2mg IV hourly. His vial signs are heart rate 130 beats/minute, respirations 32 breaths/minute, blood pressure 180/90 mmHg. Which intervention is most important for the nurse to implement? A. Request the healthcare provider to consider a different analgesic B. Evaluate the traction for amount of tension applied to each extremity C. Determine if client is experiencing cumulative effects of the total dosage D. Assess the extremities for signs of compartment syndrome q2 hours

d

************An 11-year-old client is admitted to the mental health unit after trying to run away from home and threatening self-harm. The nurse establishes a goal to promote effective coping and plans to ask the client to verbalize three ways to deal with stress. Which activity is best to establish rapport and accomplish this therapeutic goal? A. Bring the client to the team meeting to discuss the treatment plan B. Explain the purpose of each medication the client is currently taking C. Ask the client to write feelings in a journal and then review it together D. Play a board game with the client and begin talking about stressors

d

************An older adult client with chronic emphysema is admitted to the emergency room from home with acute onset of weakness, palpitations, and vomiting. Which information is most important for the nurse to obtain during the initial interview? a. History of smoking over the past 6 months. b. Sleep patterns during the previous few week. c. Activity level prior to onset of symptoms. d. Recent compliance with prescribed medications.

d

************An older client is admitted with fluid volume deficit and dehydration. Which assessment finding is the best indicator of hydration that the nurse should report to the healthcare provider? a. Urine specific gravity is 1.040 b. Systolic blood pressure decreases 10 points when standing. c. The client denies being thirsty. d. Skin tenting occurs when the client's forearm is pinched.

d

************The nurse is caring for four clients. Client A, who has emphysema and whose oxygen saturation is 94%; Client B, with a postoperative hemoglobin of 8.2 mg/dL; Client C, newly admitted with a potassium level of 3.8 mEq/L; and Client D, scheduled for an appendectomy who has a white blood cell count of 14,000 mm3. Which intervention should the nurse implement? A. Move Client D into an isolation room 24 hours before surgery B. Increase Client A's oxygen to 4 liters a minute per cannula C. Ask the dietician to add a banana to Client C's breakfast tray D. Verify that Client B has two units of packed cells available

d

************The nurse is developing a plan of care for a client who reports tingling of the feet and who is newly diagnosed with peripheral vascular disease. Which outcome should the nurse include in the plan of care for this client? a. The client will express acceptance of their newly diagnosed health status. b. The nurse will encourage the client to walk thirty minutes everyday. c. The client's blood pressure readings will be less than 160/90 mmHg. d. The client's skin on the lower legs will be intact at the next clinical visit.

d

***********A male client with stomach cancer returns to the unit following a total gastrectomy. He has a nasogastric tube to suction and is receiving Lactated Ringer's solution at 75 mL/hour IV. One hour after admission to the unit, the nurse notes 300 mL of blood in the suction canister, the client's heart rate is 155 beats/minute, and his blood pressure is 78/48 mmHg. In addition to reporting the finding to the surgeon. Which action should the nurse implement first? a. Measure and document the client's urinary output. b. Request the client's reserved unit if packed red blood cells. c. Prepare the placement of a central venous catheter. d. Increase the infusion rate of Lactated Ringer's solution.

d

***********A new mother on the postpartum unit runs out of the room screaming that her newborn infant's crib is empty and the baby is missing. What action should the nurse take first? a. Determine if the newborn is in the nursery. b. Activate the lockdown procedure. c. Ask the mother if any visitors were expected to arrive. d. Match ID bands of all infants and mothers on the unit.

d

***********The charge nurse of a critical care unit is informed at the beginning of the shift that less than the optimal number of registered nurses will be working that shift. In planning assignments, which client should receive the most care hours by a registered nurse (RN)? A. A 48-year-old marathon runner with a central venous catheter who is experiencing nausea and vomiting due to electrolyte disturbance following a race B. A 34-year-old admitted today after an emergency appendectomy who has a peripheral intravenous catheter and a Foley catheter C. A 63-year-old chain smoker admitted with chronic bronchitis who is receiving oxygen via nasal cannula and has a saline-locked peripheral intravenous catheter D. An 82-year-old client with Alzheimer's disease and a newly-fractured femur who has a Foley catheter and soft wrist restraints applied

d

***********The nurse is preparing to send a client to the cardiac catheterization lab for an angioplasty. Which client report is most important for them to explore further prior to the start of the procedure? a. Drank a glass of water in the past 2 hours. b. Reports left chest wall pain prior to admission. c. Verbalize a fear of being in a confined space. d. Experience facial swelling after eating crab

d

***********The nurse is working on an infectious disease unit. Which client should be assigned to a room with negative airflow, while requiring personnel to use a particulate respirator mask and requiring staff to observe airborne, as well as standard precautions? A. A female adolescent admitted with multiple genital herpes simplex II lesions B. An older client with scabies who is admitted from an extended care facility C. Twin siblings admitted with scarlet fever that is complicated with pneumonia D. A client with a positive Mantoux and sputum cultures results positive for AFB

d

**********The healthcare provider prescribes methylergonovine maleate for a postpartum client with uterine atony. What finding should indicate to the nurse to withhold the next dose of the medication? A. Difficulty locating the uterine fundus B. Excessive lochia C. Saturation of more than one pad per hour D. Hypertension

d

**********The nurse is caring for a client with chronic obstructive pulmonary disease (COPD) who uses oxygen at 2 L/minute per nasal cannula continuously. The nurse observes that the client is having increased shortness of breath with respirations at 23 breaths/minute. Which action should the nurse implement first? a. Determine if the client is experiencing any anxiety. b. Auscultate the client's bilateral lung sounds and oxygen saturation. c. Notify the healthcare provider about the client's distress. d. Assess the delivery mechanism of the oxygen tank, tubing, and cannula.

d

*********A young adult woman visits the clinic and learns that she is positive for BRCA1 gene mutation and asks the nurse what to expect next. How should the nurse respond? A) Provide information about survival rates women who have this genetic mutation. B) Gather additional information about the client's family history for all types of cancer. C) Offer assurance that there are a variety of effective treatments for breast cancer. D) Explain that counseling will be provided to give her information about her cancer risk

d

*********The nurse is preparing to administer an IV dose of ciprofloxacin to a client with a urinary tract infection. Which client data requires the most immediate intervention by the nurse? A) White blood cell count of 12,000 mm^3 (12 x 10^9/L SI) B) Serum sodium of 145 men/L (145 mm/L SI) C) Urine culture positive for MRSA. D) Serum creatinine of 4.5mg/dl (398 mom/L SI

d

A 14 year-old with a history of sickle cell disease is admitted to the hospital with a diagnosis of vaso-occlusive crisis. Which statements by the client would be most indicative of the etiology of this crisis? A) "I knew this would happen. I've been eating too much red meat lately." B) "I really enjoyed my fishing trip yesterday. I caught 2 fish." C) "I have really been working hard practicing with the debate team at school." D)"I went to the health care provider last week for a cold and I have gotten worse."

d

A 15-year-old male client was recently diagnosed with type 1 diabetes mellitus. He tells the nurse that he is having difficulty adhering to his meal plan when he is with his friends. What nursing intervention is best for the nurse to implement? a. Recommend he avoid fast food restaurants until he is familiar with his prescribed diet. b. Advise him to take his own food with him when going to fast food restaurants with his friends. c. Encourage him to find activities to do with his friends that do not involve eating. d. Assist him in identifying popular fast foods that are within his meal plan for diabetes.

d

A 2 day-old child with spina bifida and meningomyocele is in the intensive care unit after the initial surgery. As the nurse accompanies the grandparents for a first visit, which response should the nurse anticipate of the grandparents? A) Depression B) Anger C) Frustration D) Disbelief

d

A 300 ml unit of packed red blood cells is prescribed for a client with heart failure who has 3+ pitting adima, shortness of breath with any activity, and crackles in both lung bases. What rate should the nurse administer the blood? a. 50 ml/hour b. 150 ml/hour c. 300 ml/hour d. 75 ml/hour

d

A 6- year-old child who had surgery yesterday absolutely refuses to use the incentive spirometer. Which intervention should the nurse implement? A) Ask the mother to assist when it is time to use the spirometer. B) Allow child to choose when to perform incentive spirometry. C) Contract with the child to use spirometer only after meals. D) Blow out lights, blow bubbles, and encourage child's laughing.

d

A charge nurse is educating a group of unit nurses about delegating client tasks to assistive personnel a. "The nurse is legally responsible for the actions of the AP". b. "An AP can perform tasks outside of his range if he has been trained". c. "An experienced AP can delegate to another AP". d. "An RN evaluates the client needs to determine tasks to delegate"

d

A charge nurse is recommending postpartum client discharge following a local disaster. Which of the following should the nurse recommend for discharge? a. A 42-year-old client who has preeclampsia and a BP of 166/110 mm Hg. b. A 15-year-old client who delivered via emergency cesarean birth 1 day ago. c. A client who received 2 units of packed RBCs 6 hr. ago for a postpartum hemorrhage. d. A client who delivered precipitously 36 hr. ago and has a second-degree perineal laceration.

d

A client expresses anger when the call light is not answered within 5 minutes. The client demanded a blanket. The best response for the nurse to make is A) "I apologize for the delay. I was involved in an emergency." B) "Let's talk. Why are you upset about this?" C) "I am surprised that you are upset. The request could have waited a few more minutes." D) "I see this is frustrating for you. I have a few minutes so let's talk."

d

A client frequently admitted to the locked psychiatric unit repeatedly compliments and invites one of the nurses to go out on a date. The nurse's response should be to A) Ask to not be assigned to this client or to work on another unit B) Tell the client that such behavior is inappropriate C) Inform the client that hospital policy prohibits staff to date clients D) Discuss the boundaries of the therapeutic relationship with the client

d

A client has a chest tube in place following a left lower lobectomy inserted after a stab wound to the chest. When repositioning the client, the nurse notices 200 cc of dark, red fluid flows into the collection chamber of the chest drain. What is the most appropriate nursing action? A) Clamp the chest tube B) Call the surgeon immediately C) Prepare for blood transfusion D) Continue to monitor the rate of drainage

d

A client has a nasogastric tube after colon surgery. Which one of these tasks can be safely delegated to an unlicensed assistive personnel (UAP)? A) To observe the type and amount of nasogastric tube drainage B) Monitor the client for nausea or other complications C) Irrigate the nasogastric tube with the ordered irrigate D) Perform nostril and mouth care

d

A client has altered renal function and is being treated at home. The nurse recognizes that the most accurate indicator of fluid balance during the weekly visits is a. Difference in the intake and output b. Changes in the mucous membranes c. Skin turgor d. Weekly weight

d

A client is diagnosed with methicillin resistant staphylococcus aureus pneumonia. What type of isolation is most appropriate for this client? A) Reverse B) Airborne C) Standard precautions D) Contact

d

A client is receiving a hypertonic solution for bladder irrigation in as at risk for dilutional hyponatremia. The nurse should plan to observe for which common sign of hyponatremia? a. Irregular heartbeats b. Bradycardia c. Muscle spasms d. Mental status changes

d

A client is recovering from a hip replacement and is taking Tylenol #3 every 3 hours for pain. In checking the client, which finding suggests a side effect of the analgesic? A) Bruising at the operative site B) Elevated heart rate C) Decreased platelet count D) No bowel movement for 3 days

d

A client presents at the emergency department reporting a raspy voice, cold intolerance, and fatigue. Laboratory tests indicate an elevated thyroid stimulating hormone (TSH) and low T3 and T4 levels. After the client is admitted to the telemetry unit, which interventions is most important for the nurse to implement? A) Offer additional blankets and a warm drink. B) Assess for presence of non-pitting edema. C) Note the client's most recent hemoglobin level. D) Administer prescribed dose of levothyroxine.

d

A client says, "It's raining outside and it's raining in my heart. Did you know that St. Patrick drove the snakes out of Ireland? I've never been to Ireland." The nurse would document this behavior as A) Perseveration B) Circumstantiality C) Neologisms D) Flight of ideas

d

A client who is admitted to the intensive care unit with a right chest tube attached to a THORA-SEAL chest drainage unit becomes increasingly anxious and complain of difficulty breathing. The nurse determines the client is tachypneic with absent breath sounds in the client's right lungs fields. Which additional finding indicates that the client has developed a tension pneumothorax? A) Continuous bubbling in the water seal chamber. B) Decrease bright red blood drainage. C) Tachypnea and difficulty breathing. D) Tracheal deviation toward the left lung.

d

A client who is thought to be homeless is brought to the emergency department by police. The client is unkempt, has difficulty concentrating, is unable to sit still and speaks in a loud tone of voice. Which of these actions is the appropriate nursing intervention for the client at this time? A) Allow the client to randomly move about the holding area until a hospital room is available B) Engage the client in an activity that requires focus and individual effort C) Isolate the client in a secure room until control is regained by the client D) Locate a room that has minimal stimulation outside of it for admission process

d

A client with amyotrophic lateral sclerosis has a percutaneous endoscopic gastrostomy (PEG) tube for the administration of feedings and medications. Which nursing action is appropriate? A) Pulverize all medications to a powdery condition B) Squeeze the tube before using it to break up stagnant liquids C) Cleanse the skin around the tube daily with hydrogen peroxide D) Flush adequately with water before and after using the tube

d

A client with arthritis has been receiving treatment with naproxen and now reports ongoing stomach pain, increasing weakness, and fatigue. Which laboratory test should the nurse monitor? A) Serum Calcium. B) Erythrocyte sedimentation rate. C) Osmolality. D) Hemoglobin

d

A client with persistent low back pain has received a prescription for electronic stimulator (TENS) unit. After the nurse applies the electrodes and turns on the power, the client reports feeling a tingling sensation. How should the nurse respond? A) Remove electrodes and observe for skin redness. B) Decrease the strength of the electrical signals. C) Check the amount of gel coating on the electrodes. D) Determine if the sensation feels uncomfortable.

d

A female client is admitted for diabetic crisis resulting from inadequate dietary practice. After stabilization, the nurse talks to the client about her prescribed diet. What client characteristic is most important for successful adherence to the diabetic diet? A) Knows that insulin must be given 30 min before eating. B) Frequently eats fruits and vegetables at meals and between meals. C) Has someone available who can prepare and oversee the diet. D) Demonstrates willingness to adhere to the diet consistently.

d

A male client is admitted to the hospital due to multiple fractures following a motor vehicle collision that occurred when he ran his car into his ex-spouse's home. When the client becomes angry and starts throwing objects at the staff, which PRN prescription should the nurse implement? A) Apply soft wrist restraints if needed for client safety. B) Consult with the chaplain emotional support. C) Hydromorphone (Dilaudid) 2mg IV. D) Haloperidol (Haldol) 1mg IM.

d

A male client with cirrhosis and severe ascites, who is scheduled for a paracentesis tells the nurse that he is in pain and feel short of breath, so he wants to reschedule the procedure. How should the nurse respond? a. Advise the client that the procedure will help diagnose the cause of his symptoms b. Encourage the client to verbalize his fears about the outcome of the procedure c. Offer to notify the health care provider of his desire to reschedule the procedure d. Explain to the client that the paracentesis will provide relief from his discomfort

d

A middle aged woman talks to the nurse in the health care provider's office about uterine fibroids also called leiomyomas or myomas. What statement by the woman indicates more education is needed? A) I am one out of every 4 women that get fibroids, and of women my age - between the 30s or 40s, fibroids occurs more frequently. B) My fibroids are noncancerous tumors that grow slowly. C) My associated problems I have had are pelvic pressure and pain, urinary incontinence, frequent urination or urine retention and constipation. D) Fibroids that cause no problems still need to be taken out.

d

A mother with a Roman Catholic belief has given birth in an ambulance on the way to the hospital. The neonate is in very critical condition with little expectation of surviving the trip to the hospital. Which of these requests should the nurse in the ambulance anticipate and be prepared to do? A) The refusal of any treatment for self and the neonate until she talks to a reader B) The placement of a rosary necklace around the neonate's neck and not to remove it unless absolutely necessary C) Arrange for a church elder to be at the emergency department when the ambulance arrives so a "laying on hands" can be done D) Pour fluid over the forehead backwards towards the back of the head and say "I baptize you in the name of the father, the son and the holy spirit. Amen."

d

A newly admitted adult client has a diagnosis of hepatitis A. The charge nurse should reinforce to the staff members that the most significant routine infection control strategy, in addition to hand washing, to be implemented is which of these? A) Apply appropriate signs outside and inside the room B) Apply a mask with a shield if there is a risk of fluid splash C) Wear a gown to change soiled linens from incontinence D) Have gloves on while handling bedpans with feces

d

A nurse checks a client who is on a volume-cycled ventilator. Which finding indicates that the client may need suctioning? A) drowsiness B) complaint of nausea C) pulse rate of 92 D) restlessness

d

A nurse in a provider's office is reviewing the laboratory results of a group of clients. Which to report? a. Herpes simplex. b. Human papillomavirus c. Candidiasis d. Chlamydia

d

A nurse in emergency department is caring for a client who has full thickness burn of the thorax and upper torso. After securing the client's airway, which of the following is the nurse's priority intervention? a. Providing pain management b. Offering emotional support c. Preventing infection d. Initiating IV fluid resuscitation

d

A nurse in the emergency department is assessing newly admitted client who is experiencing drooling and hoarseness following a brain injury. Which of the following actions should the nurse take first? a. obtain a baseline EKG b. Obtain a blood specimen for ABG analysis c. insert an 18 gauge IV catheter d. Administer 100% humidified oxygen

d

A nurse is administering a scheduled medication to a client. The client reports that the medication appears different than what they take at home. Which of the following responses should the nurse take? a. "Did the doctor discuss with you that there was a change in this medication?" b. "I recommend that you take this medication as prescribed" c. "Do you know why this medication is being prescribed to you?" d. " I will call the pharmacist now to check on this medication"

d

A nurse is admitting a client who has schizophrenia and experiences auditory hallucinations. The client states, "It's hard not to listen to the voices." Which of the following questions should the nurse ask the client? a. "Do you understand that the voices are not real?" b. "Why do you think the voices are talking to you?" c. "Have you tried going to a private place when this occurs?" d. "What helps you ignore what you are hearing?"

d

A nurse is assessing a client who is 2 hrs postpartum for uterine atony. Which of the following action should the nurse take? a. Monitor the client's urinary output b. Check the client VS c. Evaluate the client's pain level d. Palpate the client's fundus

d

A nurse is caring a child who has cystic fibrosis and requires postural drainage. Which of the following actions should the nurse take? A. Hold hand flat to perform percussion on the child B. Perform the procedure twice a day C. Administer a bronchodilator after the procedure D. Perform the procedure prior to meals

d

A nurse is caring for a client who has end-stage liver cancer. Which of the following statements should the nurse make to support the client's right to autonomy? A. "You should trust that your care team has your best interest at heart" B. "I will not share any personal information without your permission" C. "The health care team will do their best to keep any promise we make to you" D. "We encourage you to participate in all decisions about your treatment"

d

A nurse is caring for a client who has hyperthermia .Which of the following actions for the nurse to take? a. Submerge the adolescent feet in ice water b. Cover the adolescent with a thermal blanket c. Administer oral acetaminophen d. Initiate seizure precautions

d

A nurse is caring for a client who has left-sided heart failure, and the provider is concerned that the client might develop. Which of the following actions should the nurse take? a. Maintain the client's oxygen saturation level at 89%. b. Place the client's lower extremities on two pillows. c. Recommended that the client follow a 3g sodium diet. d. Place the client in high fowler's position.

d

A nurse is caring for a client who has severe preeclampsia and is receiving magnesium sulfate intravenously. The nurse discontinues the magnesium sulfate after the client displaces toxicity. Which of the following actions should the nurse take? a. Position the client supine b. Prepare an IV bolus of dextrose 5% in water c. Administer methylergonovine IM d. Administer calcium gluconate IV

d

A nurse is caring for an infant who has coarctation of the aorta. Which of the following should the nurse identify as an expected finding? a. Upper extremity hypotension b. Increased intracranial pressure c. Frequent nosebleeds d. Weak femoral pulses

d

A nurse is caring for four clients who are scheduled for surgery the same day. Which of the following laboratory values indicates the need for intervention before surgery? a. Fasting blood glucose 108 mg/dl b. WBC 9,800/mm c. Creatinine 0.9 mg/dl d. Potassium 5.2 mEq/L

d

A nurse is developing a plan of care for a client who has preeclampsia and is to receive magnesium sulfate via continuous IV infusion. Which of the following actions should the nurse include in the plan? a. Restrict the client's total fluid intake to 250 mL/hr b. Give the protamine if signs of magnesium sulfate toxicity occur c. Monitor the FHR via Doppler every 30min d. Measure the client's urine output every hour

d

A nurse is planning care for a client who has a prescription for a bowel- training program following a spinal cord injury. Which of the following actions should the nurse include in the plan of care? a. Encourage a maximum fluid intake of 1,500 ml per day. b. Increase the amount of refined grains in the client's diet. c. Provide the client with a cold drink prior to defecation. d. Administer a rectal suppository 30 minutes prior to scheduled defecation times.

d

A nurse is planning care for a client who has unilateral paralysis and dysphagia following a right hemispheric stroke. Which of the following interventions should the nurse include in the plan? a. Place food on the left side of the client's mouth when he is ready to eat. b. Provide total care in performing the client's ADLs. c. Maintain the client on bed rest. d. Place the client's left arm on a pillow while he is sitting.

d

A nurse is preparing discharge information for a client who has type 2 diabetes mellitus. Which of the following resources should the nurse provide to the client? a. Personal blogs about managing the adverse effects of diabetes medications b. Food label recommendations from the Institute of Medicine c. Diabetes medication information from the Physicians' Desk Reference d. Food exchange lists for meal planning from the American Diabetes Association

d

A nurse is preparing to administer an autologous blood product to a client. Which of the following actions should the nurse take to identify the client? a. Match the client's blood type with the type and cross match specimens. b. Confirm the provider's prescription matches the number on the blood component. c. Ask the client to state the blood type and the date of their last blood donation. d. Ensure that the client's identification band matches the number on the blood unit.

d

A nurse is preparing to reposition a client who had a stroke. Which of the following actions should the nurse take? a. Raise the side rails on both sides of the client's bed during repositioning. b. Reposition the client without assistive devices. c. Discuss the client's preferences for determining a reposition schedule. d. Evaluate the client's ability to help with repositioning.

d

A nurse is providing teaching about digoxin administration to the parents of a toddler who has heart failure. Which of the following statements should the nurse include in the teaching? a. "You can add the medication to a half-cup of your child's favorite juice." b. "Repeat the dose if your child vomits within 1 hour after taking the medication." c. "Limit your child's potassium intake while she is taking this medication." d. "Have your child drink a small glass of water after swallowing the medication."

d

A nurse is providing teaching about patient-controlled analgesia (PCA) to a client. Which of the following statements should the nurse include in the teaching? a. "The PCA will deliver a double dose of medication when you push the button twice." b. "You can adjust the amount of pain medication you receive by pushing on the keypad." c. "Continuous PCA infusion is designed to allow fluctuating plasma medication levels." d. "You should push the button before physical activity to allow maximum pain control."

d

A nurse is providing teaching to an adolescent who has type 1 diabetes mellitus. Which of the following goals should the nurse include in the teaching a. HbA1c level greater than 8% b. Blood glucose level greater than 200 mg/dL at bedtime c. Blood glucose level less than 60 mg/dL before breakfast d. HbA1c level less than 7%

d

A nurse is reviewing admission prescriptions for a group of clients. Which of the following prescriptions should the nurse identify as complete? a. Furosemide 20 mg BID b. Nitroglycerin transdermal patch. c. Aspirin 1 tablet daily. d. Metoprolol 5mg IV now.

d

A nurse is reviewing the laboratory report of a client who has been having lithium carbonate for the past 12 months. The nurse notes a lithium level of 0.8 mEq/L. Which of the following orders from the provider should the nurse expect? a. Withhold the next dose. b. Increase the dosage. c. Discontinue the medication. d. Administer the medication.

d

A nurse is reviewing the laboratory results of a client who has rheumatoid arthritis. Which of the following findings should the nurse report to the provider? a. WBC count 8,000 b. platelets 150,000 c. aspartate aminotransferase 10 units d. erythrocyte sedimentation rate 75mm

d

A nurse is reviewing the medical record of a client who has a prescription for intermittent heat therapy for a foot injury. Which if the following findings should the nurse identify as a contraindication for heat therapy? a. Phlebitis b. Abdominal aortic aneurysm c. Osteoarthritis d. Peripheral neuropathy

d

A nurse is reviewing the medical record of a client who is postoperative following a total hip arthroplasty. For which of the following findings should the nurse contact the provider? A. Hear rate 100/min B. Temperature 37.8C (100F) C. Albumin level 4.0 g/dL. D. WBC count 14,000 mm3

d

A nurse is supervising an assistive personnel (AP) who is feeding a client. The nurse observes that the client coughs after each bite. After asking the AP to stop feeding the client, which of the following actions should the nurse take next? a. Provide the client with an instructional handout about swallowing exercises. b. Ask a speech therapist to evaluate the client's ability to swallow. c. Discuss the manifestations of impaired swallowing with the AP. d. Listens to the client's lung sounds.

d

A nurse is talking with another nurse on the unit and smells alcohol on her breath. Which of the following actions should the nurse take? A. Confront the nurse about the suspected alcohol use. B. Inform another nurse on the unit about the suspected alcohol use. C. Ask the nurse to finish administering medications and then go home. D. Notify the nursing manager about the suspected alcohol use.

d

A nurse is to collect a sputum specimen for acid-fast bacillus (AFB) from a client. Which action should the nurse take first? A) Ask client to cough sputum into container B) Have the client take several deep breaths C) Provide a appropriate specimen container D) Assist with oral hygiene

d

A nurse overhears two assistive personnel (AP) discussing care for a client while in the elevator. Which of the following actions should the nurse take? a. Contact the client's family about the incident. b. Notify the client's provider about the incident. c. File a complaint with the facility's ethics committee. d. Report the incident to the AP's charge nurse.

d

A parent asks the school nurse how to eliminate lice from their child. What is the most appropriate response by the nurse? A) Cut the child's hair short to remove the nits B) Apply warm soaks to the head twice daily C) Wash the child's linen and clothing in a bleach solution D) Application of pediculicides

d

A resident of a long-term care facility, who has moderate dementia, is having difficulty Eating in the dining room. The client becomes frustrated when dropping utensils on the floor and then refuses to eat. What actions should the nurse implement? A) Allow client to choose foods from a menu. B) Assign a staff member to feed the client. C) Have meals brought to the to the client's room. D) Encourage the client to eat finger foods.

d

A woman at 12 weeks' gestation comes into the clinic for her first prenatal visit. After completing health history, the nurse should discuss which topic about pregnancy at this initial visit? a. Concerns about parenting b. Knowledge about labor and delivery c. Complications associated with childbirth d. Cultural practices related to childbearing

d

After 2 days treatment for dehydration, a child continues to vomit and have diarrhea. Normal saline is infusing and the child's urine output is 50ml/hour. During morning assessment, the nurse determines that the child is lethargic and difficult to arouse. Which should the nurse implemented? A) Increase the IV fluid flow rate. B) Review 24 hour intake and output. C) Obtain arterial blood gases. D) Perform a finger stick glucose test

d

An American Indian chief visits his newborn son and performs a traditional ceremony that involves feathers and chanting. The attending nurse tells a colleague "I wonder if he has any idea how ridiculous he looks -- he's a grown man!" The nurse's response is an example of A) Discrimination B) Stereotyping C) Ethnocentrism D) Prejudice

d

An older adult male is admitted with complication related to Chronic Obstructive Pulmonary Disease (COPD). He reports progressive dyspnea that worsens on exertion and his weakness has increased over the past month. The nurse notes that he has dependent edema in both lower legs. Based on these assessment findings, which dietary instruction should the nurse provide? A) Limit the intake of high calorie foods. B) Maintain a low protein diets. C) Eat meals at the same time daily. D) Restricts daily fluid intake.

d

An older client comes to the clinic with a family member. When the nurse attempts to take the client's health history, the client does not respond to questions in a clear manner. What action should the nurse implement first? A. Provide a printed health care assessment form B. Ask the family member to answer the questions C. Defer the health history until the client is less anxious D. Assess the surroundings for noise and distractions

d

An older client with a history of pernicious anemia has developed ataxia and paresthesia. In planning care, which nursing intervention has the highest priority? A) Provide assistance with ambulation. B) Keep the head of the bed elevated. C) Offer a PRN sleep aid at night. D) Instruct about healthy diet choices.

d

An older female client living in a low-income apartment complex nurse that she is concerned about her 81-year-old neighbor, a widow whose son recently assumed her financial affairs. Lately, her neighbor has become reclusive, but is occasionally seen walking outside wearing only a robe and slippers. What response should the nurse offer? A) Provide the number for Adult Protective Services so the client can report any suspicion of elder abuse. B) Encourage the client to avoid becoming involved in the neighbor's problems, for one's own protection. C) Tell the client to talk to a healthcare provider before reporting suspicion of neglect to the authorities. D) Explain that it is not unusual for older adults to suffer from dementia, which often causes such behaviors

d

An older male client is admitted with the medical diagnosis of possible cerebral vascular accident (CVA). He has facial paralysis and cannot move his left side. When entering the room, the nurse finds the client's wife tearful and trying unsuccessfully to give him a drink of water. What action should the nurse take? A) Give the wife a straw to help facilitate the client's drinking. B) Assist the wife and carefully give the client small sips of water. C) Obtain a thickening powder before providing any more fluids. D) Ask the wife to stop and assess the client's swallowing reflex.

d

An unlicensed assistive personnel (UAP), who usually works on a surgical unit is assigned to float to a pediatric unit. Which question by the charge nurse would be most appropriate when making delegation decisions? A) "How long have you been a UAP and what units you have worked on?" B) "What type of care do you give on the surgical unit and what ages of clients?" C) "What is your comfort level in caring for children and at what ages?" D) "Have you reviewed the list of expected skills you might need on this unit?"

d

As the nurse is speaking with a group of teens which of these side effects of chemotherapy for cancer would the nurse expect this group to be more interested in during the discussion? A) Mouth sores B) Fatigue C) Diarrhea D) Hair loss

d

During a Women's Health Fair, which assignment is best for the practical nurse (PN) who is working with a registered nurse (RN)? A) Encourage a woman at risk for cancer to obtain a colonoscopy. B) Present a class on bread self-examination. C) Explain the follow up needed for a client with prehypertension. D) Prepare a woman for a bone density screening.

d

During the change-of-shift report the assigned nurse notes a Catholic client is scheduled to be admitted for the delivery of a ninth child. Which comment stated angrily to a colleague by this nurse indicates an attitude of prejudice? A) "I wonder who is paying for this trip to the hospital?" B) "I think she needs to go to the city hospital." C) "All those people indulge in large families!" D) "Doesn't she know there's such a thing as birth control?"

d

Following laser trabeculoplasty surgery for open-angle glaucoma, the client reports acute pain deep within the eye. What action should the nurse take? A) Apply bilateral eye shields to reduce photosensitivity. B) Begin postoperative prophylactic antibiotics. C) Administer an antiemetic to prevent vomiting. D) Report the complain of eye pain to the surgeon

d

The nurse has been teaching a client with Insulin Dependent Diabetes Mellitus. Which statement by the client indicates a need for further teaching? A) "I use a sliding scale to adjust regular insulin to my sugar level." B) "Since my eyesight is so bad, I ask the nurse to fill several syringes." C) "I keep my regular insulin bottle in the refrigerator." D) "I always make sure to shake the NPH bottle hard to mix it well.

d

The nurse is about to assess a 6 month-old child with nonorganic failure-to thrive (NOFTT). Upon entering the room, the nurse would expect the baby to be A) Irritable and "colicky" with no attempts to pull to standing B) Alert, laughing and playing with a rattle, sitting with support C) Skin color dusky with poor skin turgor over abdomen D) Pale, thin arms and legs, uninterested in surroundings

d

The nurse is assigned to a client newly diagnosed with active tuberculosis. Which of these protocols would be a priority for the nurse to implement? A) Have the client cough into a tissue and dispose in a separate bag B) Instruct the client to cover the mouth with a tissue when coughing C) Reinforce for all to wash their hands before and after entering the room D) Place client in a negative pressure private room and have all who enter the room use masks with shields

d

The nurse is caring for a client in hypertensive crisis in an intensive care unit. The priority assessment in the first hour of care is A) Heart rate B) Pedal pulses C) Lung sounds D) Pupil responses

d

The nurse is caring for a client who is receiving continuous ambulatory peritoneal dialysis (CAPD) and notes that the output flow is 100ml less than the input flow. Which actions should the nurse implement first? A) Continue to monitor intake and output with next exchange. B) Check the clients blood pressure and serum bicarbonate. C) Irrigate the dialysis catheter. D) Change the client's position.

d

The nurse is conducing a visual screening of a group of older adults. Which finding should the nurse report to the healthcare provider immediately? a. Gradual onset of continuous eye pain and blurred vision. b. Recent change in the ability to read and drive after dark. c. Gray-white circle around the iris of both eyes. d. Cloudy opacity of the crystalline lens.

d

The nurse is performing a peritoneal dialysis exchange on a client with chronic kidney disease (CKD). Which assessment finding should the nurse report to the healthcare provider? A) The client complains of abdominal fullness and cramping during installation. B) The client complains of a slight shortness of breath during installation. C) The amount of the returning dialysis fluid is greater than the amount instilled. D) The appearance of the returning dialysate fluid is cloudy.

d

The nurse is performing an assessment of the motor function in a client with a head injury. The best technique is A) A firm touch to the trapezius muscle or arm B) Pinching any body part C) Sternal rub D) Gentle pressure on eye orbit

d

The nurse is preparing a teaching plan for an older female client diagnosed with osteoporosis. What expected outcome has the highest priority for this client? A) Lists 5 calcium-rich foods to be added to her daily diet. B) Identifies 2 treatments for constipation due to immobility. C) State 4 risk factors for the development of osteoporosis. D) Names 3 home safety hazards to be resolve immediately.

d

The nurse is preparing an adult with Addison's disease for self-management. Which information should the nurse include in the client's instructions? a. Importance of recording daily weights b. Adherence to a high fiber low fat diet c. Need to check temperature daily d. Events requiring steroid dosage adjustments

d

The nurse is teaching a mother of a newborn with a cleft lip how to bottle feed her baby using a Medela Haberman feeder, which has a valve to control the release of milk and a slit nipple opening. The nurse discusses placing the nipple's elongated tip in the back of the oral cavity. What instruction should the nurse provide the mother about feedings? A) Alternate milk with water during the feedings. B) Squeeze the nipple base to introduce milk into the mouth. C) Position the baby in the left lateral position after feeding. D) Hold the newborn in an upright position.

d

The nurse manager informs the nursing staff at morning report that the clinical nurse specialist will be conducting a research study on staff attitudes toward client care. All staff are invited to participate in the study if they wish. This affirms the ethical principle of A) Anonymity B) Beneficence C) Justice D) Autonomy

d

The nurse observes an unlicensed assistive personnel (UAP) who is preparing to provide personal care for a client who requires contact precautions. The UAP has applied a gown and gloves and secured the tops of the gloves over the gown sleeves. What action should the nurse take? a. Remind the UAP to wash hands frequently while in the room b. Help the UAP reposition the gown sleeve over the glove edges c. Confirm that the gown is tide securely at the neck and waist d. Assist the UAP with application of a face mask or face shield

d

The nurse receives an order to give a client iron by deep injection. The nurse know that the reason for this route is to A) enhance absorption of the medication B) ensure that the entire dose of medication is given C) provide more even distribution of the drug D) prevent the drug from tissue irritation

d

The school nurse is preparing a teaching pamphlet in response to requests from parents regarding an outbreak of pinworms at the local preschool. Which information about the most commonly prescribed medication, mebendazole, should be taken? A) Insert the medication as a rectal suppository. B) A second dose of medication should be given in two weeks. C) It is safe for children of all ages to take this medication .D) Only children with perianal itching should take the medication.

d

When administering brompheniramine maleate, an extended release antihistamine tablet, the nurse is told by the male client that he cannot swallow tablets. Which intervention should the nurse implement? A) Document the client's refusal to take the medication. B) Crush tablet and mix with small amount of pudding. C) Document that the client cannot take the prescription .D) Ask the pharmacist to send it in liquid form.

d

When planning care for a client with acute pancreatitis, which nursing intervention has the highest priority? A) Administer antiemetic as needed. B) Initiate IV fluid replacement. C) Evaluate intake and output ratio. D) Withhold food and fluid intake.

d

Which bed position is preferred for use with a client in an extended care facility on falls risk prevention protocol? A) All 4 side rails up, wheels locked, bed closest to door B) Lower side rails up, bed facing doorway C) Knees bent, head slightly elevated, bed in lowest position D) Bed in lowest position, wheels locked, place bed against wall

d

Which information is a priority for the RN to reinforce to an older client after intravenous pyelogram? a. Eat a light diet for the rest of the day b. Rest for the next 24 hours since the preparation and the test is tiring. c. During waking hours drink at least 1 8-ounce glass of fluid every hour for the next 2 days d. Measure the urine output for the next day and immediately notify the health care provider if it should decrease.

d

Which nursing responsibility is related to health promotion and teaching for the client with rheumatoid arthritis? a. Prevention through nutrition and exercise b. Avoidance of foods containing purine c. Immobilization of affected joints d. Application of heat and cold therapy

d

Which of these clients who are all in the terminal stage of cancer is least appropriate to suggest the use of patient controlled analgesia (PCA) with a pump? A) A young adult with a history of Down's syndrome B) A teenager who reads at a 4th grade level C) An elderly client with numerous arthritic nodules on the hands D) A preschooler with intermittent episodes of alertness

d

Which of these clients who call the community health clinic would the nurse ask to come in that day to be seen by the health care provider? A) I started my period and now my urine has turned bright red. B) I am an diabetic and today I have been going to the bathroom every hour. C) I was started on medicine yesterday for a urine infection. Now my lower belly hurts when I go to the bathroom. D) I went to the bathroom and my urine looked very red and it didn't hurt when I went.

d

Which of these clients would the nurse recommend to keep in the hospital during an internal disaster at the agency? A) An adolescent diagnosed with sepsis 7 days ago with vital signs maintained within low normal B) A middle-aged woman documented to have had an uncomplicated myocardial infarction 4 days ago C) An elderly man admitted 2 days ago with an acute exacerbation of ulcerative colitis D) A young adult in the second day of treatment for an overdose of acetometaphen

d

Which of these findings would indicate that the nurse-client relationship has passed from the orientation phase to the working phase? The client A) Has revitalized a relationship with her family to help cope with the death of a daughter B) Had recognized regressive behavior as a defense mechanism C) Expresses a desire to be cared for and pampered D) Recognizes feelings with appropriate expression of feelings

d

Which of these nursing diagnoses of 4 elderly clients would place 1 client at the greatest risk for falls? A) Sensory perceptual alterations related to decreased vision B) Alteration in mobility related to fatigue C) Impaired gas exchange related to retained secretions D) Altered patterns of urinary elimination related to nocturia

d

Which statement by the client during the initial assessment in the the emergency department is most indicative for suspected domestic violence? A) "I am determined to leave my house in a week." B) "No one else in the family has been treated like this." C) "I have only been married for 2 months." D) "I have tried leaving, but have always gone back."

d

Which task could be safely delegated by the nurse to an unlicensed assistive personnel (UAP)? A) Be with a client who self-administers insulin B) Cleanse and dress a small decubitus ulcer C) Monitor a client's response to passive range of motion exercises D) Apply and care for a client's rectal pouch

d

While changing the dressing of a client who is immobile, the nurse notices the boundary of the wound has increased. Before reporting this finding to the healthcare provider, the nurse should evaluate which of the client's laboratory values? A) Serum potassium and sodium levels. B) C-reactive protein level. C) Platelet count. D) Neutrophil count.

d

nurse is planning discharge teaching about cord care for the parents of a newborn. Which of the following instructions should the nurse plan to include in the teaching? a. "The cord stump will fall off in 5 days." b. "Contact the provider if the cord stump turns black." c. "Clean the base of the cord with hydrogen peroxide daily." d. "Keep the cord stump dry until it falls off."

d

***********Which conditions are most likely to respond to treatment with antihistamines? A. Bronchitis B. Myocarditis C. Otitis media D. Contact dermatitis E. Allergic rhinitis

d e

***********The nurse assesses a child in 90-90 skeletal traction. Where should the nurse assess for signs of compartment syndrome?

****Click spot right on toes on injured foot (left leg)

A client with multiple sclerosis (MS) is receiving interferon beta-1b 0.1875 mg subcutaneously QOD. The nurse reconstitutes the vial by slowly injecting 1.2 ml of diluent into the interferon vial for a reconstituted solution of 0.25 mg/1 ml. How many ml should the nurse administer? (Enter numerical value only. If required, round to the nearest hundredth.)

0.75

A home health nurse is preparing to make an initial visit to a family following a referral from a local provider. Identify the sequence of steps the nurse should take when conducting a home visit. (Move the steps into the box on the right. Placing them in the order of performance) 1. Identify family needs interventions using the nursing process. 2. Record information about the home visit according to agency policy. 3. Contact the family to determine availability and readiness to make an appointment 4. Discuss plans for future visits with the family. 5. Clarify the reason for the referral with the provider's office.

1, 2, 3, 5, 4

********A client who is admitted for primary hypothyroidism has early signs of myxedema coma. In assessing the client, in which sequence should the nurse complete these actions? (descending order) Measure body temperature Assess blood pressure Palpate for pedal edema Observe breathing patterns

1. Observe breathing patterns 2. Assess blood pressure 3. Measure body temperature 4. Palpate for pedal edema

The nurse is using a straight urinary catheter kit to collect a sterile urine specimen from a female client. After positioning am prepping this client, rank the actions in the sequence they should be implemented. (Place to first action on the top on the last action on the bottom.) -Place distal end of the catheter in sterile specimen cup and insert catheter into meatus -Don sterile gloves and prepare to sterile field. -Cleanse the urinary meatus using the solution, swabs, and forceps provided. -Open the sterile catheter kit close to the client's perineum.

1. Open the sterile catheter kit close to the client's perineum. 2. Don sterile gloves and prepare to sterile field. 3. Cleanse the urinary meatus using the solution, swabs, and forceps provided. 4. Place distal end of the catheter in sterile specimen cup and insert catheter into meatus

******An older adult male reporting abdominal pain is admitted to the hospital from a long-term care facility. It has been 7 days since his last bowel movement, his abdomen is distended, and he just vomited 150mL of dark brown emesis. In what order should the nurse implement these interventions? (Highest to lowest priority) Elevate the head of the bed Complete focused assessment Offer PRN pain medication Send emesis sample to the lab

1. Send emesis sample to the lab 2. Elevate the head of the bed 3. Complete focused assessment 4. Offer PRN pain medication

**********A client develops urticaria on the trunk and neck shortly after a secondary infusion of piperacillin is initiated. In what order should the nurse implement these interventions? Document reaction to the drug Assess vital signs Initiate an adverse event report Stop the infusion Contact the healthcare provider

1. Stop the infusion 2. Assess vital signs 3. Contact the healthcare provider 4. Document reaction to the drug 5. Initiate an adverse event report

The healthcare provider prescribes potassium chloride 25 mEq in 500ml D5W to infuse over 6 hours. The available 20ml vial of potassium chloride is labeled, <How many ml of potassium chloride should the nurse add to the IV fluid? (Round to the nearest tenth.)

12.5 ml

******A heparin infusion is prescribed for a client who weighs 220 pounds. After administering a bolus dose of 80 units/kg, the nurse calculates the infusions rate for the heparin solution at 18 units/kg/hour. The available solution is Heparin Sodium 25,000 Units in 5% Dextrose Injection 250ml. The nurse should program the infusion pump to deliver how many ml/hours?

18

******The healthcare provider prescribes a fluid challenge of 0.9% sodium chloride 1,000 mL to be infused intravenously over 4 hours. The IV administration set delivers 10gtt/mL. How many gtt/minute should the nurse regulate the infusion? (Round to the nearest whole number)

42 gtt/min

*******After administering a 12 ounce can of nutritional supplement, 3 teaspoons of medication, and 120 mL of water, the nurse should document the client's fluid intake as how many mL?

495

******The healthcare provider prescribes penicillin G benzathine 2,400,000 units intramuscularly for a client who has a postoperative wound infection. The prefilled syringe is labeled, penicillin G benzathine 1,200,000 units/2mL. How many mL should the nurse administer to this client?

4mL

During the admissions assessment, the nurse auscultates heart sounds for a client with no history of cardiovascular disease. Where should the nurse listen when assessing the client's point of maximal impulse (PMI). (Click the chosen locations.)

4th intercostal left side.

********The healthcare provider prescribes cephalexin 125mg/5mL oral suspension for a client who weighs 77 pounds. The recommended safe dose 25mg/kg/24 hours in 4 divided doses. Based on the client's weight, how many mL should the nurse administer?

9

**************A client has both a primary IV infusion and a secondary infusion of medication. An infusion pump is not available. The nurse needs to change the rate of flow of the secondary infusion. Where should the nurse regulate the rate of the secondary IV?

??? maybe under yellow infusion

**********The nurse observes a client prepare a meal in the kitchen of a rehabilitation facility prior to discharge. Which behaviors indicates the client understands how to maintain balance safely? (Select all that apply) A. Brings a heavy can close to body before lifting B. Leans forward to pull on a pan from a high shelf C. Locks knees while preparing food on the counter D. Bends from the waist to pick trash off the floor E. Widens stance while working near the sink

A e

*****The nurse has complete the diet teaching of a client who is being discharged following treatment of a leg wound. A high protein diet is encouraged to promote wound healing. Which lunch choice by the client indicates that the teaching was effective? A. A tuna fish sandwich with chips and ice cream B. A salad with three kinds of lettuce and fruit C. A peanut butter sandwich with soda and cookies D. Vegetable soup, crackers, and milk

A. A tuna fish sandwich with chips and ice cream

*******A client with acute pancreatitis is admitted with severe, piercing abdominal pain and an elevated serum amylase. Which additional information is the client most likely to report to the nurse? A. Abdominal pain decreases when lying supine B. Pain lasts an hour and leaves the abdomen tender C. Right upper quadrant pain refers to right scapula D. Drinks alcohol until intoxicated at least twice weekly.

A. Abdominal pain decreases when lying supine

*****The nurse is caring for a group of clients with the help of a practical nurse (PN). Which nursing actions should the nurse assign to the PN? (Select all that apply) A. Administer a dose of insulin per sliding scale for a client with Type 2 DM B. Start the second blood transfusion for a client 12 hours following a BKA C. Initiate patient controlled analgesia (PCA) pumps for two clients immediately postoperatively D. Perform daily surgical dressing change for a client who had an abdominal hysterectomy E. Obtain postoperative vital signs for a client one day following unilateral knee arthroplasty

A. Administer a dose of insulin per sliding scale for a client with Type 2 DM D. Perform daily surgical dressing change for a client who had an abdominal hysterectomy E. Obtain postoperative vital signs for a client one day following unilateral knee arthroplasty

*********A female client with a history of heart failure (HF) arrives at the clinic after what she describes as a very long trip. Following the initial physical assessment and chart review, which priority action should the nurse implement? A. Administer the prescribed diuretic B. Give a potassium supplement C. Reteach medication regimen D. Auscultate lung and heart sounds

A. Administer the prescribed diuretic

*****A client who gave birth 48 hours ago has decided to bottle feed the infant. During the assessment, the nurse observes that both breasts are swollen, warm, and tender on palpation. Which instruction should the nurse provide? A. Apply ice to the breasts for comfort B. Wear a loose-fitting bra during the day to prevent nipple irritation C. Run warm water over breasts D. Express small amounts of milk from the breasts to relieve pressure

A. Apply ice to the breasts for comfort

*****The nurse is caring for a preterm newborn with nasal flaring, grunting, and sternal retractions. After administering surfactant, which assessment is most important for the nurse to monitor? A. Arterial blood gasses B. Breath sounds C. Oxygen saturation D. Respiratory rate

A. Arterial blood gasses

*****A client presses the call bell and requests pain medication for a severe headache. To assess the quality of the client's pain, which approach should the nurse use? A. Ask the client to describe the pain B. Observe body language and movement C. Identify effective pain relief measures D. Provide a numeric pain scale

A. Ask the client to describe the pain

*********The nurse who works in labor and delivery is reassigned to the cardiac care unit for the day because of a low census in labor and delivery. Which assignment is best for the charge nurse to give this nurse? A. Assist cardiac nurses with their assignments B. Monitor the central telemetry C. Perform the admission of a new client D. Transfer a client to another unit

A. Assist cardiac nurses with their assignments

******The nurse identifies an electrolyte imbalance, a weight gain of 4.4lbs (2kg) in 24 hours and an elevated central venous pressure for a client with full thickness burns. Which intervention should the nurse implement? A. Auscultate for irregular heart rate B. Review arterial blood gases results C. Measure ankle circumference D. Document abdominal girth

A. Auscultate for irregular heart rate

****A pediatric client is taking the beta-adrenergic blocking agent propranolol. In developing a teaching plan, the nurse should teach the parents to report which sign of overdose? A. Bradycardia B. Tachypnea C. Hypertension D. Coughing

A. Bradycardia

*******A client with a lower respiratory tract infection receives a prescription for ciprofloxacin 500mg PO every 12 hours. When the client requests an afternoon snack, which dietary choice should the nurse provide? A. Cinnamon applesauce B. Vanilla-flavored yogurt C. Calcium-fortified juice D. Low-fat chocolate milk

A. Cinnamon applesauce

****A young adult visits the client reporting symptoms associated with gastritis. Which information in the client's history is most important for the nurse to address in the teaching plan? A. Consumes 10 or more drinks of alcohol every weekend B. Snacks on foods with very high salt content on a daily basis C. Exercises vigorously every evening right before going to bed D. Recently became a vegetarian and eats a lot of high fiber foods

A. Consumes 10 or more drinks of alcohol every weekend

*****A young male client is admitted to rehabilitation following a right above-knee amputation (AKA) for a severe traumatic injury. He is in the commons room and anxiously calls out to the nurse, stating that his "right foot is aching". The nurse offers reassurance and support. Which additional intervention is most important for the nurse to implement? A. Encourage discussion of feelings about the loss of his limb B. Administer a prescription for gabapentin, a neuroleptic agent C. Tech the client how to wrap the stump with an elastic bandage D. Offer to assist the client to a quieter location so he can relax

A. Encourage discussion of feelings about the loss of his limb

******The home care nurse visits a client who has cancer. The client reports having a good appetite but experiencing nausea when smelling food cooking. Which action should the nurse implement? A. Encourage family members to cook meals outdoors and bring the cooked food inside B. Assess the client's mucous membranes and report the findings to the healthcare provider C. Advise the client to replace cooked foods with a variety of different nutritional supplements D. Instruct the client to take an antiemetic before every meal to prevent excessive vomiting

A. Encourage family members to cook meals outdoors and bring the cooked food inside

*****The nurse is wearing personal protective equipment (PPE) while caring for a client. When exiting the room, which PPE should be removed first? A. Gloves B. Mask C. Eyewear D. Gown

A. Gloves

******While assessing a client who is admitted with heart failure and pulmonary edema, the nurse identifies dependent peripheral edema, an irregular heart rate, and a persistent cough that produces pink blood-tinged sputum. After initiating continuous telemetry and positioning the client, which intervention should the nurse implement? A. Obtain sputum sample B. Document degree of edema C. Initiate hourly urine output measurement D. Administer intravenous diuretics

A. Obtain sputum sample

*********A client presents to the emergency department with muscle aches, headache, fever, and describes a recent loss of taste and smell. The nurse obtains a nasal swab for COVID-19 testing. Which action is most important for the nurse to take? A. Place the nasal swab specimen for COVID-19 directly into a biohazard bag B. Move the client to a private room, keep the door closed, and initiate droplet precautions. C. Teach the client to wear a mask, hand wash, and social distance to prevent spreading the virus D. Explain to the client to inform others that they may have been potentially exposed in the last 14 days.

A. Place the nasal swab specimen for COVID-19 directly into a biohazard bag

********The nurse is arranging home care for an older client who has a new colostomy following a large bowel resection three days ago. The client plans to live with a family member. Which actions should the nurse implement? (Select all that apply) A. Teach care of ostomy to care provider B. Assess the client for self care ability C. Provide pain medication instructions D. Request a home safety inspection E. Call home care agency to set up oxygen

A. Teach care of ostomy to care provider B. Assess the client for self care ability C. Provide pain medication instructions

*****The nurse is caring for a 24-month-old toddler who has sensory sensitivity, difficulty engaging in social interactions, and has not yet spoken two-word phrases. Which assessment should the nurse administer? A. The modified checklist for autism in toddlers (M-CHAT) B. Psychology Systems Questionnaire (PHQ-2) C. Behavioral Style Questionnaire (BSQ) D. The Ages and Stages Questionnaire (ASQ)

A. The Modified Checklist for Autism in Toddlers (M-CHAT)

*********The mother of a 2 day old infant girl expresses concern about a "flea bite" type rash on her daughter's body. The nurse identifies a pink papular rash with vesicles superimposed over the thorax, back, buttocks, and abdomen. Which explanation should the nurse offer? A. This is a common newborn rash that will resolve after several days B. The rash is due to distended oil glands that will resolve in a few weeks C. The healthcare provider is being notified about the rash D. This rash is characteristic of a medication reaction

A. This is a common newborn rash that will resolve after several days

****Prior to obtaining a trapeze bar for a client with limited mobility, which client assessment is most important for the nurse to obtain? A. Upper body muscle strength B. Balance and posture C. Risk for disuse syndrome D. Pressure sore risk

A. Upper body muscle strength

*****The nurse is developing an educational program for older clients who are being discharged with new antihypertensive medications. The nurse should ensure that the education materials include which characteristics? (Select all that apply) A. Uses common words with few syllables B. Printed using a 12-point type font C. Uses pictures to help illustrate complex ideas D. Contains a list with definitions of unfamiliar terms E. Written at a twelfth-grade reading level

A. Uses common words with few syllables C. Uses pictures to help illustrate complex ideas D. Contains a list with definitions of unfamiliar terms

*****A combination multi-drug cocktail is being considered for an asymptomatic HIV-infected client with a CD4 cell count of 500. Which nursing assessment of the client is most crucial in determining whether therapy should be initiated? A. Willing to comply with complex drug schedules B. Maintains an adequate social support system C. Qualifies for a prescription assistance program D. States various side effects of retroviral agents

A. Willing to comply with complex drug schedules

*************A nurse working on an Endocrine Unit should see which client first? A. An older client with Addison's disease whose current blood sugar level is 62 mg/dL B. A client taking corticosteroids who has become disoriented in the last two hours C. An adolescent male with type 1 diabetes who is arguing about his insulin dose D. An adult with a blood sugar of 384 mg/dL and a urine output of 350mL in the last hour

B. A client taking corticosteroids who has become disoriented in the last two hours

*******A client's morning assessment includes bounding peripheral pulses, weight gain of 2 pounds (0.91 kg), pitting ankle edema, and moist crackles bilaterally. Which intervention is most important for the nurse to include in this client's plan of care? A. Maintain accurate intake and output B. Administer prescribed diuretic C. Weigh client every morning D. Restrict daily fluid intake to 1500mL

B. Administer prescribed diuretic

*******A client who experienced a cerebrovascular accident (CVA) is aphasic and has left sided paralysis. Which nurse should be responsible for coordinating the progression of this client's care? A. Nurse case manager B. Adult nurse practitioner C. Neurology unit supervisor D. Risk management nurse

B. Adult nurse practitioner

*****A client diagnosed with calcium kidney stones has a history of gout. A new prescription for aluminum hydroxide is scheduled to begin at 0730. Which client medication should the nurse bring to the healthcare provider's attention? A. Esinapril B. Allopurinol C. Furosemide D. Aspirin, low dose

B. Allopurinol

****After administering a proton pump inhibitor (PPI), which action should the nurse take to evaluate the effectiveness of the medication? A. Auscultate for bowel sounds in all quadrants B. Ask the client about gastrointestinal pain C. Monitor the client's serum electrolyte levels D. Measure the client's fluid intake and output

B. Ask the client about gastrointestinal pain

******A preschool-aged boy is admitted to the pediatric unit following successful resuscitation from a near-drowning incident. While providing care to the child, the nurse begins talking with his preadolescent brother who rescued the child from the swimming pool and initiated resuscitation. The nurse notices the older boy becomes withdrawn when asked about what happened. Which action should the nurse take? A. Develop a water safety teaching plan for the family B. Ask the older brother how he felt during the incident C. Tell the older brother that he seems depressed D. Commend the older brother for his heroic actions

B. Ask the older brother how he felt during the incident

********The nurse is caring for a client who is still experiencing light sedation after undergoing an emergency colectomy for bowel obstruction. Which postoperative pain intervention should the nurse implement first? A. Review medical records to obtain pain tolerance expectations B. Attempt to obtain a self-report of pain level from the client C. Provide the first medication prescribed for pain management D. Wait until the client is awake before providing pain management

B. Attempt to obtain a self-report of pain level from the client

*******A male client approaches the nurse with an angry expression on his face and raises his voice, saying "My roommate is the most selfish, self-centered, angry person I have ever met and if he loses his temper one more time with me, I am going to punch him out!" The nurse recognizes that the client is using which defense mechanism? A. Splitting B. Projection C. Rationalization D. Denial

B. Projection

********When assessing an IV site that is used for fluid replacement and medication administration, the client complains of tenderness when the arm is touched above the site. Which additional assessment finding warrants immediate intervention by the nurse? A. Client uses the arm cautiously B. Red streak tracking the vein C. A sluggish blood return D. Spot of dried blood at insertion site

B. Red streaks tracking the vein

*********An older adult client with systemic inflammatory response syndrome (SIRS) has a temperature of 101.8F, heart rate of 110 beats/minute, and respiratory rate of 24 breaths/minute. Which additional finding is most important to report to the healthcare provider? A. Capillary glucose reading of 110 mg/dL B. Serum creatinine of 2.0 mg/dL C. Blood pressure of 130/88 mmHg D. Hemoglobin of 12 g/dL

B. Serum creatinine of 2.0 mg/dL

**********When conducting diet teaching for a client who is on a postoperative full liquid diet, which foods should the nurse encourage the client to eat? (Select all that apply) A. Cheese B. Tea C. Lentils D. Whole grain breads E. Potato soup

B. Tea C. Lentils E. Potato soup

*********The nurse identifies the presence of clear fluid on the surgical dressing of a client who just returned to the unit following lumbar spinal surgery. Which action should the nurse implement immediately? A. Change the dressing using a compression bandage B. Test the fluid on the dressing for glucose C. Document the findings in the electronic medical record D. Mark the drainage area with a pen and continue to monitor

B. Test the fluid on the dressing for glucose

*************The nurse is preparing a client who had a below-the-knee (BKA) amputation for discharge to home. Which recommendations should the nurse provide this client? (Select all that apply) A. Avoid range of motion exercises B. Use a residual limb shrinker C. Apply alcohol to the stump after bathing D. Inspect skin for redness E. Wash the stump with soap and water

B. Use a residual limb shrinker D. Inspect skin for redness E. Wash the stump with soap and water

**********The nurse is caring for a client who has chronic obstructive pulmonary disease (COPD) and chest pain related to a recent fall. What nursing intervention requires the greatest caution when caring for a client with COPD? A. Monitoring telemetry and cardiac rhythm B. Assisting client to cough and deep breath C. Administering narcotics for pain relief D. Increasing the client's fluid intake

C. Administering narcotics for pain relief

*****A client tells the nurse about working out with a personal trainer and swimming three times a week in an effort to lose weight and sleep better. The client states that it still is taking hours to fall asleep at night. Which action should the nurse implement? A. Advise the client that lifestyle changes often take several weeks to be effective B. Encourage the client to exercise everyday to eliminate bedtime wakefulness C. Ask the client for a description of the exercise schedule that is being followed D. Determine the amount of weight the client has lost since increasing activity

C. Ask the client for a description of the exercise schedule that is being followed

*********The nurse is preparing a discharge teaching plan for a client who had a liver transplant. Which instruction is most important to include in this plan? A. Increase activity and exercise gradually, as tolerated B. Limit intake of fatty foods for one month after surgery C. Avoid crowds for first two months after surgery D. Notify the healthcare provider if edema occurs

C. Avoid crowds for first two months after surgery

********An adolescent who was diagnosed with diabetes mellitus Type 1 at the age of 9, is admitted to the hospital in diabetic ketoacidosis. Which occurrence is the most likely cause of the ketoacidosis? A. Ate an extra peanut butter sandwich before gym class B. incorrectly administered too much insulin C. Had a cold and ear infection for the past two days D. Skipped eating lunch

C. Had a cold and ear infection for the past two days

********After an older client receives treatment for drug toxicity, the healthcare provider prescribes a 24-hour creatinine clearance test. Prior to starting the urine collection, the nurse notes that the client's serum creatinine is 0.3 mg/dL. Which action should the nurse implement? A. Evaluate the client's serum BUN level B. Initiate the urine collection as prescribed C. Notify the healthcare provider of the results D. Assess the client for signs of hypokalemia

C. Notify the healthcare provider of the results

*******The father of a 4-year-old has been battling metastatic lung cancer for the past 2 years. After discussing the remaining options with his healthcare provider, the client requests that all treatment stop and that no heroic measures be taken to save his life. When the client is transferred to the palliative care unit, which action is most important for the nurse working on the palliative care unit to take in facilitating continuity of care? A. Reassure the client that his child will be allowed to visit B. Provide the client written information about end-of-life care C. Obtain a detailed report from the nurse transferring the client D. Mark the chart with client's request for no heroic measures

C. Obtain a detailed report from the nurse transferring the client

*********The nurse should withhold which medication if the client's serum potassium level is 6.2 mEq/L? A. Metolazone B. Furosemide C. Spironolactone D. Hydrochlorothiazide

C. Spironolactone

*****After a spider bite on the lower extremity, a client is admitted for treatment of an infection that is spreading up the leg. Which admission assessment findings should the nurse report to the healthcare provider? (Select all that apply) A. Location of the initial IV site B. Red blood cell count (RBC) C. Swollen lymph nodes in the groin D. White blood cell count (WBC) E. Core body temperature

C. Swollen lymph nodes in the groin D. White blood cell count (WBC) E. Core body temperature

******A client with a prescription for "do not resuscitate" (DNR) begins to manifest signs of impending death. After notifying the family of the client's status, what priority action should the nurse implement? A. The impending signs of death should be documented B. The client's status should be conveyed to the chaplain C. The client's need for pain medication should be determined D. The nurse manager should be updated on the client's status

C. The client's need for pain medication should be determined

*****A client with hyperthyroidism is admitted to the postoperative unit after a subtotal thyroidectomy. Which of the client's serum laboratory values requires intervention by the nurse? A. T3- uptake at 50% B. Glucose 150 mg/dL C. Total calcium 5.0 mg/dL D. Thyroxine 12 mcg/dL

C. Total calcium 5.0 mg/dL

*********What might the nurse suggest to a client with fibrocystic breasts in an attempt to help relieve her symptoms? A. "Eliminate caffeine from your diet" B. "Avoid vigorous physical exercise immediately after your menstrual periods" C. "Eat a low-carbohydrate, high-protein diet" D. "Increase high-calcium foods in your diet"

D. "Increase high-calcium foods in your diet"

*******After reviewing the Braden Scale findings of residents at a long-term facility, the charge nurse should to tell the unlicensed assistive personnel (UAP) to prioritize skin care for which client? A. A poorly nourished client who requires liquid supplements B. An older adult who is unable to communicate elimination needs C. A woman with osteoporosis who is unable to bear weight D. A older man whose sheets are damp each time he is turned

D. A older man whose sheets are damp each time he is turned

***********The psychiatric nurse is caring for clients on an adolescent unit. Which client requires the nurse's immediate attention? A. A 14yo client with anorexia nervosa who is refusing to eat the evening snack B. A 16yo client diagnosed with major depression who refuses to participate in group C. A 17yo client diagnosed with bipolar disorder who is pacing around the lobby D. An 18yo client with antisocial behavior who is being yelled at by other clients

D. An 18yo client with antisocial behavior who is being yelled at by other clients

********The nurse is preparing a client for discharge who was hospitalized with an acute flare of systemic lupus erythematosus (SLE) symptoms. Which instruction is most important for the nurse to include? A. Use a walker when weakness occurs B. Take prescribed cortisone accurately C. Decrease daily intake of sodium in diet D. Avoid extreme environmental temperatures

D. Avoid extreme environmental temperatures

*******A client is admitted with a diagnosis of urolithiasis. Which finding is most important for the nurse to report to the healthcare provider? A. Volume of each voiding is more than 300mL B. Serum potassium that is elevated C. Relief of flank pain that radiated into the groin D. Hematuria that is beginning to turn pink

D. Hematuria that is beginning to turn pink

*****An older male client, who is a retired chef, is hospitalized with a diabetic ulcer on his foot. His daughter tells the nurse that her father has become increasingly obsessed with the way his food is prepared in the hospital. The nurse's response should be based on what information? A. The client probably has an organic brain disease and will likely have Alzheimer's disease within a few years B. The family needs a social worker to talk to them about how to handle their father when he becomes annoying C. The daughter is under stress and should be encouraged to think about happier times D. If the client was compulsive about food when he was younger, the aging process can magnify this

D. If the client was compulsive about food when he was younger, the aging process can magnify this

******A client at 12 weeks gestation is admitted to the antepartum unit with a diagnosis of hyperemesis gravidarum. Which action is most important for the nurse to implement? A. Obtain the client's 24-hour dietary recall B. Document mucosal membrane status C. Schedule a consult with a nutritionist D. Initiate prescribed intravenous fluids

D. Initiate prescribed intravenous fluids

*********A client who is admitted with complications related to hypopituitarism is diaphoretic and hypotensive. Which assessment finding warrants immediate intervention by the nurse?

Lethargy

The nurse supplies a blood pressure cuff around a client's left thigh. To measure client's pressure, where should the diaphragm of the stethoscope be placed? (Mark location on image)

On left thigh with arrow pointing to inner thigh Posterior left calf

An infant is unresponsive and gasping for breath. Prior to start CPR, which site should the nurse palpate for a pulse?

To perform a pulse check in an unresponsive infant, palpate a brachial pulse.

***********The nurse is assigning rooms for four clients, each newly diagnosed, and being admitted to the acute neuro unit for treatment. The client with which condition should be assigned the only private room available? A. Bacterial meningitis B. Viral encephalitis C. Septic shock D. Brain abscess

a

***********The nurse is preparing to gavage feed a premature infant through an orogastric tube. During insertion of the tube, the infant's heart rate drops to 60 beats/minute. Which action should the nurse take? A. Postpone the feeding until the infant's vital signs are stable B. Continue the insertion since this is a typical response C. Insert the feeding tube into the infant's nasal passage D. Pause and monitor for a continued drop of the heart rate

a

***********The nurse is preparing to send a client to the cardiac catheterization lab for elective cardioversion. Which intervention should the nurse implement before the client leaves the medical unit? A. Document that the client has remained NPO B. Confirm monitor reading in synchronous mode C. Notify the rapid response team of the transfer D. Secure cardioversion pads on the client's chest

a

***********Which statement by a client who is 24 hours post-subtotal thyroidectomy requires an immediate investigation by the nurse? a. "When I get out of bed quickly, I feel a little dizzy." b. "The dressing over my incision feels like it is too tight." c. "I'm most comfortable when the head of the bed is raised." d. "This IV infusion makes me urinate more often than usual."

a

Identify the conductive structure that is visualized with an otoscope. (Click the chosen location to change, click on the new location)

beginning of ear canal

***********In monitoring tissue perfusion in a client following an above the knee amputation (AKA), which action should the nurse include in the plan of care? A. Assess skin elasticity of the stump B. Observe for swelling around the stump C. Note amount and color of wound drainage D. Evaluate closest proximal pulse

d


Kaugnay na mga set ng pag-aaral

PrepU: Chapter 11: Maternal Adaptation During Pregnancy

View Set

BIO 2006 Chapter 22 Smartbook Questions

View Set

MKT 3013 Sample Quiz & Test, Ch 17 (Final Exam)

View Set

OB Passpoint Quizzes, Exam #1 Review

View Set

COMMUNICATION FINAL EXAM Chapter 10-17

View Set